Sunteți pe pagina 1din 90

Anul IV, Nr.

1 Ianuarie Iunie 2002


RECREATII
MATEMATICE
REVI S T| DE MATEMATI C| PENTRU ELEVI { I PROFES ORI
E d i t u r a Cr e n g u a G l d u
I AI , 2002
e
iS
= 1

Anul IV, Nr. 1 Ianuarie - Iunie 2002


RECREATI I
MATEMATICE
REVI S T| DE MATEMATI C| PENTRU ELEVI { I PROFES ORI
Apare cu sprijinul
FILIALEI IAI a SOCIETII de TIINE MATEMATICE
IASI, 2002
e
iS
= 1

Semnificaia formulei de pe copert:


ntr-o form concis, formula 1
S i
e leag cele patru ramuri fundamentale ale
matematicii:
ARITMETICA reprezentat de 1
GEOMETRIA reprezentat de S
ALGEBRA reprezentat de i
ANALIZA MATEMATIC reprezentat de e
Redacia revistei :
Petru ASAFTEI , Temistocle BRSAN, Dan BRNZEI, Ctlin CALISTRU,
Constantin CHIRIL, Constantin COCEA, Eugenia COHAL, Adrian CORDUNEANU,
Paraschiva GALIA, Mihai GRTAN, Paul GEORGESCU,
Dumitru GHERMAN (Pacani), Gheorghe IUREA, Lucian Georges LDUNC,
Gabriel MRANU, Gabriel POPA, Dan POPESCU (Suceava),
Florin POPOVICI (Braov), Maria RACU, Petru RDUCANU, Alin SPUM
Coordonatorul numrului : Temistocle BRSAN
Tehnoredactare computerizat:
Ionu AILINCI, Ctlin CALISTRU, Paul GEORGESCU, Gabriel POPA,
Vlad ROTARIU i un grup de elevi de la Liceul Teoretic Gr. Moisil
i Liceul Teoretic G. Ibrileanu
Adresa redaciei:
Catedra de Matematic Universitatea Tehnic Gh. Asachi Iai
Bd. Carol I, nr.11, 6600, Iai
Tel. 032 213737 / int. 123
E-mail: acord@math.tuiasi.ro
EDITURA CRENGUA GLDU
Toate drepturile rezervate
ISSN 1582 - 1765
Bd. N. Iorga, Bl. K2, ap. 4
Tel. / Fax: 032 - 230598
IAI, 6600
1
Alin Spum
Cu mult tristee i prere de ru anunm dispariia n luna decembrie, anul 2001,
a unuia dintre cei mai ataai i entuziati membri ai redaciei revistei noastre.
S-a nscut n Iai , la 22 noiembrie 1974. n acest ora drag lui, a parcurs n mod
strlucit toate treptele nvmntului, de la cel elementar i pn la cel superior: a absolvit
Liceul de informatic Gr. Moisil i a obinut licena n matematic la Universitatea Al.
I. Cuza. A ocupat prin concurs un post de preparator la Catedra de algebr, Facultatea de
matematic a universitii ieene.
Pasionat de matematic, s-a ndreptat spre domeniul Algebrei abstracte i
aplicaiilor acesteia: dup doi ani de masterat, s-a nscris la doctorat i a susinut examenele
i referatele programate. Lucra la teza de doctorat i urma s plece n Germania, beneficiind
de o burs de studii, n momentul dispariiei sale.
Timpul a pus capt irului de proiecte n care se avnta i pentru realizarea crora
era att de nzestrat.
A muncit cu mult pasiune i pricepere i a obinut de timpuriu frumoase rezultate.
Ca elev, a participat la fazele finale ale Olimpiadelor de matematic, Concursurilor
rezolvitorilor Gazetei Matematice i multor concursuri interjudeene i a fost rspltit cu
premii i meniuni. Ca student, a fost premiat la mai multe ediii ale Concursului studenesc
Traian Lalescu i la Sesiunile de comunicri tiinifice studeneti. Cadru didactic n
nvmntul superior fiind, a participat cu regularitate la Conferinele naionale de Algebr
din ultimii ani.
A dorit mult ca Iaul s aib o revist de matematic elementar i chiar a avut o
iniiativ personal n acest sens. ntors n ar, dup o perioad de studii la Universitatea
din Udine (Italia), s-a ataat imediat de revista Recreaii matematice, care n acel moment
pregtea apariia primului su numr.
Cu competen i fr a-i menaja energia i timpul, a contribuit la creterea
calitii revistei cu fiecare numr nou aprut. Priceperea sa n tehnoredactarea pe calculator
a fost o ans a revistei la primele sale apariii. A fost neobosit n promovarea i distribuirea
revistei, folosind orice prilej pe care-l avea: concursuri, tabere de matematic etc.
Era att de tnr i era o prezen att de vie printre noi, membrii redaciei i
colaboratorii revistei, nct vestea dispariiei lui a fost primit ca un fapt absurd i
incredibil.
Va rmne mereu n amintirea noastr ca un exemplu de pasiune i druire,
hrnicie i modestie, entuziasm i competen.
i nu avem alt mod mai potrivit de a cinsti memoria celui ce a fost Alin Spum,
dect asigurnd apariia revistei, la care a inut att de mult, fcnd-o ct mai interesant i
atractiv.
REDACIA REVISTEI
Alin Spum
Petre Osmatescu ( 1925 2001 )
Petre Osmatescu s-a nscut n Romnia la 20 iunie 1925 n satul Vdeni, comuna
Moldova, judeul Cetatea Alb. Ca o nedreptate istoric, oraul omonim din antichitate,
nfiinat n secolul VI nainte de Christos sub denumirea de Tyras i fcnd parte din
Moldova lui tefan cel Mare, este rebotezat astzi Bielgorod Dnestrovski, iar comuna
Moldova - Krutoiarovska i aparin amndou Ucrainei. coala primar a terminat-o n
comuna natal n 1938 (care, la acea dat, mai aparinea Romniei).
n perioada 1945-1947, urmeaz cursurile pregtitoare de pe lng Institutul
Pedagogic Ion Creang din Chiinu i apoi (1947-1951) cursurile institutului propriu-
zis, ncepndu-i cariera ca profesor de matematic i fizic la coala Medie nr. 2 din Orhei
(1951-1956), unde este ales preedintele seciei metodice de fizic i matematic (1954-
1956). n 1956, este transferat la Institutul Pedagogic din Tiraspol.
n 1962, nfiineaz la Tiraspol Seminarul de Topologie General , Geometrie
Analitic i Funcii Reale pentru studeni i tinerele cadre. n 1964, i susine teza de
doctorat n matematic i fizic .
n 1965, devine confereniar, iar n 1979 profesor la Catedra de matematici
superioare nr. 1 de la Institutul Politehnic din Chiinu. Cu aceast ocazie, introduce
metodele analizei vectoriale n cursul de analiz matematic. Tot n 1965, nfiineaz i
conduce Seminarul de Topologie General de la Universitatea de Stat din Moldova. El
este membru al Societii Matematice din Moldova i Preedinte al Societii East-West.
A fost numit n 1996 Persoan Eminent a Educaiei Publice i a primit
Mileniul 2000. Medalie de Onoare. El a fost fondatorul i organizatorul celor apte
Simpozioane Tiraspoliene de Topologie i Aplicaii, pe care le-a editat. Cercetrile i
rezultatele sale sunt de Topologie general i Teoria funciilor .
A inut conferine n Canada, Egipt, Elveia, Germania, Romnia, Spania i
Turcia. Un ntreg ciclu de conferine l-a inut la Universitatea Tehnic din Iai, cu care s-a
semnat i un acord de cooperare interuniversitar.
O maladie necrutoare a ntrerupt firul vieii sale n noiembrie 2001, punnd astfel
capt unei activiti prodigioase.
Regretatul Profesor Petre Osmatescu , al crui prieten am fost, s-a impus pe
dou fronturi, care se ntreptrund : pe de o parte, pe plan matematic, n topologie, prin
introducerea spaiilor subtile , unde elementul de baz este diviziunea punctului (ceva
analog cu divizarea atomului din fizic), teorie care i-a gsit aplicaii n economie, finane
i sociologie, ct i prin nfiinarea la Chiinu a revistei Scripta Scientiarum
Matematicarum, pe care a condus-o pn n ultima clip, iar pe de alt parte, pe plan
romnesc, s-a dedicat luptei, prin mijloace culturale, pentru reunificarea Basarabiei cu
Romnia, deziderat, care sperm s nu rmn doar un vis al nostru.
Prof. dr. Petru CARAMAN
4
Niels Henrik Abel 200 de ani de la natere
A fost una dintre fiinele de excepie ce natura
rareori d natere n cursul unui secol.
August Leopold Crelle
N. H. Abel (1802-1829) s-a nscut la Finde ntr-o familie cu frumoase tradiii.
Moartea timpurie a tatlui i falimentul bunicului su au dus familia la o existen precar.
Se cunosc puine lucruri despre Abel. Ocupaiile lui favorite au fost teatrul i matematica.
La 19 ani, student fiind, N. H. Abel propune o rezolvare algebric a ecuaiei de gradul
a cincilea. Aceasta coninea o eroare pe care tot el o gsete doi ani mai trziu. n
Mmoires sur les l'quations algbriques o on dmontre l'impossibilit de la rsolution
de l'quation gnrale du cinquime degr (1824), Abel pune capt ncercrilor de
continuare a lucrrilor lui Tartalia, Cardano i Ferrari, ce au durat mai bine de un secol.
n 1825 face o cltorie de studii n Germania, Frana i Italia. La Berlin cunoate pe
A. L. Crelle, fondatorul revistei Journal fr die reine und angewandte Mathematik. Multe
din lucrrile lui Abel au fost publicate n aceast revist.
n iulie 1826 se afl la Paris, unde cunoate pe cei mai importai membri ai Academiei
de tiine: Cauchy, Laplace, Legendre .a. Pregtete i depune la acest nalt for de tiin
cea mai important lucrare a sa Mmorie sur une proprit gnrale dune classe trs
tendue des fonctions transcendentes. A ateptat zadarnic un rspuns din partea Academiei
de tiine, n particular din partea lui Cauchy i Legendre. Aici, n Paris, apar primele
semne ale bolii sale. Rspunsul mult ateptat vine la cteva luni dup moartea sa. n 1830 i
s-a acordat postum, pentru aceast lucrare, marele premiu al Academiei de tiine
(mpreun cu Jacobi). Memoriul a disprut pentru un timp i a fost publicat abia n 1841.
Din Paris se ntoarce acas unde i triete ultimii doi ani de via mpovrat de griji.
Moare la numai 26 de ani, rpus de tuberculoz, boal incurabil n acea vreme.
n cinci ani de creaii originale i profunde, Niels Henrik Abel a reformat o parte a
analizei matematice i a deschis drumuri noi n teoria grupurilor i n geometria analitic i
algebric. A adus contribuii n teoria ecuaiilor algebrice i ecuaiilor abeliene, teoria
seriilor binomiale i a seriilor n general, teoria funciilor eliptice i, mai general, a
funciilor algebrice. Un numr mare de noiuni i teoreme importante poart numele lui :
integrale abeliene, grupuri abeliene, teorema Abel-Ruffini privind imposibilitatea rezolvrii
prin radicali a ecuaiilor algebrice de grad mai mare dect patru etc.
Recent guvernul norvegian a hotrt crearea unui premiu care s poarte numele
marelui matematician, Premiul Abel. Acesta este destinat domeniului matematicilor i se
acord anual ncepnd cu anul 2002. Valoarea premiului este de 200.000.000 coroane
norvegiene ( 27.100.000 euro). Crearea unui Premiu Abel a fost propus i n 1902 de ctre
regele Oscar II al Suediei i Norvegiei. Datorit separrii celor dou ri n 1905, proiectul
a fost abandonat. Pn acum, cel mai prestigios premiu pentru matematicieni a fost medalia
Fields, acordat din patru n patru ani, ncepnd din 1936. Premiul Abel este ca valoare i
importan comparabil cu Premiul Nobel (ce nu se acord matematicienilor).
Prof. dr. Temistocle BRSAN
5
ARTICOLE SI NOTE MATEMATICE
Asupra ipotezei lui Goldbach
Petru MINU
1
Una din problemele care au impulsionat considerabil dezvoltarea teoriei numerelor
i care nu este nc rezolvat, n ciuda eforturilor fcute n ultimii 250 de ani de
matematicieni dintre cei mai renumii, este aa numita ipotez a lui Goldbach.
Problema a fost pus pentru prima dat n corespondena dintre Christian
Goldbach i Leonhard Euler, la vremea respectiv matematicieni la Academia din
Sankt Petersburg. La 7 iunie 1742, Goldbach i scrie lui Euler: Evident, orice numr
(natural) este suma a trei numere prime. La 30 iunie 1742, Euler i rspunde:
Consider ca o teorem pe deplin adevrat c orice numr par este suma a dou
numere prime, dei nu pot s o demonstrez. Prin tradiie s-a pstrat sub denumirea de
ipoteza lui Goldbach urmtoarea afirmaie:
Propoziia 1. Orice numr natural par, mai mare ca 2 , este suma a dou numere
prime (de exemplu, 2 + 2 = 4 , 3 + 3 = 6 , 5 + 3 = 8 , 5 + 5 = 7 + 3 = 10 etc.).
A fost formulat i o ipotez mai tare:
Propoziia 2. Orice numr natural par, mai mare ca 6, este suma a dou numere
prime diferite.
Propoziia 2 a fost verificat de Pipping pentru toate numerele pare pn la 100000.
Teorema 1. Propoziia 2 este echivalent cu afirmaia: orice numr natural mai
mare ca 17 este suma a trei numere prime diferite.
Demonstraie. Observm c dac n se reprezint sub forma r q p n + + = , r q p , ,
prime, n cazul cnd n este impar toate trei numerele r q p , , sunt impare, iar n cazul
cnd n este par unul dintre ele este par (deci 2) i celelalte dou impare.
S mai observm c nu exist numere N e n care s admit patru reprezentri de
forma:
1
3 3 r n + + = ,
2
5 5 r n + + = ,
3
7 7 r n + + = ,
4
11 11 r n + + = (
4 3 2 1
, , , r r r r prime).
ntr-adevr, n caz contrar ar rezulta c 16
4 1
+ = r r , 12
4 2
+ = r r , 8
4 3
+ = r r . Ca
urmare, dac
4
r este de forma k r 3
4
= , atunci
2
r este multiplu de 3; dac 1 3
4
+ = k r ,
atunci
3
r este multiplu de 3 i dac 2 3
4
+ = k r , atunci
1
r este multiplu de 3, absurd.
Presupunem c Propoziia 2 este adevrat fie 17 > n , n impar. Numerele pare
3 n , 5 n , 7 n , 11 n sunt mai mari ca 6 i se scriu sub forma unei sume de dou
numere prime diferite:
1 1
3 q p n + = ,
2 2
5 q p n + = ,
3 3
7 q p n + = ,
4 4
11 q p n + = .
Conform observaiei de mai sus, din aceste patru reprezentri ale lui n ca sume de trei
numere prime exist cel puin una n care toi termenii sunt diferii.
Dac n este par, 2 n este par i se poate scrie sub forma q p n + = 2 , unde p
i q sunt dou numere prime impare diferite, deci q p n + + = 2 .
1
Prof. dr. , Catedra de algebr, Univ. Al. I. Cuza, Iai
6
Invers, presupunem c orice numr mai mare ca 17 este suma a trei numere prime
diferite. Dac n este par, 15 > n , avem 2 + n par , 17 2 > + n , q p n + + = + 2 2 , deci
q p n + = , p i q numere prime diferite. Pentru 15 6 s < n avem: 8=3+5, 10=3+7,
12=5+7, 14=3+11. Cu aceasta demonstraia este complet.
Teorema 2. Dac ipoteza lui Goldbach este adevrat, orice numr natural impar,
mai mare ca 7, este suma a trei numere prime impare.
Demonstraie. Dac 7 1 2 > + n , ( ) ( ) 4 1 2 3 1 2 > = + n n i conform ipotezei lui
Goldbach ( ) q p n + = + 3 1 2 , q p, prime, impare. Deci q p n + + = + 3 1 2 .
Observaie. I.M. Vinogradov a demonstrat, n 1937, c orice numr natural impar
mai mare ca
16
3
3 este sum de trei numere prime.
Considerm propoziia:
Propoziia 3. Orice numr natural impar, mai mare ca 7, este sum de trei numere
prime impare.
Pentru a arta c Propoziia 3 este o teorem (propoziie adevrat) ar trebui
verificat c orice numr natural impar n,
16
3
3 7 < < n se poate scrie ca o sum de trei
numere prime impare.
Teorema 3. Dac ipoteza lui Goldbach este adevrat, atunci orice numr ntreg
impar n se poate reprezenta, ntr-o infinitate de moduri sub forma r q p n + = , unde
r q p , , sunt numere prime.
Demonstraie. Fie n un numr ntreg impar. Putem alege, ntr-o infinitate de
moduri, un numr prim impar r astfel nct 4 > + r n . Conform ipotezei lui Goldbach
exist dou numere prime impare p i q astfel nct q p r n + = + .
Teorema 4. Orice numr natural mai mare ca 11 este suma a dou numere
compuse.
Demonstraie. Dac n este par, 4 n este par (deci compus) i ( ) 4 4 + = n n .
Dac n este impar, 9 n este par (deci compus) i ( ) 9 9 + = n n .
Observaie. G.H. Hardy i J.E. Littlewood au formulat ipoteza c orice numr
natural n, suficient de mare este suma unui numr prim i a unui ptrat :
2
k p n + = , p
prim, N e k . Ipoteza nu a putut fi nc confirmat sau infirmat. O alt ipotez a lui
Hardy i Littlewood a devenit teorem prin demonstraia dat de I.V. Linnik n 1959 :
Teorema 5. Orice numr natural, suficient de mare, este suma ntre un numr
prim i dou ptrate:
2 2
h k p n + + = , p prim , N e h k, .
Demonstraia acestei teoreme nu poate fi fcut cu mijloace elementare.
Bibliografie
1. D. A. Buhtab Teoria cisel, Moskva, 1960.
2. C. Creang, C. Cazacu, P. Minu, Gh. Opai, C. Reischer Introducere n teoria
numerelor, Editura Didactic i Pedagogic, Bucureti , 1965.
3. P. Minu Teoria numerelor. Capitole introductive, Ed. Crengua Gldu, Iai, 1997.
4. W. Sierpinski Ce tim i ce nu tim despre numerele prime, Ed. tiinific, Bucureti,
1966.
7
Generalizarea teoremei de omologie a lui Barbilian
Constantin COCEA
1
Strlucitul matematician Dan Barbilian a demonstrat c dou triunghiuri echilaterale,
de acelai centru, sunt triomoloage. Vom extinde n cele ce urmeaz acest rezultat. Are loc
urmtoarea
Teorem. Fie ABC un triunghi cu centrul cercului nscris I;
1 1 1
, , C B A punctele de
contact ale cercului nscris cu laturile, iar
2 2 2
C B A un triunghi avnd centrul cercului
circumscris n I i invers asemenea cu
1 1 1
C B A . Atunci dreptele
2
AA ,
2
BB ,
2
CC sunt
concurente (adic triunghiurile ABC i
2 2 2
C B A sunt omologice)
Demonstraie. Fie 0, a, b, c afixele punctelor C B A I , , , , iar C(I,1) cercul nscris de
raz unitate. Atunci:
1
1 1 1
= = = c b a (1)
Triunghiul
2 2 2
C B A fiind invers asemenea cu
1 1 1
C B A , cu centrul cercului
circumscris I , rezult c exist e C nct
1 2
a a = ,
1 2
b b = ,
1 2
c c = (2)
n loc s demonstrm concurena dreptelor
2
AA ,
2
BB ,
2
CC , vom demonstra (avnd
n vedere teorema lui Desargues) c punctele { }
2 2
C B BC = o , { }
2 2
A C CA = | i
{ }
2 2
B A AB = sunt coliniare.
Panta complex a dreeptei IA
1
este
1
1
1
a
a
k
IA
= . Cum BC este tangent cercului nscris,
rezult c ecuaia lui BC este
(BC) ( )
1
1
1
1
a z
a
a
a z = sau 0 2
1 1
= + a z a z . (3)
Se tie c raportul n care o dreapt de ecuaie 0 = + + | o z z mparte un segment
(BC), afixele lui B i C fiind b respectiv c, este
| o
| o
+ +
+ +
=
c c
b b
R .
Prin urmare, innd seama de (2) i (3), avem :
.
2
2
1 1 1 1
1 1 1 1
2
2
+
+
=
a c a c
a b a b
C
B


o
o
(4)
Avnd n vedere expresiile analoage cu (4) ale rapoartelor
2
2
A
C
|
|
i
2
2
B
A

, obinem:

1
Profesor , Liceul Teoretic D.Cantemir, Iai
8
1
2
2
2
2
2
2
=
B
A
A
C
C
B

|
|
o
o
ceea ce probeaz coliniaritatea punctelor | o , , , deci triunghiurile ABC i
2 2 2
C B A sunt
omologice.
Observaie. Dac ABC este echilateral , I este centrul comun al triunghiurilor ABC i
1 1 1
C B A i se obine:
Teorema lui BARBILIAN. Dou triunghiuri echilaterale de acelai centru, ABC i
2 2 2
C B A , sunt n trei moduri omologice:
|
|
.
|

\
|
2 2 2
C B A
C B A
,
|
|
.
|

\
|
2 2 2
A C B
C B A
,
|
|
.
|

\
|
2 2 2
B A C
C B A
.
Demonstraia decurge din teorema anterioar, deoarece dou triunghiuri echilaterale
sunt n trei moduri invers asemenea.
Observaie. Teorema se poate extinde i astfel:
Teorem. Fie ABC un triunghi cu
a
I centrul cercului exnscris corespunztor laturii
BC, iar
1
A ,
1
B ,
1
C punctele de contact ale acestui cerc exnscris cu laturile BC, CA
respectiv AB. Fie
2 2 2
C B A un triunghi invers asemenea cu triunghiul
1 1 1
C B A avnd
centrul cercului circumscris n
a
I . Atunci triunghiurile ABC si
2 2 2
C B A sunt omologice.
Consecina 1. Fie ABC un triunghi, iar
1
A ,
1
B ,
1
C punctele de contact ale cercului
nscris C (I,r) cu laturile. Paralelele prin
1
B i
1
C la BC retaie cercul C n
2
B ,
2
C . S
se arate c dreptele
1
AA ,
2
BB ,
2
CC sunt concurente.
Demonstraie. Triunghiurile
1 1 1
C B A i
2 2 2
C B A au acelai centru al cercului
circumscris (punctul I) i sunt invers egale, deci invers asemenea.
Consecina 2. Fie ABC un triunghi, iar
1
A ,
1
B ,
1
C punctele de contact ale cercului
nscris C (I,r) cu laturile. Notm cu
2
A ,
2
B ,
2
C simetricele punctelor
1
A ,
1
B ,
1
C fa de
un diametru oarecare al cercului C . Atunci dreptele
2
AA ,
2
BB ,
2
CC sunt concurente.
Demonstraie. Centrul cercului circumscris triunghiului
2 2 2
C B A este I, iar
triunghiurile
2 2 2
C B A i
1 1 1
C B A sunt invers egale, deci invers asemenea.
Bibliografie
1. D. Barbilian - I. Barbu - Pagini inedite, Editura Albatros, Bucureti, 1981.
2. C. Cocea - Proprieti remarcabile ale triunghiurilor invers asemenea, S nelegem
matematica, Bacu, 1992.
3. C. Cocea - Teoreme de triortologie i triparalelogie, 1992.
4. P. S. Modenov - Probleme de geometrie , Editura Nauka, Moscova, 1979.
5. N. Mihileanu - Utilizarea numerelor complexe n geometrie, Ed. Tehnic, Bucureti,
1968.
9
Asupra unor iruri de integrale
Iuliana GEORGESCU
1
i Paul GEORGESCU
2
Articolul de fa prezint un mod de calcul al limitelor unor iruri ( )
1 > n n
x de termen
general ( )
)
=
b
a
n
n
dx x f x , cu f satisfcnd anumite ipoteze ce vor fi precizate ulterior. n
particular, se pot determina limitele irurilor ( )
1 > n n
a , ( )
1 > n n
b , ( )
1 > n n
c , ( )
1 > n n
d de termen
general
)
=
2 /
0
sin
t
xdx a
n
n
,
)
=
2 /
0
cos
t
xdx b
n
n
,
)
=
2 /
0
tg
t
xdx a
n
n
,
)
=
e
n
n
xdx a
1
ln .
Insistm mai nti asupra unei soluii eronate date n [2] pentru faptul c 0 lim =

n
n
a .
Soluie. Aplicnd teorema de medie, ( ) 2 / , 0 t e -c astfel nct
)
=
2 /
0
sin
t
xdx
n
c
n
sin 2 = t . Deoarece ( ) 1 , 0 sin e c , avem 0 sin lim =

c
n
n
i, deci, . 0 sin lim
2 /
0
=
)

t
xdx
n
n
Totui, c nu este constant, ci depinde de n , i nu putem deduce c . 0 sin lim =

c
n
n
n cele ce urmeaz vom indica un mod de calcul al unor limite de acest tip.
Teorema 1. Dac | | | | 1 , 0 , : b a f este continu, iar ( ) | | ( ) { } 1 ; , = e = x f b a x f U este
finit, atunci ( )
)
=

b
a
n
n
dx x f . 0 lim
Demonstraie. Presupunem c ( ) { }
k
x x x f U ! , ,
2 1
= i fie 0 > c arbitrar, dar fixat.
Considerm ( )
i i i
b a I , = cu
i i
I x e i l ( ) ( ) 1 / + < k I
i
c , k i , 1 = i l ( )
i
I = lungimea
intervalului
i
I (daca a x =
1
sau b x
k
= , atunci | )
1 1 1
, b a I = , respectiv ( |
k k k
b a I , = ).
Fie | |
*
k
i
i i
b a D
1
1
,
=
= i | | ( )
*
k
i
i i
b a b a D
1
2
, \ ,
=
= . Cum
2
D este o reuniune finit de
intervale nchise i mrginite, i f este continu, exist ( ) x f M
D x
2
sup
e
= , iar 1 < M . De aici,
rezult c ( ) ( )
)
s
2
2 /
D
n n
M dx x f t . Deoarece ( ) 1 s x f , | | b a x , e , se deduce c
( ) ( )
_
)
=
s
k
i
i i
D
n
a b dx x f
1
2 /
1
t . n consecin , ( ) ( ) ( ) 2 / 1 / t c
n
b
a
n
M k k dx x f + + s
)
.
1
Profesor, Liceul cu Program Sportiv, Iai
2
Profesor, Liceul Teoretic Grigore Moisil, Iai
10
Alegnd acum ( ) c
0
n astfel nct ( ) ( ) ( ) c c t
0
, 1 / 2 / n n k M
n
> + < , se obine c
( ) ( ), ,
0
c c n n dx x f
b
a
n
> <
)
deci ( )
)
=

b
a
n
n
dx x f . 0 lim
Observaia 1. Ipoteza ( ) f U este finit din enun poate fi nlocuit cu ( ) f U este
Jordan neglijabil, teorema rmnnd valabil cu aceeai demonstraie.
Corolar 1. Dac | | | | 1 , 0 , : b a f este continu i strict cresctoare (sau strict
descresctoare), iar ( ) 1 = b f (respectiv ( ) 1 = a f ), atunci ( )
)
=

b
a
n
n
dx x f . 0 lim
Demonstraie. Suntem n ipotezele teoremei , cu ( ) } {b f U = , respectiv ( ) } {a f U = .
Aplicaia 1. Are loc relaia 0 lim lim lim lim = = = =

n
n
n
n
n
n
n
n
d c b a .
Soluie. Se aplic Corolarul 1, inndu-se seama de monotonia funciilor de sub
semnul de integral.
n cele ce vor urma vom nota cu ( ) f D mulimea punctelor de discontinuitate ale unei
funcii f . Observm c este valabil deasemenea urmtoarea mbuntire a Teoremei 1.
Teorema 2. Dac | | | | 1 , 1 , : b a f , iar ( ) f D i ( ) f U sunt Jordan-neglijabile,
atunci ( )
)
=

b
a
n
n
dx x f 0 lim .
Demonstraie. Deoarece f este integrabil Riemann,
n
f f , i
n
f sunt de asemenea
integrabile Riemann. Fie acum 0 > c arbitrar, dar fixat. Deoarece ( ) f D este Jordan-negli-
jabil ,
*
N e -k i ( )
k i i
I
s s 1
astfel nct ( )
i i i
b a I , = , ( )
*
k
i
k
I f D
1 =
c i ( )
_
=
<
k
i
i i
a b
1
2 / c .
Notm | |
*
k
i
i i
b a D
1
1
,
=
= , | | ( )
*
k
i
i i
b a b a D
1
2
, \ ,
=
= . Mai departe, ( ) 2 /
1
c s
)
dx x f
D
n
, i cum
n
f este continu pe
2
D , ( )
)
2
D
n
dx x f se poate majora cu ajutorul Teoremei 1. Rezult de
aici c ( )
)
=

b
a
n
n
dx x f 0 lim , deci ( )
)
=

b
a
n
n
dx x f . 0 lim
Aplicaia 2. Fie 0 > b , iar ( )
( |

=
e
=
0 , 0
, 0 ,
1
sin
x
b x
x x f . Atunci f este integrabil
Riemann, iar ( )
)
=

b
a
n
n
dx x f 0 lim .
11
Soluie. Se observ c | | b x f , 0 , 1 1 e s s i ( ) { } 0 = f D . Atunci f este
integrabil Riemann, conform criteriului Lebesgue de integrabilitate Riemann. Mai departe,
( ) ( ) { } N e + = k k f U ; 1 2 2 t este de msur Jordan nul i se poate aplica Teorema 2.
Observaia 2. Folosind noiuni de teoria msurii i integralei Lebesgue, mai precis
teorema lui Egorov, se poate demonstra urmtorul rezultat:
Teorema 3. Fie | | I b a f , : continu, | | R b a g , : integrabil Riemann i
( )
1 > n n
g un ir de funcii continue, R I g
n
: , astfel nct ( )( ) ( ) | | b a x x g x f g
n
, e D ,
i ( )( ) | | b a x M x f g
n
, , e s D . Atunci ( )( ) ( )dx x g dx x f g
b
a
b
a
n
) )
D .
Aplicaia 3. Pentru 0 = a , 2 t = b , ( )
| )

=
e
=
2 , 1
2 , 0 , 0
t
t
x
x
x g , ( )
n
n
x x g = , ( ) x x f sin = ,
remarcnd c sunt ndeplinite condiiile Teoremei 3, iar ( ) 0
2
0
=
)
dx x g
t
, obinem c
0 lim =

n
n
a . Analog deducem c 0 lim lim lim = = =

n
n
n
n
n
n
d c b .
Aplicaia 4 (V. Drul i I. Paralescu, Problema 24154, G.M. 7-8/1999 ). Calculai:
a) dx
x x
x
n
n
)
+ +

2
0
2 2
sin sin 1
2 sin
lim
t
b)
)
+ +

2
0
2
sin sin 1
1
lim
t
dx
x x
n
n
.
Soluie. Aplicm Teorema 3. Lum
a) 0 = a , 2 t = b , ( )
n
n
x x
x x
x g
+ +

=
2
2
1
1 2
, ( ) x x f sin = , ( )
| )

=
e
+
=
2 , 3 / 1
2 , 0 ,
sin 1
2 sin
2
t
t
x
x
x
x
x g ,
b) 0 = a , 2 t = b , ( )
n
n
x x
x g
2 2
1
1
+ +
= , ( ) x x f sin = , ( )
| )

=
e +
=
2 , , 3 1
2 , 0 , 1 1
2
t
t
x
x x
x g .
Bibliografie
1. D. M. Btineu et al. Primitive i integrale , Ed. Brchi, Timioara, 1998.
2. V. Schneider Culegere de probleme de analiz matematic, Ed. Hyperion, Craiova, 1993.
12
O generalizare a lemei lui Riemann
Dan POPESCU
1
i Florin POPOVICI
2
n aceast not sunt generalizate urmtoarele dou rezultate:
Teorema 1 (Lema lui Riemann [3]). Dac | | t 2 , 0 : f R este o funcie
integrabil Riemann, atunci irurile
*
2
0
sin ) (
N n
dx nx x f
e
|
.
|

\
|
)
t
i
*
2
0
cos ) (
N n
dx nx x f
e
|
.
|

\
|
)
t
sunt convergente i avem
) )

= =
t t 2
0
2
0
cos ) ( lim 0 sin ) ( lim dx nx x f dx nx x f
n n
.
Teorema 2 (Problema XII.19 [1]). Dac | | T f , 0 : R este o funcie continu i
| ) | ) , 0 , 0 : g este o funcie continu i periodic de perioad T, atunci
. ) ) ( ) ) (
1
) ( ) ( lim
0 0 0
|
.
|

\
|
|
.
|

\
|
=
) ) )

T T T
n
dx x g dx x f
T
dx nx g x f
Amintim mai nti un rezultat la care vom face referire.
Teorema 3[2]. Dac | | b a f , : R este o funcie integrabil Riemann i
| | | | b a d c , , : este o funcie bijectiv, derivabil i cu derivata integrabil Riemann,
atunci funcia ( ) ' D f este integrabil Riemann i are loc formula schimbrii de
variabil
dt t t f dx x f
b
a
b
a
) ( )) ( ( ) (
) (
1
) (
1

' =
) )

.
Corolar. Fie e o R* astfel nct funcia de gradul nti
| | | | b a d c , , :
,
| o + = x x) ( este o funcie bijectiv. Dac
| | b a f , :
R este o funcie integrabil
Riemann atunci funcia D f este integrabil Riemann i are loc formula
. )) ( ( ) (
) (
1
) (
1
dt t f dx x f
b
a
b
a
) )

o
Rezultatul principal este dat de
Teorema 4 (Lema lui Riemann generalizat). Dac | | T f , 0 : R este o funcie
integrabil Riemann i | ) , 0 : g R este o funcie periodic de perioad T, astfel nct
restricia
] , 0 [ T
g este integrabil Riemann, atunci avem:
. ) ) ( ) ) (
1
) ( ) ( lim
0 0 0
|
.
|

\
|
|
.
|

\
|
=
) ) )

T T T
n
dx x g dx x f
T
dx nx g x f (1)

1
Profesor, Colegiul Naional tefan cel Mare, Suceava,
2
Profesor, Liceul Teoretic N. Titulescu ,Braov
13
Demonstraie. Conform Corolarului, funcia dat de ) (nx g x , ] , 0 [ T xe este
integrabil Riemann. Urmeaz c irul
*
0
) ( ) (
N n
T
dx nx g x f
e
|
.
|

\
|
)
este corect definit.
Pentru orice e n N* fie
( ) T x x x
nn n n n
= < < < = = !
1 0
0 A
, unde
( ) { }. , 1 , 0 , n i
n
T
i x
ni
! e =
Conform primei teoreme de medie, pentru orice
{ } n i ! , 1 , 0 e
exist
{ } { } | | ] , [ ) ( sup , ] , [ ) ( inf
1 , 1 , ni i n ni i n ni
x x x x f x x x x f

e e e
astfel nct
( )
n
T
x x dx x f
ni i n ni
ni
x
i n
x
ni
= =

) 1 ,
1 ,
) ( (2)
Evident, avem
. ) ( ) ( ) ) ( ( ) ( ) ( ) ( ) (
1
1 ,
1
1 ,
1
1 ,
0
dx nx g dx nx g x f dx nx g x f dx nx g x f
n
i
ni
x
i n
x
ni
n
i
ni
x
i n
x
n
i
ni
x
i n
x
ni
T
_ ) _) _) )
=

+ = = (3)
Deoarece funcia g este periodic de perioad T i restricia
] , 0 [ T
g
este integrabil
Riemann, rezult c funcia g este mrginit, deci exist ( ) e , 0 M astfel nct
( ) . ) , 0 [ , ) ( e s x M x g Urmeaz c
( ). ) ( ) ( ) (
) ( ) ( ) ( ) ) ( (
1
1 ,
1
1 ,
1
1 ,
f s f S M dx x f M
dx nx g x f nx g x f
n n
n
i
ni
x
i n
x
ni
n
i
ni
x
i n
x
ni
n
i
ni
x
i n
x
ni
A A


s s
s s
_)
_
)
_
)
=

unde
) ( ), ( f s f S
n n
A A noteaz sumele Darboux superioar i inferioar relative la
n
A
.
De aici rezult c
. 0 ) ( ) ) ( ( lim
1
1 ,
=
_)
=


dx nx g x f
n
i
ni
x
i n
x
ni
n

(4)
innd cont de relaia (2) i de periodicitatea funciei g, se obin egalitile:
( )
. ) ( ) (
1
) ( ) (
1
) (
1
) (
1
) ( ) (
0 0
1
1 ,
0
1
0
1
) 1 (
1 1 1
1 ,
|
.
|

\
|
|
.
|

\
|
=
|
|
.
|

\
|
|
.
|

\
|
= =
= = =
) ) _ ) _ )
_ ) _ ) _ )
=

=
=

= =

T T
n
i
ni i n ni
T
n
i
T
ni
n
i
iT
T i
ni
n
i
n
T
i
n
T
i
ni
n
i
ni
x
i n
x
ni
dx x f dt t g
T
x x dt t g
T
dt t g
n
T
T
dt t g
n
dx nx g dx nx g


(5)
Din (3), (4) i (5) rezult c are loc (1).
Observaie. Teoremele 1 i 2 sunt particularizri ale Teoremei 4.
Bibliografie
1.D. Popescu - Problema XII.19, Recreaii matematice 1/2001, p. 77.
2.F. Popovici, M. Bencze - Asupra schimbrii de variabile in integrala Riemann,
G.M. metodic, 3/1996, 161-164.
3.Gh. Sirechi - Calcul diferenial i integral, vol I, Ed. tiinific i Enciclopedic,
Bucureti, 1985.
14
Cteva relaii metrice deduse vectorial
Marian TETIVA
1
Ne propunem s prezentm n cele ce urmeaz o modalitate de deducere a unor relaii
metrice n triunghi sau n tetraedru cu ajutorul calculului vectorial. Ideea (simpl, dar
eficient) am ntlnit-o n lucrarea [3] i este exprimat n
Propoziia 1. Fie K, M, L trei puncte necoliniare. Pentru un punct P oarecare,
urmtoarele afirmaii sunt echivalente:
a) Punctele L, M, P sunt coliniare;
b) Exist numerele reale l, m astfel nct l+m = 1 i KM m KL l KP + =
Demonstraie. eLM P R e -l astfel nct = ML l MP R e -l astfel nct
= ) ( KM KL l KM KP R e -l astfel nct KM l KL l KP ) 1 ( + = , q.e.d .
Mai departe vom rezolva cteva probleme folosind rezultatul din Propoziia 1.
ncepem cu problema din [3] care ne-a condus la aceast not.
Problema 1. Fie ABC un triunghi, iar E, D puncte situate pe laturile AB, respectiv AC.
Considerm punctul M, intersecia dreptelor BD i CE; P i N sunt interseciile dintre AM
i BC, respectiv AM i DE. S se arate ca are loc egalitatea:
MA
PM
NA
PN
2 = .
Soluie. S notm p=
NP
AN
n
MP
AM
m
DC
AD
q
EB
AE
= = = , , , . Cum BC Pe , exist
numerele y x, astfel nct x+y=1, i AC y AB x AP + = . Din EB p AE = rezult
imediat AB
p
p
AE
1 +
= ; la fel , ,
1
AC
q
q
AD
+
=
AP
m
m
AM
1 +
= i AP
n
n
AN
1 +
= . Egalitatea de
mai sus poate fi scris n urmtoarele forme
echivalente :
AD y
q
q
AB x AM
m
m 1 1 +
+ =
+
,
,
1 1 1
AD y
q
q
AE x
p
p
AN
n
n +
+
+
=
+
AC y AE x
p
p
AM
m
m
+
+
=
+ 1 1
. innd seama de unicitatea scrierii unui vector n bazele
) , ( ), , ( AC AE AD AE respectiv ) , ( AD AB i de Propoziia 1, obinem egalitile:
y
q
q
x
p
p
n
n 1 1 1 +
+
+
=
+
(1); y x
p
p
m
m
+
+
=
+ 1 1
(2); y
q
q
x
m
m 1 1 +
+ =
+
(3). Adunnd (2)
1
Profesor, Colegiul Naional Gh. Roca Codreanu, Brlad
15
cu (3) i innd seama de (1) i de faptul c x+y=1 , obinem c
n m
1 2
= , q.e.d.
Problema 2 (relaia lui Van Aubel). Cu notaiile din problema precedent, are loc
egalitatea
MP
AM
DC
AD
EB
AE
= + .
Soluie. Relaia (2) de mai sus se scrie: x x
p m
+
|
|
.
|

\
|
+ = + 1
1
1
1
1 , adic mx=p . Analog,
din (3) obinem my=q . Atunci
MP
AM
m y x m q p
DC
AD
EB
AE
= = + = + = + ) ( , q.e.d.
Problema 3 ([1]). Pstrm notaiile din Problema 1. n plus, considerm punctele T,
S pe laturile AB, AC astfel nct | o = =
SC
SA
TB
TA
, . S se arate c 1 = + e
| o
q p
TS M .
Soluie. Vom avea, ca mai sus, .
1
,
1
AC AS AB AT
+
=
+
=
|
|
o
o
Din relaia
AC y AB x AP + = rezult c AS y AT x AM
m
m
|
|
o
o 1 1 1 +
+
+
=
+
. Conform Propoziiei 1
i innd seama de unicitatea scrierii unui vector n baza ( ) AS AT, , avem:
| o | o | o |
|
o
o q p
m
q
m
p
m
y x
m
y x
m
m
TS M + = + = + =
+
+
+
=
+
e 1
1 1 1 1 1
, q.e.d.
Dou cazuri particulare mai des ntlnite ale Problemei 3 sunt acelea n care punctul M
este centrul de greutate G , respectiv centrul cercului nscris I pentru triunghiul ABC;
cititorul se poate convinge c este valabil urmtorul enun:
Problema 3. Fie ABC un triunghi i punctele T, S pe laturile AB, AC. Atunci:
a) ; 1 = + e
SA
SC
TA
TB
TS G
b) , a
SA
SC
c
TA
TB
b TS I = + e unde a, b, c reprezint lungimile laturilor triunghiului.
Iat n continuare i variantele n spaiul cu trei dimensiuni ale chestiunilor discutate
anterior; rezultatul similar celui din Propoziia 1 este coninut n:
Propoziia 2. Fie K, L, M, N patru puncte necoplanare. Un punct P este situat n
acelai plan cu L, M, N dac i numai dac exist numerele reale l, m, n astfel nct
l+m+n=1 i KN n KM m KL l KP + + = . Altfel spus, dac x,y,z sunt coordonatele vectorului
KP n baza, ( ) KN KM KL , , , atunci ( ) LMN Pe dac i numai dac x+y+z=1.
Demonstraia este asemantoare. Avem ( ) e LMN P R e - n m, astfel nct
+ = LN n LM m LP R , e - n m astfel nct ( ) ( ) + = KL KN n KL KM m KL KP
R e - n m l , , , l=1-m-n (deci cu l+m+n=1), astfel nct KN n KM m KL l KP + + = , q.e.d.
Folosind Propoziia 2 putem rezolva:
16
Problema 4 ([3]). Fie ABCD un tetraedru i F, G, H puncte pe AB, AC, AD respectiv;
fie M punctul comun planelor (FCD), (GBD), (HBC), iar P i N punctele de intersecie ale
dreptei AM cu planele (BCD) respectiv (FGH). Atunci are loc:
MA
PM
NA
PN
3 = .
Soluie. Ca n Problema 1, s notm
FB
AF
p = ,
GC
AG
q = ,
HD
AH
r = ,
MP
AM
m = ,
.
NP
AN
n = innd cont de unicitatea scrierii unui vector ntr-o baz i de Propoziia 2,
obinem mai nti existena numerelor reale z y x , , astfel nct x+y+z=1 i
AC z AC y AB x AP + + = , iar apoi echivalenele:
r
z
q
y
p
x
n
z
r
r
y
q
q
x
p
p
n
n
+ + =
+
+
+
+
+
=
+ 1 1 1 1 1
;
p
x
m
z y x
p
p
m
m
= + +
+
=
+ 1 1 1
;
q
y
m
z y
q
q
x
m
m
= +
+
+ =
+ 1 1 1
;
r
z
m
z
r
r
y x
m
m
=
+
+ + =
+ 1 1 1
. De aici
n r
z
q
y
p
x
m
1 3
= + + = , adic relaia din enun.
De asemenea vom avea , 1 m r q p
m
r q p
z y x = + +
+ +
= + + = deci am rezolvat i
Problema 5. Cu aceleai notaii din Problema 4 avem
MP
AM
HD
AH
GC
AG
FB
AF
= + + .
n ncheiere propunem cititorului s rezolve problemele :
Problema 6. Pstrm notaiile din Problemele 4 i 5; n plus fie punctele S, T, U pe
AB, AC, respectiv AD astfel nct . , , | o = = =
UD
AU
TC
AT
SB
AS
S se arate c:
( ) 1 = + + e
| o
r q p
STU M .
Caz particular. Centrul de greutate al tetraedrului se afl n planul (STU) dac i
numai dac 1 = + +
AU
UD
AT
TC
AS
SB
([4]).
Problema 7. Fie ABCD un patrulater convex i punctele M ,N, P, Q pe laturile sale
AB, BC, CD, DA respectiv, astfel nct . , b
QD
AQ
NC
BN
a
PC
DP
MB
AM
= = = = Fie S intersecia
dreptelor MP i NQ. S se calculeze rapoartele
SN
QS
i
SP
MS
(O frumoas soluie sintetic a
acestei probleme poate fi gsit n [2]).
Bibliografie
1. M. Andronache Problema 4 (p. 158) , G.M. 4/2000.
2. D. Brnzei, R. Brnzei Metodica predrii matematicii, Editura Paralela 45, 2000.
3. V.N. Dubrovski Soluia problemei M1062, Kvant 1/1988.
4. Gh. Szllsy Problema C: 2275, G.M. 4/2000.
17
Inegaliti geometrice. Aplicaii
Dan-tefan MARINESCU
1
i Ioan ERDEAN
2
Scopul propus este de a demonstra, prin mijloace mai puin folosite, dou
inegaliti geometrice care au un numr mare de aplicaii.
Propoziia 1. Fie neN, n>2 i M, A
1
, A
2
, , A
n
puncte din spaiu date. Dac a
1
,
a
2
, , a
n
eR, atunci
_ _ _
s < s = =
>
|
|
.
|

\
|
|
|
.
|

\
|
n j i
j i j i i
n
i
i
n
i
i
A A a a MA a a
1
2 2
1 1
. (1)
Egalitatea se obine dac i numai dac . 0
1
=
_
=
i
n
i
i
MA a
Demonstraie. Avem: 0
1 1
>
|
|
.
|

\
|

|
|
.
|

\
|
_ _
= =
n
i
i i
n
i
i i
MA a MA a , de unde
0 2
1 1
2 2
> +
_ _
s < s = n j i
j i j i
n
i
i i
MA MA a a MA a sau . 0 ) cos( 2
1 1
2 2
> +
.
s < s =
_ _ j i
n j i
j i j i
n
i
i i
MA A MA MA a a MA a
Cum din teorema cosinusului n MA
i
A
j
, eventual degenerat, avem
2 2 2
) cos( 2
j i j i j i j i
A A MA MA MA A MA MA + =
.
, urmeaz c
( )
_ _ _
s < s s < s =
> + +
n j i 1
2
i
n j i 1
2 2
i
1
2 2
j i j j i j
n
i
i i
A A a a MA MA a a MA a i, deci, are loc inegalitatea (1).
Observaie. Exist i alte demonstraii ale inegalitii (1) pentru cazul n care
punctele sunt coplanare i a
1
, a
2
, , a
n
>0.
Urmtorul caz particular este util n aplicaii :
Corolar. Fie n>3, A
1
A
2
A
n
un poligon i M un punct dat. Atunci () a
1
, a
2
, ,
a
n
eR are loc inegalitatea (1).
Aplicaia 1. Dac ABC este un triunghi oarecare i o,|,eR, atunci
(o+|+)
2
R
2
> a
2
| +b
2
o+c
2
o |.
L. Panaitopol
Soluie. n (1) lum a
1
=o, a
2
=|, a
3
= i M punctul O (centru cercului
circumscris). Ca un caz particular al acestei aplicaii se obine
Aplicaia 2. Dac ABC este un triunghi i o, |, eR, atunci
a
2
|+b
2
o+c
2
o|0.
Aplicaia 3. Fie
*
, ,
+
eR z y x . Atunci
( ) ( ) ( ) ( ) ( ) ( )
.
1 1 1
16
1
2 2 2 2 2 2
|
|
.
|

\
|
s
+ +
+
+ +
+
+ +
zx yz xy
y x y z
zx
x z x y
yz
z y z x
xy
D-t. Marinescu, I. erdean (etapa local 1998)

1
Profesor, Liceul teoretic Iancu de Hunedoara, Hunedoara
2
Profesor, Colegiul Naional Aurel Vlaicu, Ortie
18
Soluie. n Aplicaia 1 se va lua o=x, |=y, =z i a=y+z, b=z+x, c=x+y. Din
S
abc
R
4
= , obinem
) ( 4
) )( )( (
z y x xyz
x z z y y x
R
+ +
+ + +
= etc.
Aplicaia 4 . Pentru orice priamid triunghiular MABC avem :
abc cMC bMB aMA > + +
2 2 2
(a=BC, b=AC, c=AB). G.M. 2/1995, p.88
Soluie. Se obine din (1) pentru n=3 i a
1
=a, a
2
=b, a
3
=c.
Aplicaia 5. Fie ABC un triunghi i M un punct al planului su. Atunci
) /(
2 2 2 2 2 2 4 4 4
c b a c b a MC MB MA + + > + + . I. Tomescu, G.M. 6/1972
Soluie. Conform inegalitii Cauchy Buniakovski Schwarz, avem:
( )( ) ( ).
2 2 2 2 2 2 4 4 4
cMC bMB aMA c b a MC MB MA + + > + + + + .
Combinnd aceasta cu inegalitatea din Aplicaia 4, deducem inegalitatea cerut.
Aplicaia 6 . Fie ABC un triunghi, P un punct n planul su, iar PA=x, PB=y,
PC=z. S se arate c ayz+bxz+cxy>abc. C.
Cocea
Soluie. Lum n Corolar n=3, M=P, x a a /
1
= , etc. Obinem:
( ) ( )
.
2 2 2
abc cxy bxz ayz
xyz
abc
xyz
cxy bxz ayz
cz by ax
xyz
abc
xy
abc
xz
c ab
yz
bc a
cz by ax
z
c
y
b
x
a
> + + >
+ +

+ + = + + > + +
|
|
.
|

\
|
+ +
Aplicaia 7. Dac A
1
A
2
A
n
este un poligon i M un punct oarecare, atunci
_ _
s < s =
>
n j i
j i
n
i
i
A A MA n
1
2
1
2
. G.M. 5/1995, p.193
Soluie. Lum n i a
i
, 1 , 1 = = n Corolar.
Aplicaia 8. Dac A
1
A
2
, A
n
este un poligon nscris n cercul C(O,R), atunci
_
s < s
>
n j i
j i
A A R n
1
2 2 2
. M. Chiri, M. Dinc Numere complexe
Soluie. n Corolar se consider n i a
i
, 1 , 1 = = i punctul M n O.
Cel de-al doilea rezultat general este urmtorul:
Propoziia 2. Pentru orice neN, n>3 i orice x
1
, x
2
, , x
n
eR avem:
( )
_ _ _
=
+ +
= =
= > + +
n
k
n k k
n
k
k
n
k
k
x x
n
n n
n
x x x x
1
1 1
2
1
1 1
2
) ( cos
2
cos 1 2 2 2
t t
. (2)
Egalitatea are loc dac i numai dac x
1
= x
2
= = x
n
=1/2.
Demonstraie. Pentru n=3 inegalitatea revine la
_ _ _
=
+
= =
> + + =
3
1
1
3
1
3
1
2
0
4
9
3 2
k
k k
k
k
k
k
x x x x A (cu x
4
= x
1
). Deoarece
_ _ +
>
1
2
k k k
x x x ,
19
avem: = + + >
_ _ _ +
4
9
3 2
1
2
k k k k
x x x x A ( ) 0
2
3
4
9
3
2
2
> |
.
|

\
|
= +
_ _ _ k k k
x x x ,
deci (2) are loc pentru n=3.
Pentru n > 4 inegalitatea (2) revine la
. 0 sin
2
cos 2
2
cos 1 2 2
2
1
2
> + |
.
|

\
|
=
_ _ _ +
n
n
n
x x
n
x x B
k k k k
t t t
Cum ( ) n k x x x x
k k k k
, 1 , 2
2
1
2
1
= + >
+ +
, prin multiplicare cu
n
t 2
cos (ce este >0 pentru
n>4) i apoi sumare dup k, obinem: >
_ +
n
x x
k k
t 2
cos 2
1
n
x
k
t 2
cos 2
2
_
.
Ca urmare, avem: = + |
.
|

\
|
|
.
|

\
|
>
_ _
n
n
n
x
n
x B
k k
t t t
2 2
sin
2
cos 1 2
2
cos 1 2
( ) 0 sin 1 2 sin sin 4 sin 4
2 2 2 2 2 2
> = + =
_ _ _
n
x
n
n
n
x
n
x
k k k
t t t t
q.e.d
Aplicaia 9. Se consdider triunghiul echilateral ABC de latur 1 i punctele
A
1
e(BC), B
1
e(CA), C
1
e(AB). S se arate c
4
3
2
1 1
2
1 1
2
1 1
> + + A C C B B A .
Concurs de matematic 1988 etapa final
Soluie. Notnd x
1
=BA
1,
x
2
=CB
1,
x
3
=AC
1
i , conform teoremei cosinusului ,
inegalitatea revine la
( ) ( )
4
3
3 ) 1 ( ) 1 ( ) 1 ( 2 2
1 3 3 2 2 1 3 2 1
2
3
2
2
2
1
> + + + + + x x x x x x x x x x x x , adic (2)
pentru n=3.
Aplicaia 10. Pe laturile unui ptrat ABCD de latur 1 se consider punctele
Me(AB), Ne(BC), Pe(CD), Qe(DA) astfel nct . 2
2 2 2 2
= + + + QM PQ NP MN
S se arate c MNPQ este ptrat. C. Nsturic, Cardinal 3/1991, p.55
Soluie. Notm AM=x
1
, BN=x
2
, CP=x
3
, DQ= x
4
. Egalitatea se scrie:
( ) ( ) 2 4 2 2
4 3 2 1
2
4
2
3
2
2
2
1
= + + + + + + + x x x x x x x x , adic n (2) pentru n=4 are loc egalitate,
deci x
1
=x
2
=x
3
=x
4
=1/2 i M, N, P, Q, vor fi mijloacele laturilor ptratului ABCD. n
consecin MNPQ este ptrat.
Aplicaia 11. Fie A
1
A
2
, A
n
, n>4, un poligon regulat de latur 1 i P
1
e( A
1
A
2
),
P
2
e( A
2
A
3
), , P
n
e( A
n
A
1
). S se arate c
n
n P P P P P P
n
t 2
cos ...
2
1
2
3 2
2
2 1
> + + + .
R.M.T 1,2 / 1989 , Problema 6539
Soluie. Notnd cu P
1
A
2
=x
1
,P
2
A
3
=x
2
,, P
n
A
1
=x
n
, teorema cosinusului aplicat
triunghiurilor P
1
A
2
P
2
, P
2
A
3
P
3
, ,P
n
A
1
P
1
ne conduce la
n
n
x x x x P P
n
n
x x x x P P
n n n n n
t t ) 2 (
cos ) 1 ( 2 ) 1 ( , ,
) 2 (
cos ) 1 ( 2 ) 1 (
2 2 2
1 1 1
2
1
2
1
2
2 1

+ =

+ = "
Prin sumare obinem: n
n
x x x x P P P P P P
n
k
k k
n
k
k
n
k
k n
+ + = + + +
_ _ _
= = = 1 1 1
2 2
1
2
3 2
2
2 1
2
cos ) 1 ( 2 2 2 ...
t
i, n conformitate cu (2), deducem inegalitatea cerut.
20
Asupra unei clase de iruri recurente
Dan POPESCU
1
Scopul acestui articol este prezentarea unei metode de abordare a unei clase de iruri
recurente de primul ordin. Mai precis, dat irul
1
) (
> n n
x prin
R D x c e
1
i *, ), (
1
N n x f x
n n
e =
+
(1)
unde D D f : , se pune problema existenei limitei
n
n
nx

lim n R .
n ipoteza c } 0 { lim e

R x
n
n
, irul
1
) (
> n n
nx are limita evident; rmne de
investigat cazul n care 0 lim =

n
n
x . Rezultatul principal este cuprins n urmtoarea
Teorem. Fie * R D c avnd originea ca punct de acumulare, funcia D D f : i
irul
1
) (
> n n
x definit prin (1) astfel nct sunt ndeplinite condiiile: 0 lim 1
0
=

n
n
x i
R l
x xf
x f x
x
e =

) (
) (
lim 2
0
0
. Au loc implicaiile:
(a) dac } 0 { eR l , atunci
l
nx
n
n
1
lim =

;
(b) dac l=0 i exist o vecintate V
1
a originii nct D x
x xf
x f x
e >

1
V , 0
) (
) (
,
atunci + =

n
n
nx lim ;
(c) dac l=0 i exist o vecintate V
2
a originii nct D x
x xf
x f x
e <

2
V , 0
) (
) (
,
atunci =

n
n
nx lim .
Demonstraie. Se aplic criteriul Stolz-Cesro irului
1
1
>
|
|
.
|

\
|
n
n
nx
scris sub forma
1
/ 1
>
|
.
|

\
|
n
n
n
x
. n ipotezele impuse avem: .
) (
) (
lim
) (
) (
lim
1 ) 1 (
/ 1 / 1
lim
1
R l
x xf
x f x
x f x
x f x
n
x x
n
n n
n n
n
n n
n
e =

=
+


+

.
Urmeaz c l
nx
n
n
=

1
lim i afirmaiile (a), (b), (c) decurg imediat.
Prezentm mai nti cteva aplicaii directe ale acestei teoreme.
1 (G.M. 11,12/1986, C: 649, M. Bencze). Fie irul
1
) (
> n n
x definit prin 0
1
> x i
* N n ), 1 ln(
1
e + =
+ n n
x x . S se demonstreze c 2 lim =

n
n
nx .
2 (G.M. 3/1987, 21056, F. Dumitrel). Dac irul
1
) (
> n n
x se definete prin
0
1
> x i * N n , 2
1
e =

+
n
x
n n
x x , s se calculeze
n
n
nx

lim .

1
Profesor, Colegiul Naional tefan cel Mare, Suceava
21
3 (G.M. 10/1987, 21253, M. Lascu). Fie irul
1
) (
> n n
x definit prin 0
1
> x i
* N n )), ( 1 ln(
1
e + =
+ n n
x arctg x . S se arate c irul ( )
1 > n
n
nx este convergent la 2.
4 (R.M.T. 2/1987, 6256, V. Bivolaru). Se consider irul
1
) (
> n n
x nct 0
1
> x i
* N n ,
) 1 ln( 2
2
1
e
+
=
+
n n
n
n
x x
x
x . S se arate c irul este convergent i s i se calculeze
limita.
5 (G.M. 4/1995, 23241, V. Nicula). Fie * N p e , ) / 1 , 0 (
1
p x e i
* N n ), 1 )...( 2 1 )( 1 (
1
e =
+ n n n n n
px x x x x . S se calculeze limita
n
n
nx

lim .
6 (G.M. 1/1997, 23668, A. Vernescu). Dac irul
1
) (
> n n
x este definit prin 0
1
> x i
* N n ,
2
1
4
1
1
e + =
+ n n
x x , s se studieze natura irului ( )
1 > n
n
nx i, n caz de
convergen, s se calculeze
n
n
nx

lim .
Condiia
0
1 din teorem se verific uor pentru fiecare dintre irurile precedente. n
privina condiiei
0
2 , pentru funcia aferent ) , 0 ( ) , 0 ( : f indicm:
1.
2
1
) 1 ln(
) 1 ln(
lim
0
=
+
+
x x
x x
x
,
3.
2
1
) 1 ln(
) 1 ln(
lim
0
=
+
+
x arctg x
x arctg x
x
,
5.
2
) 1 (
) 1 )...( 2 1 )( 1 (
) 1 )...( 2 1 )( 1 (
lim
2
0
+
=

p p
px x x x
px x x x x
x
2. 2 ln
2
2
lim
2
0
=

x
x
x
x
x x
,
4.
2
1
) 1 ln( 2
) 1 ln( 2
lim
3
2
0
=
+
+

x x
x
x x
x
x
x
,
6. . 1
2
1
4
1
2
1
4
1
lim
0
=
|
|
.
|

\
|
+ +
+ +

x x
x x
x
Observaie. n privina problemei 5, menionm c pentru p= 1 se obine o problem
din revista Matematika v kole, 5/1984.
Prezentm acum cteva probleme care sunt sub incidena Teoremei, fapt care nu
este ns evident.
7 (G.M. 5,6/1988, 21458, M. Banyai). Fie k>0, x
1
>k i ), /(
2
1
k x x x
n n n
=
+
* N n e . S se calculeze
n
n x
n

lim .
Soluie. Se constat c
1
) (
> n n
x este cresctor i =

n
n
x lim . Aplicm Teorema
irului
1
) (
> n n
y definit prin * N n ,
1
e =
n
n
x
y . Avem ) 1 (
1 n n n
ky y y =
+
, * N n e , i
22
. 0 lim =

n
n
y Funcia ) 1 , 0 ( ) 1 , 0 ( : f dat de ) 1 ( ) ( kx x x f = are .
) (
) (
lim
0
k
x xf
x f x
x
=

. Ca urmare,
k
ny
x
n
n
n
n
1
lim lim = =

.
8 (G.M. 1/1989, 21668, M. Lascu). Fie irul
1
) (
> n n
x definit de 1
1
= x i
* N n ,
1
1
e + =
+
n
n n
x
x x . S se calculeze
n
x
n
n
2
lim

(enun parial).
Soluie. Se constat c =

n
n
x lim . Notm * N n x y
n n
e = , / 2
2
, i deducem c
). 4 4 /( 4
2
1
+ + =
+ n n n n
y y y y Utiliznd Teorema, obinem: . 1
1
lim
2
lim = =

n
n
n
n
ny n
x
9 (G.M. 6,7/1990, 22115, M. P. Mihail). Dac 0 , ,
1
> x b a i
), /(
2
n
n
n
bx a x x + =
* N n e , s se calculeze
n
x x n
1
lim
0
.
Soluie. Se introduce irul
1
) (
> n n
y prin 1 ,
2
> = n x y
n n
. Urmeaz
0 , ) /( ) ( , 1 , ) /(
2 2 2 2
1
> + = > + =
+
x x b a x x f n y b a y y
n n n
, etc. Discuie dup b i
1
x .
10 (G.M. 11/1993, C: 1463, L. Panaitopol). Se consider irul
1
) (
> n n
x definit prin
1
1
> x i * N n , 1
1
e + =
+ n n n
x x x .S se calculeze
2
lim
n
x
n
n
.
Soluie. Cu 1 , / 1 > = n x y
n n
, obinem 1 , 1 /
2
1
> + =
+
n y y y y
n n n n
, i
gsim, n cele din urm,
4
1
lim
2
=

n
x
n
n
.
11 (R.M.T. 2/1997, X127, V.Bivolaru). Fie irul
1
) (
> n n
x definit prin 0
1
> x ,
* ,
1
N n a x x
n
x
n n
e =

+
, unde a>1. Dac * ,
ln
...
2 1
N n
n
x x x
y
n
n
e
+ + +
= , s se
calculeze . lim
n
n
y

Soluie. Cu criteriul Stolz-Cesro, avem:
a
nx
n n
nx
n n
x
y
n
n
n
n
n
n
n
n ln
1
lim
) 1 ln( ln
1
lim
) 1 ln( ln
lim lim = =

=

=

ultima egalitate stabilindu-se cu ajutorul Teoremei (ca n Problema 3).
12 (G.M. 1/1998, C:2005, M. Bencze). Fie irul
1
) (
> n n
x astfel ca 0
1
> x i
* , 1 ) 1 (
2
1
N n x x x
n n n
e + = +
+
. S se demonstreze c
1
) (
> n n
x este convergent i s se
calculeze ). 1 ( lim
n
n
x n

Soluie. Considerm 1 , 1 > = n x y
n n
. Avem 1 ), 2 /( ) (
2
1
> =
+
n y y y y
n n n n
,
i 0 ), 2 /( ) ( ) (
2
< = x x x x x f , etc. Limita este egal cu zero.
23
Un criteriu de concuren a dreptelor
Temistocle BRSAN
1
n revista Recreaii tiinifice, IV (1886), pag. 48, este enunat urmtoarea
Problem. n orice triunghi, dreptele ce unesc picioarele nlimilor corespunztoare
la dou laturi, picioarele bisectorelor alturate cu nlimile i picioarele
perpendicularelor duse din centrul cercului nscris pe cele dou laturi trec prin acelai
punct.
n acelai volum, la pag. 118, este prezentat o soluie sintetic a problemei, dat de
N.Gr. Blnescu, elev la coala de Poduri i osele din Paris.
ncepem prezenta not cu un criteriu de concuren a trei drepte determinate de
punctele lor de intersecie cu dou dintre laturile unui triunghi. Cu ajutorul acestuia vom da
apoi o soluie tehnic problemei de mai sus.
Fie un triunghi ABC i dreptele d
1
, d
2
i d
3
ce intersecteaz n M, P i respectiv R
dreapta AB i n N, Q i respectiv S dreapta AC. Considerm c poziiile acestor puncte sunt
determinate de rapoartele urmtoare:
. , ; , , s
SA
SC
r
RA
RB
q
QA
QC
p
PA
PB
; n
NA
NC
m
MA
MB
= = = = = = (1)
Menionm c aceste puncte au poziii oarecare pe dreptele AB i AC, dar c, n cele ce
urmeaz, vom exclude tacit anumite cazuri triviale, cum ar fi: dou dintre drepte sunt
paralele sau coincid, cele trei drepte sunt concurente ntr-un punct situat pe AB sau AC, una
dintre drepte trece prin vrful A etc.
Propoziia 1. Dreptele d
1
, d
2
i d
3
determinate de numerele m,n,p,q,r,s date de (1) sunt
concurente dac i numai dac aceste numere ndeplinesc condiia
( ) ( ) ( ) 0
1 1 1
0 = = + +
s q n
r p m pn mq ms rn rq ps . (2)
Demonstraie. Din (1) obinem uor relaiile:
AC
n
n
NB AC
n
NA AB
m
m
MB AB
m
MA
1
,
1
1
;
1
,
1
1

= i analoagele. (3)
Fie {X}= d
1
d
2
i { }
3 1
d d X = ' . Utiliznd
teorema lui Menelaus relativ la AMN A i
transversalele PQ i RS i innd seama de (3),
avem: d
1
, d
2
, d
3
concurente X i X ' coincid
= =
SN
SA
RA
RM
QN
QA
PA
PM
N X
M X
XN
XM
'
'
=

NA QA
QA
PA
MB PB

NA SA
SA
RA
MB RB
1
Prof. dr., Catedra de matematic, Univ. Tehnic Gh. Asachi, Iai
24
( ) ( ) ( ) . 0
1
1
1
1
1
1
1
1
1 1
1
1
1
1
1
1
1
1
1 1
= + +

pn mq ms rn rq ps
n s
s
r
m
m
r
r
n q
q
p
m
m
p
p
Soluia Problemei. Fie d
1
, d
2
, d
3
dreptele ce unesc picioarele nlimilor, picioarele
bisectoarelor i, respectiv, punctele de contact ale cercului nscris (puncte pe dreptele AB i
AC). Atunci , ;
cos
cos
,
cos
cos
b
a
p
A c
C a
n
A b
B a
m = = =
a p
c p
s
a p
b p
r
c
a
q

= = , ; i
condiia (2) se verific prin calcul direct.
Observaie. Fie A
0
punctul de concuren a dreptelor din problem i B
0
, C
0
punctele
analoage acestuia. Vom arta ntr-o not urmtoare c AA
0
, BB
0
, CC
0
sunt concurente.
n particular, dac M coincide cu B i S coincide cu C (i.e. m=s=0), vom obine:
Propoziia 2. Fie ABC un triunghi oarecare. Dreapta PQ trece prin punctul de
intersecie a cevienelor BN i CR dac i numai dac numerele p,q,n,r definite ca n (1)
satisfac condia . 0
0
0
1 1 1
1 = = +
q n
r p
n
q
r
p
(4)
Observaie. Acest ultim rezultat este cunoscut ([3, Teorema 3], [2, Teorema 1 o
form diferit a condiiei] etc.). n locurile citate (ct i n alte locuri!) sunt indicate
condiiile n care dreapta PQ trece prin diferite puncte remarcabile ale triunghiului:
)
C sin
A sin
,
B sin
A sin
( O ),
A cos c
C cos a
,
A cos b
B cos a
( H ),
c
a
n ,
b
a
r ( I ), n r ( G
2
2
2
2
1 = = = = ,
),
a p
c p
,
a p
b p
( ),
c
a
,
b
a
( K
2
2
2
2

I ) , (
c p
a p
b p
a p
N

, unde K, , N sunt punctele


lui Lemoine, Gergonne i respectiv Nagel. Se obin rezultatele:
1. 1 = + e
QA
QC
PA
PB
PQ G , 2. tgA
QA
QC
tgC
PA
PB
tgB PQ H = + e ,
3. , a
QA
QC
c
PA
PB
b PQ I = + e 4. A sin
QA
QC
C sin
PA
PB
B sin PQ O 2 2 2 = + e
5.
2 2 2
a
QA
QC
c
PA
PB
b PQ K = + e , 6.
a p
QA
QC
c p
PA
PB
b p
PQ

e
1 1 1
I
7. ) ( ) ( ) ( a p
QA
QC
c p
PA
PB
b p PQ N = + e etc.
Foarte adesea aceste condiii sunt date pentru cazul restrictiv cnd Pe(AB) i Qe(AC), fapt
care justific renunarea la lucrul cu segmente orientate.
Bibliografie
1. Colecia revistei Recreaii tiinifice (1883-1886).
2. C. Chiser Condiii necesare i suficiente ca o dreapt s treac prin puncte importante
dintr-un triunghi, G.M. 9/2000.
3. N. Oprea Un punct i o dreapt remarcabil din planul unui triunghi, G.M. 11/1996.
25
Cteva consecine ale unei relaii a lui Gergonne
Ioan SCLEANU
1
n triunghiul ABC considerm cevienele A A ' , B B ' , C C ' concurente n M.
Notm
A M
AM
x
'
= ,
B M
BM
y
'
= ,
C M
CM
z
'
= ; constatm c dac M este centrul de greutate
al triunghiului, atunci z y x , , sunt numere naturale. Ne propunem s determinm toate
punctele MeInt ABC cu proprietatea urmtoare:
rapoartele z y x , , corespunztoare lui M sunt numere naturale.
(*)
n particular, vom aeza ntr-un cadru firesc i vom extinde rezultatul obinut n [1].
Pornim de la urmtoarea
Teorem(Gergonne). n triunghiul ABC, cevienele ' , ' , ' CC BB AA sunt
concurente n M. Atunci 1 =
'
'
+
'
'
+
'
'
C C
C M
B B
B M
A A
A M
(vezi [2]).
Cu notaiile iniiale, putem rescrie concluzia 1
1
1
1
1
1
1
=
+
+
+
+
+ z y x
; n mod
necesar, unul dintre termenii sumei din stnga trebuie s fie cel puin
3
1
. Vom presupune
c
3
1
1
1
>
+ x
, de unde 2 s x . Cum e x N*, rezult c 1 = x sau . 2 = x
Dac 1 = x , atunci
2
1
1
1
1
1
=
+
+
+ z y
, i.e. 4 ) 1 )( 1 ( = z y i deoarece
e z y, N*, rezult c )} 2 , 5 ( ), 3 , 3 ( ), 5 , 2 {( ) , ( e z y .
Dac 2 = x , atunci
3
2
1
1
1
1
=
+
+
+ z y
. Unul dintre termenii din stnga trebuie s
fie cel puin
3
1
; s presupunem c
3
1
1
1
>
+ y
, adic 2 s y . Dac 1 = y , obinem 5 = z ,
iar dac 2 = y , rezult 2 = z .
n concluzie, ) , , ( z y x poate lua valorile (2,2,2), (1,3,3), (1,2,5), precum i
toate permutrile posibile ale acestor triplete. Avem astfel 3 tipuri de puncte M cu
proprietatea (*), pe care le vom nota corespunztor M[2,2,2], M[1,3,3], M[1,2,5]. Ne
propunem n continuare s caracterizm geometric fiecare dintre aceste tipuri de puncte.
Propoziia 1. Un punct M este de tipul M[2,2,2] dac i numai dac este centrul
de greutate al triunghiului ABC.
1
Profesor, Liceul Teoretic tefan cel Mare, Hrlu
26
Demonstraie. Suficiena este evident. Pentru a demonstra necesitatea, aplicm
teorema lui Menelaus n triunghiul M B A ' cu transversala C A B ' ; obinem
1 =
'
'

'

' M A
A A
CA
C B
B B
BM
. Din ipotez, 3
'
'
=
M A
AA
,
3
2
'
=
BB
BM
i atunci
2
1 '
=
CA
C B
, adic B '
este mijlocul lui [AC]. Rezult c M se afl situat pe mediana [ ' BB ] astfel nct
2
1
'
'
=
BB
MB
, deci M este centrul de greutate al triunghiului ABC.
Propoziia 2. Un punct M este de tipul M[1,3,3] dac i numai dac este
mijlocul medianei [ ' AA ] (analog pentru M[3,1,3] i M[3,3,1]).
Demonstraie. Dac M[1,3,3], atunci
3
1 ' '
= =
MB
M B
MC
M C
, deci. BC C B // ' '
conform reciprocei teoremei lui Thales aplicat n triunghiul MBC. Rezult de aici c
C B
AB
B C
AC
'
'
'
'
= i folosind teorema lui Ceva obinem c 1
'
'
=
C A
BA
, deci [ A A ' ] este median,
iar M este mijlocul su.
Reciproc, dac M este mijlocul medianei | | A A ' , evident c 1 = x . Din teorema lui
Ceva obinem c
C B
AB
B C
AC
'
'
'
'
= , deci BC C B // ' ' , de unde CMB MB C A A ~ ' ' i
ABC C AB A A ~ ' ' . Obinem de aici c
AC
AB
BC
C B
z y
' ' ' 1 1
= = = . Pe de alt parte, aplicnd
teorema lui Menelaus n triunghiul C A A ' cu transversala ' BMB , rezult c
1
' '
' '
=
MA
MA
A B
CB
BC
BA
i cum
2
1 '
=
BC
BA
, 1
'
=
MA
MA
, avem c 2
'
'
=
A B
CB
, i.e.
3
1 '
=
AC
AB
. n
concluzie, 3 = = z y , deci M este de tipul M[1,3,3].
Propoziia 3. Un punct M este de tipul M[1,2,5] dac i numai dac aparine
liniei mijlocii ] [
1 1
B C i o mparte n dou segmente avnd raportul 2 (au loc i afirmaiile
analoage pentru M[1,5,2] etc.).
Demonstraie.
Fie M[1,2,5]; cum 1 = x , rezult c M aparine
liniei mijlocii ] [
1 1
B C . Ducem AC MT ,
TeBC; atunci CT MB
1
este paralelogram
i deci | | | | TC MB
1
. Aplicnd teorema lui
Thales n triunghiul ' BCB obinem 2
'
= =
MB
BM
TC
BT
,
deci
1 1
3
2
3
1
C B BC CT = = , adic
1 1 1
3
2
C B MB = .
C
1
B
1
B '
M
T A '
C B
A
27
Reciproc, fie | |
1 1
B C M e astfel nct 2
1
1
=
M C
MB
. Cum M aparine liniei mijlocii,
evident c 1 = x . Cu notaiile precedente, BC BC B C MB CT
3
1
2
1
3
2
3
2
1 1 1
= = = = , deci
2 =
TC
BT
. Din teorema lui Thales aplicat n ' BCB A rezult c 2
'
= =
TC
BT
MB
BM
, adic
. 2 = y Folosind acum relaia lui Gergonne, se deduce c 5 = z .
Aplicaia 1. Centrul cercului circumscris triunghiului ABC are proprietatea (*)
dac i numai dac triunghiul este echilateral.
Demonstraie. Presupunem c O are proprietatea (*). Dac, prin absurd,
Oe
1 1
C B , atunci AB O C
1
i O B
1
AC , deci A, B, C ar fi coliniare. Rezult c
Oe
1 1
C B i de aici urmeaz c O este de tipul O [2,2,2], adic OG, deci triunghiul
ABC este echilateral. Reciproca este imediat.
Aplicaia 2. Centrul cercului nscris are proprietatea (*) dac i numai dac
triunghiul ABC este echilateral.
Demonstraie. Presupunem c I are proprietatea (*). Dac, prin, absurd,
Ie
1 1
C B , atunci < IB C
1
< IBC i deci < IB C
1
< BI C
1
, adic triunghiul BI C
1
este
isoscel cu [
1
BC ] [ I C
1
]. Analog, [
1
CB ] [ I B
1
], de unde + = = I C C B BC
1 1 1
2 2
AC AB CB BC I B + = + = +
1 1 1
2 2 2 , imposibil. Rezult c I este punct de tipul I[2,2,2],
adic I G, deci triunghiul ABC este echilateral. Reciproca este evident.
Observaia 1. Acest rezultat este demonstrat pe o alt cale n [1].
Observaia 2. Exist triunghiuri neechilaterale al cror ortocentru are proprietatea
(*). Caracterizarea acestora este dat mai jos (Aplicaia 4) i o propunem spre rezolvare,
mpreun cu dou probleme ajuttoare:
Aplicaia 3. Fie triunghiul ABC, M si N mijloacele laturilor [AB], respectiv [AC]
iar H intersecia inlimilor ' AA i ' BB . Au loc echivalenele:
1) ] [
2 2 5
3
MN H
AC AB BC
e = = i HN=2HM 1
'
=
HA
HA
i 2
'
=
HB
HB
;
2) = =
3 3
2
AC AB BC
H este mijlocul lui [MN] 1
'
=
HA
HA
i 3
'
=
HB
HA
.
Aplicaia 4. Ortocentrul triunghiului ABC are proprietatea (*) dac i numai dac
laturile triunghiului sunt direct proporionale fie cu 1,1,1, fie cu 3, 2 2 , 5 , fie cu 2,
3 , 3 .
Bibliografie
1. Iulica Georgescu Asupra unei clase de triunghiuri, G.M. 2-3/1982.
2. Liviu Nicolescu, Vladimir Boskoff Probleme practice de geometrie, Ed. Tehnic,
Bucureti, 1990.
28
Unele iruri monotone cu limita e sau e
-1
Gheorghe COSTOVICI
1
n Propoziiile care urmeaz, vom demonstra prin metode elementare monotonia
unor iruri cu limita e sau e
-1
.
Propoziia 1. irul
n
n
x n
x e )
1
1 ( ) (
+
+ = este cresctor, 0 > x fixat.
Demonstraie. Din ) .... )( (
1 1 1 1 n n n n n n
b ab b a a b a b a + + + + =
+ +
, n ipoteza
0 > > b a , se obine c
1
)] 1 )( ( [
+
< +
n n
b n b a a a . Lund n aceast inegalitate
x n
a
+
+ =
1
1 i
x n
b
+ +
+ =
1
1
1 , gsim c 1 )[ (x e
n
) ( ]
) 1 )( (
1
x e
x n x n
x
n+
<
+ + +
+ ,
care atrage ) ( ) (
1
x e x e
n n +
< , 1 > n , 0 > x fixat.
Propoziia 2. irul
1
)
1
1 ( ) (
+
+
+ =
n
n
x n
x a este descresctor, ] 0 , 1 ( e x fixat.
Demonstraie. Considerm funcia ) , 1 [ : f R ,
1
)
1
1 ( ) (
+
+
+ =
t
x t
t f , cu
] 0 , 1 ( e x arbitrar fixat. Avem ]
) )( 1 (
1
)
1
1 )[ln( ( ) (
x t x t
t
x t
t f t f
+ + +
+

+
+ = ' , 1 > t .
Folosind ( ) , 0 , 1 ln > < + y y y rezult
+
<
+ + +
+

+
+
x t x t x t
t
x t
1
) )( 1 (
1
)
1
1 ln(
0
) )( 1 ( ) )( 1 (
1
s
+ + +
=
+ + +
+

x t x t
x
x t x t
t
, adic ( ) 1 , 0 ' > < t t f i deci f este
descresctoare. Atunci ), 1 ( ) ( + > n f n f i.e. ) ( ) (
1
x a x a
n n +
> , 1 > n , ceea ce ncheie
demonstraia.
Propoziia 3. irul
n
n
x n
x b )
1
1 ( ) (
+
= este descresctor, 1 > x fixat.
Demonstraie. Observm c
) 1 (
1
)
1
1
1 (
1
)
1
1 ( ) (

=
+
+
=
+
=
x e
x n
x n
x b
n
n
n
n
,
1 > n . irul ) 1 ( x e
n
este cresctor pentru 1 0 1 > > x x i are termeni pozitivi,
deci ) (x b
n
este descresctor pentru orice 1 > x fixat.
Propoziia 4. irul
1
)
1
1 ( ) (
+
+
=
n
n
x n
x c este cresctor, ] 1 , 0 ( e x .
Demonstraie. Observm c

1
Conf. dr., Catedra de matematic, Univ. Tehnic Gh. Asachi, Iai
29
1 ,
) 1 (
1
)
1
1
1 (
1
)
1
1 ( ) (
1
1
>

=
+
+
=
+
=
+
+
n
x a
x n
x n
x c
n
n
n
n
.
Conform cu Propoziia 2, irul ) 1 ( x a
n
este descresctor pentru e ] 0 , 1 ( 1 x
] 1 , 0 ( e x i are termenii pozitivi, deci ) (x c
n
este cresctor pentru ] 1 , 0 ( e x .
Propoziia 5. irul ( )
p n
n
n
p E
+
+ = )
1
1 ( este descresctor, e p N*.
Demonstraie. Din identitatea
, 0 ), ... )( (
1 2 2 1
> > + + + + =
+ + + + + +
b a b ab b a a b a b a
p n p n p n p n p n p n
rezult
1
) )( (
+ + +
+ >
p n p n p n
b p n b a b a . Punnd aici
n
a
1
1+ = i
1
1
1
+
+ =
n
b ,
se obine )
) 2 (
1 ( )
1
1
1 ( )
1
1 (
+
+
+
+
+ > +
+ +
n n
p n
n n
p n p n
pentru 1 > n i p fixat. innd
seama c
1
1
1
) 2 (
1
+
+ >
+
+
+
n n n
p n
, va rezulta
+
+ > +
+ + + p n p n
n n
1
)
1
1
1 ( )
1
1 (
) ( ) (
1
p E p E
n n +
> , ceea ce ncheie demonstraia.
Propoziia 6. irul
x n
n
n
x E
+
+ = )
1
1 ( ) ( este descresctor, 1 > x fixat.
Demonstraie. Se consider funcia ) , 1 [ : f R,
x t
t
t f
+
+ = )
1
1 ( ) ( , unde 1 > x
fixat. Atunci ]
) 1 (
)
1
1 )[ln( ( ) (
+
+
+ = '
t t
x t
t
t f t f i folosind inegalitatea y y < + ) 1 ln(
pentru 0 > y , avem
) 1 (
1
) 1 (
1
) 1 (
)
1
1 ln(
+

=
+
+
<
+
+
+
t t
x
t t
x t
t t t
x t
t
, de unde ( ) 0 ' < t f i
deci f este descresctoare pentru 1 > t . Avem + > ) 1 ( ) ( n f n f ) ( ) (
1
x E x E
n n +
>
pentru 1 > n , ceea ce ncheie demonstraia.
Observaie. Propoziia 5 este un caz particular al Propoziiei 6, avnd ns o
demonstraie elementar.
Propoziia 7. irul
x n
n
n
x u
+
= )
1
1 ( ) ( este cresctor, 0 > x fixat.
Demonstraie. Putem scrie
) 1 (
1
)
1
1 (
1
)
1
1
1 ( ) (
1
1
1
+
=
+
=
+
=
+ +
+ +
+
x E
n
n
x u
n
x n
x n
n
.
Conform Propoziiei 6, irul ) 1 ( + x E
n
este descresctor pentru 0 1 1 > > + x x i
are termenii pozitivi. Rezult ca irul ) (
1
x u
n+
este cresctor, adic
... ) ( ... ) ( ) (
3 2
< < < < x u x u x u
n
. Dar ) (
1
x u =0, deci avem i ) ( ) (
2 1
x u x u < , ceea ce
ncheie demonstraia.
30
Propoziia 8. irul
x n
n
n
x v
+
+
= )
1
1
1 ( ) ( este descresctor, 0 s x fixat.
Demonstraie. Se consider funcia ) , 0 ( : g R,
x t
t
t g
+
+
= )
1
1
1 ( ) ( , unde
0 s x fixat. Avem ]
) 1 (
)
1
1
1 )[ln( ( ) (
+
+
+
+
= '
t t
x t
t
t g t g i folosind inegalitatea
, 1 0 , ) 1 ln( < < < y y y gsim 0
) 1 ( ) 1 ( 1
1
) 1 (
)
1
1
1 ln( s
+
=
+
+
+
+
<
+
+
+
+

t t
x
t t
x t
t t t
x t
t
pentru ). , 0 ( e t Deci 0 ) ( < ' t g , ) , 0 ( e t , adic g este descresctoare pe ) , 0 ( .
Vom avea ) ( ) ( ) 1 ( ) (
1
x v x v n g n g
n n +
> + > , 1 > n , ceea ce ncheie demonstraia.
Propoziia 9. irul
x n
n
n
x v
+
+
= )
1
1
1 ( ) ( este cresctor, 1 > x fixat.
Demonstraie. Avem
) (
1
)
1
1 (
1
)
1
1
1 ( ) (
x E
n
n
x v
n
x n
x n
n
=
+
=
+
=
+
+
, 1 > n .
Conform Propoziiei 6, irul ) (x E
n
este descresctor pentru 1 > x i are termenii
pozitivi, deci ( ) x v
n
este cresctor, 1 > x fixat.
Observaii. n demonstraia Propoziiei 1, ne-am inspirat din [1;p. 24-25].
Relativ la irul ) (x E
n
, n [2; p. 223-224] se arat pe cale neelementar c este
descresctor pentru
2
1
> x i cresctor pentru
2
1
< x . In [3; p. 13-14] se arat pe cale
elementar ca irul )
2
1
(
n
E este descresctor. irurile studiate n Nota de fa au
aplicaie n studiul unor serii de funcii.
Bibliografie
1. Fr. Junker Hhere Analysis I, 1920.
2. G. Klambauer Problems and Propositions in Analysis, New York, 1979.
3. A. Vernescu iruri de numere reale, Bucureti, 2000.
31
Studiu comparativ privind cteva medii uzuale
Claudiu-tefan POPA
1
Vom folosi n cele ce urmeaz urmtoarele convenii de notare: dac , 0 , > y x atunci
,
2
y x
xy
m
h
+
= , xy m
g
= ,
2
y x
m
a
+
=
2
2 2
y x
m
p
+
= sunt mediile armonic,
geometric, aritmetic, respectiv ptratic ale numerelor x i y. De asemenea, introducem
y x
y x
m
+
+
=
2 2
media ponderat a numerelor x i y cu ponderile reale x, respectiv y. n
acest context,
h
m poate fi privit ca media ponderat a numerelor x i y cu ponderile y,
respectiv x. Au loc urmtoarele inegaliti ntre medii:
m m m m m
p a g h
s s s s (1)
cu egalitate pentru . y x =
Se observ uor prin calcul c ,
h a a
m m m m = (2)
deci media aritmetic a numerelor x i y este medie aritmetic i pentru numerele
h
m i m.
De asemenea, aa cum
g
m este i medie geometric ntre
h
m i ,
a
m la fel
p
m este medie
geometric ntre
a
m i m, deoarece .
2 2
2 2 2 2
y x
y x y x y x
+
+

+
=
+
(3)
Vom presupune n continuare y x < i, ca n figura de mai jos, notm ,
g a
m m A =
,
a p
m m B = ,
h g
m m C =
p
m m D = :
Propoziie. Cu notaiile de mai sus, avem: A D B C
A
< < < <
2
. (4)
Demonstraie. Pentru prima inegalitate, avem echivalent:
g h a h g g a
m m m m m m m C
A
3 2 2 2
2
< + < < .
ns . 3 3 2 2
4
2
2
4
2
2
g h a
m ab ab
b a
ab b a
b a
ab b a
m m = < =
+

+
s
+
+
+
= + La fel,
B C <
+
<
+

+
< xy
y x
y x
xy y x
m m m m
a p h g
2
2
2
2 2
( )
( )

+
<
|
|
.
|

\
|
+
+
+

xy
y x
xy
y x
y x
y x
2 2 2
2 2 2 2 2
( )
0
1
2
1
2
2 2 2
>
(
(

|
|
.
|

\
|
+
+

y x
xy
y x y x
1
Profesor, coala Alecu Russo, Iai
32
B A xy
y x
y x
y x
xy
y x
y x
> +
+
> +
+

|
|
.
|

\
|
+
+
>
=
2
1
2
1
2 2 2 2
.
Pentru a arta c , B A > s observm mai nti c , 2
2 2 2
a g p
m m m = + relaie ce rezult n
urma unui calcul de rutin; deci . 2 / ) (
2 2
g p a
m m m + = Pe de alt parte, aplicnd (1)
numerelor
p
m i ,
g
m avem c 2 / ) ( 2 / ) (
2 2
g p g p
m m m m + > + , cu egalitate pentru
. y x m m
g p
= = Cum , y x = rezult c 2 / ) (
g p a
m m m + > , adic . B A >
n continuare s aplicm inegalitatea xy y x xy s (echivalent cu
a g
m m s )
numerelor
a
m i . m . innd seama de (3) rezult c , D B m m m m
p a p
s s
egalitatea fiind atins pentru cazul exceptat . y x =
n fine, inegalitatea A D < rezult din faptul c D B C A + = + (relaie echivalent cu
(2)) i . B C <
Observaia 1. Rezultatul demonstrat generalizeaz unele probleme aprute n Gazeta
Matematic. Astfel problemele
E: 11 997. Dac , 0 , > y x demonstrai c .
4 y x
xy y x
xy
+
+
+
s
Gh. Neculeu i Ion Neculeu
E: 12 162. Dac , 0 , > y x s se arate c .
2
2 2
xy
y x
y x +
+
> +
Claudiu-tefan Popa
sunt simple transcripii ale inegalitilor , C A > respectiv . B A >
Observaia 2. Se pot obine inegaliti mai interesante, n care s apar mai multe
dintre medii. Prezentm un exemplu: adunnd membru cu membru inegalitile , B A >
C A > i A A > obinem ,
2
2 3
1
2
3
2 2
|
|
.
|

\
|
+

+
>
+
+ + >
y x
xy y x
xy
y x
C B A A
inegalitate a crei demonstraie direct este destul de laborioas.
Observaia 3. Relaiile (4) arat c oricum am alege trei dintre numerele A, B, C, D,
ele pot constitui laturile unui triunghi (n cazul y x = ).
Observaia 4. Continund ideea, aezm pe o ax numerele A D B C < < < i
comparnd lungimile intervalelor care apar, obinem noi inegaliti n care apar mediile. n
acest sens, problema:
E: 12 177. Dac , 0 , > y x demonstrai c
2 4
y x
xy
y x
xy y x
s
+
+
+
.
Manuela Prajea
este banal, ntruct se reduce la , y x C A s inegalitate grosier.
33
NOTA ELEVULUI
Aplicaii ale monotoniei mediilor n raport cu ordinul lor
Codrin ANDREI i tefan RUSU
1
Fie date numerele x
1
, x
2
, ..., x
n
R
*
+
(n N
*
) i R. Se numete media de
ordin a numerelor x
1
, x
2
, ..., x
n
numrul M

(x
1
,...,x
n
) (pe scurt, M

) definit prin

= + + +
=
|
|
.
|

\
|
+ + +
=
0 ,
0 ,
2 1
/ 1
2 1
o
o
o
o o o
o
n
n
n
x x x
n
x x x
M
"
"
(1)
Observm c M
0
=
n
n
x x x ...
2 1
= G
n
este media geometric, M
1
=
n
n
x x x + + + ...
2 1
=A
n
-
media aritmetic i c M
-1
=
n
x x x
n
1
...
1 1
2 1
+ + +
=H
n
- media armonic a numerelor date.
Mai general, se numete media ponderat de ordin a numerelor x
1
, x
2
, ..., x
n
cu
ponderile (pozitive) p
1
, p
2
, ..., p
n
numrul M

(x, p) (pe scurt, M

) definit prin
( ) ( ) ( ) | |

=
+ + +
=
|
|
.
|

\
|
+ + +
+ + +
=
0 ,
...
...
0
...
...
) , (
)
2 1
/( 1
2
2
1
1
,
/ 1
2 1
2 2 1 1
o
o
o
o
o o o
n
n
n
n
n n
p p p
p
x
p
x
p
x
p p p
x p x p x p
p x M
(2)
( pentru p
1
=p
2
==p
n
=1, (2) devine (1) ).
Vom prezenta mai nti un rezultat binecunoscut, anume :
Propoziie. Mediile (ponderate sau nu) sunt monoton cresctoare n raport cu
ordinul lor, adic , R, < M
s
M

.
Demonstraie. S presupunem mai nti c avem 0<<. Ca urmare, exist t >1
astfel nct = t . Utilizm inegalitatea lui Jensen pentru funcia convex f (x)=x
t
,
x
( )
, 0 , i scriem :
n
p p p
t
n
x
n
p
t
x p
t
x p
t
n
p p p
n
x
n
p x p x p
+ + +
+ + +
s
+ + +
+ + +
|
|
.
|

\
|
...
2 1
...
2 2 1 1
...
2 1
...
2 2 1 1
. (3)
nlocuim n aceasta x
1
, x
2
, ..., x
n
respectiv cu
o o o
n
x x x ,..., ,
2 1
; obinem :
1
Elevi, Liceul Teoretic Gr. Moisil, Iai
34
n
p p p
t
n
x
n
p
t
x p
t
x p
t
n
p p p
n
x
n
p x p x p
+ + +
+ + +
s
+ + +
+ + +
|
|
.
|

\
|
...
2 1
...
2 2 1 1
...
2 1
...
2 2 1 1
o o o o o o
i , prin ridicare la puterea pozitiv
o t
1
deducem c M

s M
t
, adic M

s M
.
Dac <<0, atunci exist t >1 astfel nct =t . Se procedeaz n mod
similar: n (3) se nlocuiesc x
1
, x
2
, ..., x
n
cu
| | |
n
x x x ,..., ,
2 1
, se ridic la puterea negativ
| t
1
ambii membri ai inegalitii rezultate i se obine M
>
M
t
, adic M
s
M

.
n sfrit, dac n inegalitatea M

< M

( 0< < ) facem ca s tind la zero,


vom obine M
0
< M

( >0). Tot ca un caz limit se obine i M

< M
0
( <0). Q.e.d.
Indicm cteva aplicaii ale propoziiei de mai sus.
1. S se rezolve ecuaia . 0 1 , 2
1 1
> > = |
.
|

\
|
+ |
.
|

\
|
+ y i x x
x
x
x
x
y
y y
Soluie. Avem trei cazuri :
I. y=1. Ecuaia devine x
x
x
x
x 2
1 1
= + +
|
.
|

\
|
|
.
|

\
|
i aceasta este verificat de orice 1 > x .
Aadar perechile (x,1), 1 > x , sunt soluii ale ecuaiei date.
II. 0<y<1. Utiliznd inegalitatea M
y
s M
1
pentru numerele distincte
y
x
x
|
.
|

\
|
+
1
i
y
x
x
|
.
|

\
|

1
obinem :
y
y
y y
x
y
y
x
x
x
x
x
x 2 2
1 1
2
1 1
=
|
|
.
|

\
|
(

|
|
.
|

\
|
+ + |
.
|

\
|
+ < |
.
|

\
|
+ |
.
|

\
|
+ , deci ecuaia dat
nu are soluii n acest caz.
III. y>1. Utilizm de aceast dat inegalitatea M
1
s M
y
pentru aceleai numere distincte i
obinem c membrul nti al ecuaiei este strict mai mare ca 2x . Deci nu avem soluii.
Rezumnd, ecuaia dat are ca soluii perechile (x,1), 1 > x .
2. Fie a, b, c >0 cu abc=1. Demonstrai inegalitatea 2(a+b+c) s a
2
b+ab
2
+a
2
c+
+ac
2
+b
2
c+bc
2
(V. Popa, G : 615, R. M. Galai, nr. 17-18).
Soluie. Prin calcul direct constatm c a
2
b + ab
2
+a
2
c +ac
2
+ b
2
c+ bc
2
-2 (a+b+c)
=(a+b+c)(ab+bc+ca-2)-3abc i, deci,avem de artat c (a+b+c)(ab+bc+ca-2) > 3 (1) (am
utilizat condiia abc=1). Inegalitatea A> G pentru numerele a, b, c revine la a+b+c > 3 (2)
iar G> H conduce la ) /( 3 1 ca bc ab + + > sau ab+bc+ca-2 > 1 (3) (s-a inut seama de relaia
abc=1). Din (2) i (3) , prin nmulire, se obine (1).
35
3. S se arate c , 1
2
cos
2
sin
2
1
1
s + s

o o
n n
n
o e R i n e N*.
Soluie. Partea dreapt se dovedete astfel : 1 )
2
cos
2
(sin
2
cos
2
sin = + s +
n n n
o o o o .
Pentru a dovedi inegalitatea din partea stng, utilizm M
1
s M
n
pentru numerele o
2
sin i
o
2
cos ; obinem :
1
2 / 1 ] 2 / )
2
cos
2
[(sin 2
2
cos
2
sin

= + > +
n n n n
o o o o , q.e.d.
4. Fie ABC un triunghi dreptunghic n A i numrul R. Au loc inegalitile :
(i) , 2
2 / 1 o o o o
a c b

s + dac 2 s o (ii) , 2
2 / 1 o o o o
a c b

> + dac 2 > o .
Soluie. Avem s s ) , ( ) , ( 2
2
c b M c b M
o
o
| | | |
o o o o o o o
a a c b c b c b
2 / 1 2 / 1 2 2 / 1
2 2 / 2 / ) ( 2 / ) (

s + = + s + .
Similar se procedeaz pentru a dovedi (ii).
5. Fie ABC un triunghi echilateral nscris n cercul C(O,R) i MeC(O,R)
oarecare. Artai c R e | o , , astfel nct 4 4 2 , 2 > s s s | o i au loc
inegalitile:
(i)
o o o o o
R MC MB MA ii R MC MB MA > + + s + +
4 / 2 /
6 3 ) ( ; 2 3
(iii) .
4 / 2 /
6 3 2 3
| | | | | | |
R MC MB MA R + + s s
Soluie. Sunt cunoscute relaiile urmtoare : MA
2
+MB
2
+MC
2
=6R
2
i
MA
4
+MB
4
+MC
4
=18R
4
([1], pp. 27-28). Ca urmare, M
2
(MA, MB, MC)=R 2 i
M
4
(MA,MB,MC)=R
4
6 . Pe de alt parte, avem : M

s M
2
sM

sM
4
sM
.
Din M

s M
2
urmeaz c
o o o
MC MB MA + + =3(M

s 3(M
2
)

=
o o
R
2 /
2 3 , adic (i). Analog se
obin (ii) i (iii).
6. Fie ABCD un ptrat nscris nscris n cercul C(O, r). Fie MeC(O,r) i
R e | o , , astfel nct 4 4 2 , 2 > s s s | o i . Artai c au loc inegalitile :
(i)
o o o o o o
r MD MC MB MA ii r MD MC MB MA > + + + s + + +
4 / 2 /
6 4 ) ( ; ) 2 / 3 ( 4
(iii) .
4 / 2 /
6 4 ) 2 / 3 ( 4
| | | | | | | |
r MD MC MB MA r + + + s s
Soluie. Se utilizeaz egalitile:
2 2 2 2 2
6r MD MC MB MA = + + + i
4 4 4 4 4
24r MD MC MB MA = + + + [4] i , procednd ca n problema precedent, se obin
inegalitile cerute.
Bibliografie
1.C. Cocea - 200 de probleme din geometria triunghiului echilateral, Ed. Gh. Asachi,
Iai, 1992 .
2.L. Pran, C.-G. Lazanu -Probleme de algebr i trigonometrie, Ed. Facla,
Timioara,1983.
3.V. tefnescu, N. Deval - Asupra unor inegaliti ntre medii generalizate i aplicaiile lor,
G.M. 1/1985, 8-11.
4.G. ieica Culegere de probleme de geometrie, Ed. Tehnic, Bucureti, 1956.
36
CHESTIUNI METODICE
Funcii care admit / nu admit primitive
Dumitru GLEAT
1
i Gabriel POPA
2
Ne propunem n ceea ce urmeaz o abordare ct mai intuitiv a unor probleme
considerate n general ca fiind dificile, grupate n urmtoarele clase:
I. Dat o funcie, s se stabileasc faptul c admite primitive;
II. Dat o funcie, s se arate c nu admite primitive;
III. Dat o funcie a crei expresie depinde de anumii parametri, s se determine
valorile acestora astfel nct funcia s admit primitive.
n redactarea acestui articol am pornit de la constatarea c soluiile date n diverse
cri unora dintre problemele din [2], pag. 13-14, sunt artificiale i greu de urmrit de ctre
elevi. Propunem n cele ce urmeaz o prezentare pe care o credem logic i adecvat
predrii la clas.
I. (i) Orice funcie continu admite primitive.
Problema 1. Funcia f : R R , f(x)=

=
=

0 , 0
0 ,
1 2
1
5
x
x e
x
x
admite primitive.
Soluie. Substituia t
x
=
2
1
conduce la
2
5
5
1
t
x
= dac 0 > x i
2
5
5
1
t
x
= dac
0 > x , deci 0 lim ) ( lim
2
5
0
= =
!
t
t x
e
t
x f ;
t
t x
e
t
x f
2
5
0
lim ) ( lim

=
|
= 0, de unde ) ( lim
0
x f
x
= 0 =
f(0), adic f este continu n 0. Cum f este evident continu pe (- , 0) i pe (0 , ),
rezult c f continu pe R , deci f admite primitive.
Problema 2. Funciile f : R R , f(x)=

=
=
0 , 0
0 ,
1
sin
x
x
x
x
i g : R R ,
g(x)=

=
=
0 , 0
0 ,
1
cos
x
x
x
x
admit primitive.
Soluie. Pentru f, avem 0 0
1
sin
0
s
x
x
x
x , deci
) ( lim
0
x f
x
= 0 = f(0),
adic f este continu . Analog se procedeaz pentru g .
(ii) Anumitor funcii li se pot construi efectiv primitivele.

1
Profesor, Grup colar tefan Procopiu, Iai
2
Profesor, Liceul Teoretic Garabet Ibrileanu, Iai
37
Problema 3. Funciile f : R R , f(x)=

=
=
0 , 0
0 ,
1
cos
1
sin 2
x
x
x x
x
i g : R R ,
g(x) =

=
= +
0 , 0
0 ,
1
sin
1
cos 2
x
x
x x
x
admit primitive.
Soluie. Considerm funcia

=
=
=
0 , 0
0 ,
1
sin
) (
2
x
x
x
x
x F , care este derivabil pe R
-
i F'(x) = f(x), e x R
-
. S mai artm c F este derivabil n origine, iar F'(0) = 0:
0
1
sin lim
1
sin
lim
0
) 0 ( ) (
lim ) 0 ( '
0
2
0 0
= = =

=
x
x
x
x
x
x
F x F
F
x x x
(vezi Problema 2), deci F este primitiv a lui f . Analog se procedeaz pentru g .
Generalizare 1. Funciile f
n
: R R , f
n
(x)=

=
=

0 , 0
0 ,
1
cos
1
sin
2 1
x
x
x
x
x
nx
n n
i
g
n
: R R , g
n
(x)=

=
= +

0 , 0
0 ,
1
sin
1
cos
2 1
x
x
x
x
x
nx
n n
admit primitive, n> 2.
Generalizare 2. Funciile f
n
: R R ,
f
n
(x)=

=
= +
0 , 0
0 ,
1
cos
1
sin ) 1 (
x
x
x
n
x
x n
n n
n
i g
n
: R R , g
n
(x)=

=
= + +
0 , 0
0 ,
1
sin
1
cos ) 1 (
x
x
x
n
x
x n
n n
n
admit primitive, n > 1.
(iii) O combinaie liniar a unor funcii ce admit primitive, admite primitive.
Problema 4. Funciile f : R R , f(x)=

=
=
0 , 0
0 ,
1
sin
x
x
x
i g : R R , g(x)=
=

=
=
0 , 0
0 ,
1
cos
x
x
x
admit primitive.
Soluie. Funcia f se poate scrie ca o combinaie liniar cu coeficienii 2, 1 a
funciilor g din Problemele 2 i 3, deci admite primitive. Analog pentru g .
Observaia 1. Dac F este o primitiv a funciei f, atunci funcia ) (
o
x
f admite
primitiva |
.
|

\
|
o
o
x
F .
38
Observaia 2. Funciile f i g snt exemple de funcii necontinue (cu
discontinuitate de specia a doua n origine) care admit primitive.
Problema 5. Funciile f : R R , f(x)=

=
=
0 ,
2
1
0 ,
1
sin
2
x
x
x
i g : R R , g(x)=
=

=
=
0 ,
2
1
0 ,
1
cos
2
x
x
x
admit primitive.
Soluie. Avem

=
=
=
0 ,
2
1
0 , )
2
cos 1 (
2
1
) (
x
x
x
x f =

=
=

0 , 0
0 ,
2
cos
2
1
2
1
x
x
x
i analog
pentru g.
Problema 6. Funciile f : R R , f(x)=

=
=
0 , 0
0 ,
1
sin
3
x
x
x
i g : R R , g(x)=
=

=
=
0 , 0
0 ,
1
cos
3
x
x
x
admit primitive.
Soluie.
x x x
1
sin 4
1
sin 3
3
sin
3
= , deci

=
=

=
=
=
0 , 0
0 ,
3
sin
4
1
0 , 0
0 ,
1
sin
4
3
) (
x
x
x
x
x
x
x f ,
fiecare dintre funciile din dreapta admind primitive cf. Problemei 4 i Observaiei 1.
Generalizare. Funcia f : R R , f(x)=

=
=
+
0 , 0
0 ,
1
sin
1 2
x
x
x
n
admite primitive
(O.M.,etapa judeean, 1982).
II. (i) Funciile care nu au P.D. nu admit primitive. n acest sens, innd seama
de proprietile unei funcii cu P.D., putem proceda n mai multe moduri:
- Dac Im f nu este interval, atunci f nu admite primitive;
- Dac f are discontinuiti de specia I , atunci f nu admite primitive.
Problema 7. Funcia f : R R, f(x) =| | x nu admite primitive.
Soluie. Im f = Z , care nu este interval.
Problema 8. Funcia f : R R , f(x) = | | { } x x x = nu admite primitive.
Soluie. Pentru k e Z, avem , 0 ) ( lim , 1 ) ( lim = =
! |
x f x f
k x k x
deci f are
discontinuiti de specia I.
Problema 9. Funcia h : (a,b) R , h(x)=

e
e
Q b a x x g
Q b a x x f
/ ) , ( , ) (
) , ( , ) (
, cu f i g
39
funcii continue distincte, nu admite primitive (O.M., etapa final, 1981).
Soluie. Cum f = g, exist x
0
e (a, b) a.. f(x
0
) = g(x
0
) . Cum R este separat
Hausdorff, putem alege V
f
e V(f(x
0
)) i V
g
e V(g(x
0
)) a.. V
f
V
g
= . Deoarece f, g sunt
continue, exist U
f
,U
g
e V(x
0
) (pe care le putem presupune intervale deschise) a.. x
e (a, b) U
f
f(x) e V
f
i x e (a, b) U
g
g(x) e V
g
. Observm acum c (a, b) U
f
U
g
este un interval, iar imaginea sa prin h este o reuniune de mulimi disjuncte incluse
n V
f
respectiv V
g
,care nu poate fi interval.
Observaie. Particulariznd convenabil funciile f i g, obinem soluii pentru
problemele II.5, 6, pag. 13 din [2].
(ii) Pentru anumite funcii, putem demonstra efectiv c nu admit primitive.
Problema 10. Funcia f
1
: R R , f
1
(x)=

=
=
0 , 0
0 ,
1
cos
1 1
sin
x
x
x x x
nu admite
primitive (v. i Problema 3).
Soluie. Observm c
x x x x
x
1
cos
1 1
sin
1
sin =
'
|
.
|

\
|
, deci o eventual primitiv a
lui
1
f are n mod necesar forma F(x) =

=
= +
0 ,
0 ,
1
sin
2
1
x c
x c
x
x
. Cum F trebuie s fie
continu, atunci c
1
=
2
0
) 0 ( ) ( lim c F x F
x
= =

, deci c
1
= c
2
. Funcia F trebuie s fie
derivabil, deci exist limita ,
1
sin lim
1
sin
lim
0
) 0 ( ) (
lim
0 0 0 x x
x
x
x
F x F
x x x
= =

ceea ce nu este
posibil. Urmeaz c
1
f nu admite primitive.
(iii) Reducere la absurd.
Problema 11. Funcia f : R R , f(x) =

=
=
0 ,
2
1
0 ,
1
sin
x
x
x
nu admite primitive.
Soluie. Presupunem c f ar admite primitive. Cum funcia g(x) =

=
=
0 , 0
0 ,
1
sin
x
x
x
admite primitive (v. Problema 4), atunci diferena lor admite primitive, deci funcia h(x) =
f(x) g(x) =

=
=
0 ,
2
1
0 , 0
x
x
admite primitive. ns Im h = {0,
2
1
} nu este interval , deci h nu
admite primitive. Presupunerea fcut este fals.
Observaia 1. Problema de mai sus furnizeaz un exemplu de funcie cu P.D. care
nu admite primitive.
Observaia 2. Putem da, folosind acest raionament, nenumrate exemple de
funcii cu ramuri care nu admit primitive. De altfel, n unele culegeri de teste gril n
vederea pregtirii a diverse examene, se ntlnesc probleme de forma:
40
Funcia f : R R , f(x) =

=
=
0 ,
0 ,
1
sin
x a
x
x
admite primitive d.n.d. a =
Elevii trebuie obinuii s simt valoarea potrivit a constantei nc nainte de a trece la
rezolvarea propriu-zis a problemei !
III. Apar deseori aplicaii de tipul:
Problema 12. Se consider funcia f : R R , f(x) =

>
+
s
0 ,
1
0 ,
x
x
b x
x ae
x
cu
a, be R. Determinai a, b a.. f s admit primitive.
Metoda uzual de abordare a unor astfel de probleme este cutarea primitivelor lui
f pe ramuri, considerarea unei primitive F sub forma general
F(x)=

> +
+ +
+ +
=
< +
0 ,
) 1 1 (
ln 1 2
0 ,
0 ,
3
2
2
1
x c
x x
x
x
x c
x c ae
b
x
i determinarea apoi a constantelor a i b
a.. F s fie continu i derivabil.
Propus pentru prima oar n clas, problema primete ns de obicei urmtoarea
rezolvare: Pentru ca f s admit primitive, trebuie ca f s fie continu i putem afla a, b
din continuitatea lui f . Lsnd la o parte confuzia care se face ntre condiia necesar i
cea suficient, n contextul dat un astfel de rspuns nu este chiar att de greit. n |4| este
enunat i parial demonstrat urmtorul rezultat (Problema 10, pag. 118):
Teorem. Fie I un interval, x
0
punct interior lui I, iar f : I R o funcie cu P.D.
Dac f are limite laterale n x
0
, atunci f este continu n x
0
.
Demonstraia dat n |4| este valabil numai n cazul limitelor laterale finite
(adic al discontinuitilor de specia I), ns ea poate fi adaptat i n cazul general.
Presupunem, prin reducere la absurd, c f(x
0
- 0) < f(x
0
) (limita putnd fi finit sau -) i
fie o un numr ntre cele dou valori. Atunci Ve V(f(x
0
0)), - U e V(x
0
) a.. x e U,
x < x
0
f(x) e V. Considernd V = (f(x
0
0), o), exist U pe care o putem lua
U = |a, x
0
) a.. x e U f(x) e V ; altfel spus, f(x) < o, x e |a, x
0
). Rezult c f
nu ia valoarea o pentru xe |a, x
0
|, fals.
Revenind la Problema 12, f admite primitive, deci are P.D. n plus, f are limite
laterale n 0 : f(0 0) = a, iar f(0 + 0) =

=
=
1 ,
2
1
1 ,
b
b
. Din teorema de mai sus, rezult c
a=
2
1
, b =1.
Bibliografie
1. V. Arsinte Probleme elementare de calcul integral , Ed. Univ. Bucureti, 1995.
2. N. Boboc, I. Colojoar Analiz matematic , manual cl. a XII-a, E.D.P., 1999.
3. M. Ganga Elemente de analiz matematic , cl. a XII-a, Ed. Mathpress, 1999.
4. Gh. Gussi et al. Analiz matematic , manual cl. a XI-a, E.D.P., 1999.
41
CHESTIUNI COMPLEMENTARE MANUALELOR
Construcii geometrice cu echerul
Eugenia COHAL
1
Nota Redaciei. S-au primit de la mai muli elevi rezolvitori soluii necorespunztoare la problema VII.20
din Rec.Mat-1/2001 : acetia au confundat echerul abstract cu echerul colar ce are un unghi de 30q. Este
binevenit aceast lucrare ce aduce clarificri n aceast privin.
Geometria constructiv sau teoria construciilor geometrice este o parte a geometriei
fascinant prin problematica abordat , vechimea i istoria ei, printr-un numr de probleme
celebre formulate n antichitate i care i-au gsit rezolvarea dup multe secole de cercetri
i cu utilizarea cunotinelor de algebr i analiz matematic. Muli i strlucii
matematicieni au fost atrai de acest domeniu de care i-au legat numele prin contribuiile
aduse.
Rezolvarea problemelor de construcii (geometrice) depinde de instrumentele admise
a fi utilizate. Pe lng rigl i compas , de-a lungul timpului au fost folosite i alte
instrumente : rigla bilateral, rigla cu etalon, distanierul, echerul etc. sau au fost
inventate instrumente speciale cu care puteau fi trasate anumite curbe.
Fiecare instrument (sau grup de instrumente) se definete prin operaiile fundamentale
ce pot fi efectuate cu acesta (acestea).
n prezenta Not atenia se concentreaz asupra unui singur instrument: echerul.
Acesta trebuie imaginat ca dou rigle fixate sub un unghi drept n dou capete ale lor. I se
mai spune echerul drept n contrast cu echerul oblic (colarul). Evident, echerul se
deosebete de instrumentul colar cu acelai nume.
n fapt, echerul este definit de operaiile (construciile) ce se pot face cu el pornind de
la elementele date de problem (puncte, drepte etc.) ct i de la anumite elemente luate
arbitrar. Operaiile fundamentale ce definesc echerul sunt n numr de trei:
1q construirea dreptei ce trece prin dou puncte i determinarea punctului de
intersecie a dou drepte ;
2q construirea perpendicularei la o dreapt ce trece printr-un punct;
3q construirea unui punct pe o drept din care dou puncte se vd sub un unghi
drept.
Dm acum cteva construcii ce pot fi fcute cu ajutorul echerului.
1. Construcia paralelei la o dreapt d printr-un punct P exterior ei. Se construiete
cu echerul dc perpendicular pe d i care trece prin P (2q). Apoi se construiete d c c
perpendicular n P dreptei d c (2q). Evident, d c c este drepta ce trebuia construit.
2. Multiplicarea segmentelor. Pentru a dubla, tripla etc. un segment AB dat procedm
astfel (fig. 1):
Prin punctul A costruim dou drepte d i d
1
(1q), d oarecare iar d
1
perpendicular pe
d (2q). Perpendiculara n B pe dreapta AB (2q) taie dreptele d
1
i d n punctele B
1
,
respectiv B
2
(1q). Fie d
2
perpendiculara n B
2
pe d (2q). Perpendiculara n B
1
pe drepta d
1
se intersecteaz cu d
2
n punctul C
2
(2q, 1q). Perpendiculara pe drepta AB ce trece prin C
2
1
Profesor, Liceul teoretic Mihai Eminescu, Iai
42
intersecteaz dreptele d
1
i AB n punctele C
1
i repectiv C. Obinem similar punctele D
2
,
D
1
, D . a. m. d. Segmentele AB, BC, CD etc. sunt congruente ca proiecii pe dreapta AB
ale segmentelor congruente AB
1
, B
1
C
1
, C
1
D
1
etc.
d2
d1
D2
C2
D
C
B
D1
C1
B1
B 2
d
A
F i g . 1
S R Q P
I
C D B A
Fi g. 2
d
M
B
y
A c O A
Fi g. 3
3. mprirea segmentelor. S urmrim pe fig. 2, mprirea segmentului AB n trei
pri congruente ; avem AB d (construcia 1), segmeentele PQ, QR, RS congruente
(construcia 2) i } {I BP AS . Punctele C i D , ce mpart segmentul AB n trei
pri congruente, se obin ca interseciile dreptei AB cu dreptele RI i QI.
njumtirea se face printr-o construcie mai simpl : se proiecteaz pe AB centrul
unui dreptunghi oarecare construit (cu echerul) pe segmentul AB.
4. Construcia bisectoarei unui unghi . Procedeul este indicat de fig.3, unde s-au luat:
A arbitrar, ' A simetricul lui A fat de O (construcia 2), B se obine punnd vrful
echerului pe latura Oy astfel ca laturile lui s treac prin punctele A i ' A (operaia 3q) i
B A OM ' . Faptul c semidreapta OM este bisectoarea unghiului xOy decurge din
congruena unghiurilor marcate.
Observaie. Dat un triunghi ABC se pot construi cu echerul punctele H, G, O, I.
5. Construcia centrului unui cerc. Fie A un punct pe cercul dat. Punem vrful
echerului n A i marcm cu B i C punctele n care laturile lui intersecteaz cercul.
Dreapta BC trece prin centrul cercului. Lum pe cerc un alt punct ' A i obinem o dreapt
C B c ' n acelai mod. Centrul cercului este punctul de intersecie a acestor drepte.
6. Intersecia unei drepte cu un cerc. Fie dat un cerc printr-un diametru AB
(echivalent, prin centru i un punct al su) i o dreapt d. Conform cu 3q , gsim cu echerul
un punct d P din care diametrul AB se vede sub unghi drept. Atunci, P se afl i pe cerc.
Al doilea punct de intersecie cutat se poate construi (cu echerul) ca simetricul lui P n
raport cu dreapta perpendicular pe d i care trece prin centrul cercului.
Rezolvarea urmtoarelor probleme (cu echerul) o propunem cititorilor ( eventual se
gsete n [2] , p.73 ).
7. Date o dreapt d i un punct d O , s se determine un punct d P astfel nct
AB OP , segmentul AB fiind dat.
8. Date o dreapt d i un punct P , s se duc prin P o dreapt care s formeze cu
d un unghi de mrime egal cu a unui unghi cunoscut.
Observaie . Orice construcie cu rigla i compasul poate fi efectuat cu echerul.
Bibliografie
1. N.F. Cetveruhin Metodele construciilor geometrice (l. rus), Moscova, 1952.
2. A. Tth Noiuni de teoria construciilor geometrice, Ed. Did. i Ped., Bucureti, 1963.
43
DIN ISTORIA MATEMATICII
Teorema celor patru culori

Silviana IONESEI
1
Cum se face c matematica produs
prin excelen al gndirii umane,
independent de experien poate fi
att de admirabil adaptat obiectelor
lumii reale?
Albert Einstein
Este binecunoscut faptul c marile probleme ale matematicii, cum ar fi Marea
Teorem a lui Fermat sau Conjectura lui Goldbach, au contribuit enorm la dezvoltarea
acestei tiine. Din eforturile matematicienilor (timp de zeci sau chiar sute de ani) de a gsi
o rezolvare la ntrebri n aparen simple s-au nscut noi discipline n matematic, cu
aplicaii spectaculoase.
Problema celor patru culori are toate valenele unei probleme de mare carier:
n primul rnd formularea ei este extrem de simpl, nu presupune cunotine matematice; n
al doilea rnd, ea a rmas nerezolvat timp de peste un secol, fiind surprinztor de grea i a
suscitat preocuparea multor matematicieni de prestigiu.
Iat cteva repere istorice.
n 1852 un geograf din Edinburgh (istoria nu i-a reinut numele) l-a informat pe
prietenul su, student n matematici, c folosete cel mult patru culori pentru o hart
mprit n regiuni, fr ca dou regiuni vecine s aib aceeai culoare (precizm c este
vorba despre hri plane, cu regiuni nchise, iar vecine sunt regiunile cu o linie de
frontier comun; dou regiuni care se ntlnesc ntr-un numr finit de puncte nu sunt
considerate vecine).
Tnrului matematician, pe nume Francis Guthrie, i-au plcut cele aflate i a cerut
informaii mai ample, ns geograful l-a ncredinat c acest procedeu e foarte rspndit i
aplicat pretutindeni din cauza economiei care-l prezint. Rspunsul nu a fost mulumitor
pentru Guthrie ; el i-a propus s demonstreze acest fapt dar nu a reuit.
Fratele su, Frederick, studia chimia la Londra i aflnd de problema care-l
preocupa pe Francis a cerut ajutorul profesorului August De Morgan, dar nici acesta nu a
gsit o demonstraie satisfctoare.
n civa ani, problema a ajuns la mod printre matematicieni. Astfel, A. Cayley
nefiind nici el capabil s demonstreze valabilitatea teoremei, a propus-o Societii
Matematice din Londra.
S trecem n revist cteva din rezultatele pariale ale demonstrrii teoremei.
Faptul c trei culori nu sunt suficiente pentru colorarea oricrei hri plane a fost
repede constatat (vezi fig.1).

1
Profesor, Colegiul Naional C. Negruzzi , Iai
44
De Morgan a demonstrat c nu exist hart format din 5 regiuni astfel nct s fie
dou cte dou vecine, deci aceasta poate fi colorat cu patru culori.
A. B. Kempe, un avocat din Londra, membru al Societii Matematice din Londra
i deosebit de pasionat de matematic a publicat n 1879 un articol n care susinea c a
demonstrat teorema. Raionamentul lui era deosebit de ingenios; el redusese problema la
hri normale, adic hri n care nu exist ri nchise complet n alte ri i nici puncte

Fig.1
n care se ntlnesc mai mult de trei regiuni. Dei raionamentul s-a dovedit incomplet, el
coninea ideile de baz ce au condus la demonstraia corect un secol mai trziu. Astfel,
oricrei hri i se poate asocia un graf n care fiecare regiune este reprezentat printr-un
punct i dou puncte vor fi legate printr-o muchie dac i numai dac punctele corespund la
dou regiuni vecine (vezi fig.2).
Fig.2
n acest mod problema colorrii regiunilor de pe hart revine la problema colorrii
punctelor din graful asociat astfel nct punctele legate printr-o muchie s fie colorate
diferit.
Problema celor patru culori a contribuit la cercetari importante n teoria grafurilor,
cum ar fi numerele cromatice ale grafurilor.
Cu ajutorul unui graf special, matematicianul P. J. Heawood a artat n 1890 c
demonstraia lui Kempe are o eroare nu tocmai uor de nlturat.
D
2
4
3 1
E
C
B
A
E
E
C D
A B
45
Mai trziu, n 1913, matematicianul Ph. Franklin de la Massachusetts Institute of
Technology ridic limita numrului de regiuni pentru care problema este rezolvat de la 5
la 21, iar n 1940 Winn reuete s ajung la 35 de regiuni.
Un alt rezultat deosebit de interesant se datoreaz lui P. J. Heawood, care i-a
consacrat 60 de ani din via studierii problemei. Iat-l: probabilitatea de a gsi o hart cu
mai mult de 36 regiuni care s nu poat fi colorat cu patru culori este mai mic dect
10
-10000
! (de remarcat c 10
10000
este un numr mai mare dect numrul atomilor din
ntreaga galaxie ).
O teorem a celor cinci culori (faptul c cinci culori sunt eficiente pentru a
colora o hart) a fost obinut relativ uor; o demonstraie elementar a acestui rezultat
poate fi gsit n Despre numere i figuri de H. Rademacher i O. Toeplitz.
Pe la mijlocul sec. XX s-a conturat ideea de rezolvare a problemei prin mrirea
numrului de regiuni pentru care patru culori sunt suficiente i examinarea unor aa-zise
configuraii inevitabile. Dac ar fi fost posibil s se produc toate aceste configuraii i s se
arate c ele pot fi colorate cu patru culori, atunci demonstraia ar fi fost complet.
Cea mai eficient metod de producere a configuraiilor s-a dovedit a fi un
algoritm implementat pe calculator de W. Haken i K. Appel , Universitatea Illinois, SUA ,
care au lucrat aproape 1200 ore i, n fine, demonstraia a fost ncheiat [1].
Un an mai trziu, folosind o alt procedur de reducere a configuraiilor
inevitabile, F. Allaise de la Universitatea Waterloo, Ontario, CA, a reuit s obin
demonstraia teoremei n numai 50 de ore de dialog om-calculator.
Entuziasmul firesc strnit n lumea matematicienilor de acest reuit neobinuit
pn atunci a fost temperat de voci sceptice care susineau c aceasta nu e o teorema de
matematic n sensul clasic. Astfel, T. Tymoczko n articolul Problema celor patru culori
i semnificaia ei filozofic (Journal of Philosophy, 1979) afirm c teorema exprim un
adevr a posteriori i nu a priori, ca orice adevr matematic. Argumentele sale se bazeaz
n principal pe imposibilitatea de a verifica manual ( cu creionul ) demonstraia, dat fiind
faptul c nu exist un unic algoritm care s verifice toate programele posibile pe calculator.
n replic, E. R. Swart scrie n [5] c inconvenientul semnalat de preopinentul su
este aparent, deoarece calculele foarte numeroase efectuate pe calculator erau de rutin, iar
programul utilizat poate fi verificat. Sarcina calculatorului a fost copleitoare prin
dimensiuni, sarcin pe care omul tia cum s o abordeze, dar n-ar fi putut-o termina
niciodat.
A fost prima situaie memorabil n urma creia lumea matematicienilor a trebuit
s admit existena unor demonstraii parial accesibile omului, ct i dreptul calculatorului
de a ne sprijini n stabilirea adevrurilor matematice.
Bibliografie
1. K. Appel, W. Haken Every Planar Map Is Four Colorable , Bulletin of the American
Mathematical Society, 82(1976), 711-712.
2. F. Cmpan Probleme celebre, Ed. Albatros, Bucureti, 1972.
3. Gh. Pun Din spectacolul matematicii, Ed. Albatros, Bucureti, 1983.
4. L.A. Steen Mathematicians Today, Twelve Informal Essays, SpringerVerlag, 1978.
5. E.R Swart The Philosophical Implications of the Four-Color Problem, The American
Mathematical Monthly, 87(1980), 697-707.
46
MATEMATICA N CLASELE PRIMARE
Introducerea operaiei de adunare la clasa I
Petru ASAFTEI
1
Introducerea operaiei de adunare la clasa I se poate face fie folosind reuniunea a dou
mulimi disjuncte, fie folosind rigletele.
n prezenta not metodic utilizm prima variant, considernd c aceasta este mai
puternic ancorat n experiena de via. Vom putea folosi astfel un vocabular mai bogat,
contribuind astfel la o mai bun nelegere a operaiei de adunare.
n predarea-nvarea noiunilor cu coninut matematic la colarii mici, conform
cerinelor psiho-pedagogice, trebuie s parcurgem urmtoarele etape:
I. Etapa operrii cu mulimile de obiecte concrete (etapa perceptiv). n aceast etap se
realizeaz aciunea nemijlocit cu obiecte concrete din mediul nconjurtor.
Exemplu. Pe o farfurie punem trei mere i patru pere, apoi prin numrare constatm c
avem apte fructe. (Se recomand s folosim cel puin o fraz n care s fie inclui termenii
premergtori operaiei de adunare ; exemplu : Copii, dac la trei mere adugm patru pere,
obinem apte fructe.)
II. Etapa formrii reprezentrilor imaginativ-concrete (etapa semi-abstract). n
aceast etap se construiesc mulimi cu mere i pere
(jetoane), apoi se face reuniunea lor (fig. 1). Aceast
construcie se efectueaz simultan la tabl i pe
caiete. Fiecrei mulimi care intr n reuniune i
atam cifra care indic numrul de fructe (3,
respectiv 4). Mulimii obinut prin reuniune i
atam cifra 7 (prin numrare). Aceast etap putem
s o ncheiem cu o fraz de genul : Copii dac
punem la un loc 3 mere i 4 pere obinem 7 fructe .
III. Etapa scrierii i efecturii adunrii (etapa abstract). Acum reprezentm att merele
ct i perele cu aceleai simboluri (stelue, cerculee etc.) (fig. 2). Se explic elevilor c
pentru a arta faptul c am pus la un loc o mulime cu trei elemente i alta cu patru
elemente se folosete semnul ( simbolul matematic ) ,,+ , numit plus, i scriem 3+4.
Scrierea 3+4 se citete trei plus patru sau trei adunat cu patru.
Din etapele precedente, prin numrare, elevii au
constatat c mulimea obinut prin punere la un loc
are apte elemente. Atenionm elevii ca 3+4 i 7
reprezint tot att. Pentru a exprima acest lucru se
folosete simbolul ,,=" i scriem 3+4=7 se citete
trei plus patru este egal cu apte . Spunem elevilor
c scrierea 3+4 reprezint adunarea neefectuat a
numerelor 3 i 4 iar scrierea 3+4=7 nseamn
efectuarea adunrii numerelor 3 i 4. Explicm

1
Profesor, coala Normal Vasile Lupu, Iai
4 7
Fig. 1
3
4 3+4
Fig. 2
3
47
elevilor c trecerea de la scrierea 3+4=7 s-a fcut prin operaia de adunare.
Trebuie s dm elevilor un algoritm eficient de efectuare a operaiei de adunare n acest
stadiu. S scriem adunarea 5+2 i s numrm de la 5, nainte, nc dou numere n irul
numerelor naturale. Ajungem astfel la numrul 7 i spunem elevilor c 7 este rezultatul
adunrii numerelor 5 i 2. Scriem i n acest caz 5+2=7. Pentru verificarea corectitudinii
rezultatului obinut putem s apelm la procedeul punerii la un loc. n continuare putem s
dm terminologia specific operaiei de adunare. Referindu-ne la ultima scriere 5+2=7
spunem elevilor c numerele 5 i 2 sunt numere care se adun i se numesc termenii
adunrii, primul termen, respectiv, al doilea termen. Despre 7 spunem c este rezultatul
adunrii sau totalul.
Un caz special l reprezint adunarea cnd unul
dintre termeni este zero. Folosim o nou reprezentare
(fig. 3) din care elevii constat c dac punem la un
loc o mulime cu dou elemente i o mulime fr nici
un element, obinem tot o mulime cu dou elemente.
Faptul c am pus elementele celor dou mulimi la un
loc se scrie 2+0. Numrnd elementele noii mulimi
gsim 2 elemente. Cum 2+0 i 2 reprezint tot att
putem scrie 2+0=2. Spunem elevilor c 0 l las pe 2
neschimbat prin operaia de adunare. Cu un demers asemntor se pot justifica i scrierile
0+2=2 i 0+0=0. Deoarece 0 are aceste proprieti l numim element neutru la adunare.
Suntem pregtii s punem n eviden proprietatea de simetrie a relaiei de egalitate
folosind relaia tot att. Exemplu. Deoarece 7 reprezint tot att ca i 5+2, putem s scriem
7=5+2 i 5+2=7.
Dup ce am consolidat algoritmul de adunare, putem s dm i cele dou proprieti
importante ale ei: comutativitatea i asociativitatea. Propunnd elevilor s efectueze
adunrile 5+4 i 4+5 vor constata c obin acelai rezultat, numrul 9, deci 5+4 i 4+5
reprezint tot att. Din acest motiv scriem 5+4=4+5 . Spunem elevilor c rezultatul unei
adunri nu se schimb dac schimbm termenii ntre ei ; n aceasta const proprietatea de
comutativitate. Pe baza proprietii de comutativitate se introduce conceptul de prob a
adunrii prin adunare.
Exemplu. Propunem elevilor s efectueze adunarea 5+3. Numrnd de la 5, nainte, nc
trei numere consecutive, vor obine rezultatul 8. Le spunem elevilor c putem verifica acest
rezultat efectund adunarea 3+5, adic numrnd de la 3, nainte, nc cinci numere
consecutive din irul numerelor naturale. Elevii vor constata c obin acelai rezultat :8 .
Pentru a justifica proprietatea de asociativitate, propunem elevilor s adune numerele 2,
4 i 3 astfel: nti s adune numerele 2 i 4 iar rezultatul obinut s l adune cu 3, apoi s
adune numerele 3 i 4 iar rezultatul s l adune cu 2. n primul caz elevii au efectuat
(2+4)+3 iar n al doilea caz au efectuat (4+3)+2 care este tot att ca i 2+(4+3). Cum n
ambele cazuri elevii au obinut acelai rezultat, numrul 9, putem scrie (2+4)+3=(4+3)+2
i spunem elevilor c au utilizat asocierea, nti a lui 2 cu 4, a doua oar a lui 4 cu 3. Putem
s introducem scrierea 2+4+3, numit sum cu trei termeni, nsemnnd (2+4)+3 sau
(4+3)+2.
Observaii. 1) nvtorul trebuie s explice rolul parantezelor n astfel de scrieri.
2) Dup ce elevii pot s scrie cu uurin aceste proprieti pe exemple numerice, se poate
trece la scrierea literal a lor, fcndu-se meniunea c literele pot reprezenta numere.
0 2+0=2
Fig. 3
2
48
CONCURSURI SI EXAMENE
Concurs de admitere 2001 , Iai
Facultatea de Informatic , Universitatea Al. I. Cuza
Elemente de analiz matematic
I. a)
1. Privitor la numrul de asimptote, care dintre afirmaiile de mai jos pot fi adevrate
pentru o funcie f: RR ?
i) f are dou asimptote orizontale, dou asimptote oblice i o asimptot vertical ;
ii) f are o asimptot orizontal, o asimptot oblic i o asimptot vertical ;
iii) f are o asimptot orizontal, o asimptot oblic i nici o asimptot vertical ;
iv) f are dou asimptote oblice, dou asimptote verticale i nici una orizontal.
Justificai rspunsul (rspunsurile) ales(e).
2. Care din propoziiile de mai jos exprim o proprietate adevrat privind continuitatea
i derivabilitatea?
i) Orice funcie continu ntr-un punct este derivabil n acel punct ;
ii) Orice funcie derivabil ntr-un punct este continu n acel punct ;
iii) Orice funcie continu pe intervalul [a,b] este derivabil pe intervalul (a,b) ;
iv) Orice punct n care o funcie oarecare este derivabil i continu este punct de
extrem al acelei funcii.
3. Pentru o funcie f : IR, I interval deschis, f derivabil pe I, fie aeI un punct de
minim local sau global. Atunci:
i) ) ( ) (
' '
a f a f
d s
= ; ii) 0 ) (
'
> a f ; iii) 0 ) (
'
< a f ; iv) 0 ) (
'
= a f .
4. Fie o funcie oarecare f :[a,b]R, a<b, f derivabil pe intervalul (a,b). Atunci:
i) ntre dou rdcini consecutive ale lui f exist cel mult o rdcin a lui
'
f ;
ii) ntre dou rdcini consecutive ale lui
'
f exist cel mult o rdcin a lui f ;
iii) ntre dou rdcini consecutive ale lui f exist cel puin o rdcin a lui
'
f ;
iv) ntre dou rdcini consecutive ale lui
'
f exist cel puin o rdcin a lui f .
Justificai rspunsul (rspunsurile) ales(e).
5. Care dintre afirmaiile de mai jos sunt adevrate pentru orice ir de numere reale?
i) Orice ir convergent este marginit ; ii) Orice ir convergent este monoton ;
iii) Orice ir mrginit este convergent; iv) Orice ir monoton este convergent .
Not. Rspunsurile se vor da numai pe teza cu coluri nnegrite, sub forma: I.a) 1.x, unde x { i, ii, iii,
iv} etc. Fiecare ntrebare are cel puin una din cele patru variante corect. Dac la o ntrebare sunt
adevrate mai multe variante, atunci se vor indica toate variantele adevrate. Punctajul la o ntrebare nu se
acord dac indicai i variante false. Acolo unde se cere, se va aduga justificarea rspunsului dat.
b) Fie funcia f : RR, .
0 , 1
0 ,
) (

> +
<
=
x x
x x
x f
1. S se studieze continuitatea funciei n punctul x=0.
2. S se studieze derivabilitatea funciei n punctul x=0.
3. S se comenteze rezultatele de la puntele I.b)1. i I.b)2. prin prisma rspunsului
care l-ai dat la punctul I.a) 2 .
49
II. a) Fie aeR, a>0, fixat i ( )
0 > n
n
x irul definit prin relaia de recuren:
, 1 ,
2
1
1
1
>
|
|
.
|

\
|
+ =

n
x
a
x x
n
n n
cu . 2
0
= x S se studieze convergena acestui ir.
b) S se reprezinte grafic funcia f : RR ,
2
2
2
) (
) (
b
a x
e x f

= , a i b constante reale.
Algebra
III. a) Fie (G, -) un grup cu proprietatea c ( )
2 2 2
y x y x = pentru oricare x, y din G.
Demonstrai c G este grup abelian.
b) S se discute dup valorile parametrilor
8 8
, Z b Z a e e i s se rezolve n Z
8
ecuaia
. 0

= + b x a Fiecare caz identificat va fi ilustrat printr-o ecuaie (a=?, b=?) i prin soluiile
respective.
IV. a) Definiia corpului comutativ.
b) Fie a, b, c numere reale. Definim pe R legile de compoziie:
2 + = by ax y x , oricare ar fi x i y numere reale; c y x xy y x + = 2 2 , oricare ar fi
x i y numere reale. Determinai a, b i c astfel nct (R, , ) s fie corp.
Facultatea de automatic i calculatoare, Univ. Tehnic Gh. Asachi
1. Suma cuburilor rdcinilor trinomului , ) 1 ( 3 3
2 2
a x a x + + + aeR, are valoarea
maxim dac:
(a)
4
1
= a
; (b)
4
3
= a
; (c)
4
3
= a
; (d) 0 > a ; (e) ] 1 , 2 [ e a .
2.Mulimea rdcinilor ecuaiei
2 2
2 2
1
b a
x
b
x
a
+ =

, e b a, C, ab0, este:
(a) {0, 1}; (b)

+
+
+

2 2
2
2 2
2
,
b a
ab i a
b a
ab i a
; (c)

+
+
+

2 2
2
2 2
2
,
b a
iab a
b a
iab a
;
(d)

+
+
+

2 2
2
2 2
2
2
,
2
b a
ab i a
b a
ab i a
; (e)

+
+
+

2 2
2
2 2
2
2
,
2
b a
iab a
b a
iab a
.
3. Funcia + ) , 1 ( ) 1 , 0 ( : f R, definit de
x
x x f
ln
1
) ( = , este:
(a) strict descresctoare pe (0, 1) i strict cresctoare pe (1, +);
(b) strict cresctoare pe (0, 1) i strict descresctoare pe (1, +);
(c) injectiv; (d) constant; (e) surjectiv.
4. Fie sistemul

= +
+ = +
a ay x
a y ax
1
1
cu e a R. S se determine a, astfel nct sistemul s fie
compatibil determinat i soluia s satisfac condiia . 0 , 0 > > y x
50
(a) 2 1 2 + < < a ; (b) 1 2 1 < < a ; (c) 1 2 1 < < a
(d) 1 2 2 < < a ; (e) 2 1 2 1 + < < a .
5. Se consider irul ) (
n
x , dat prin relaia
) 1 (
2
1
1
+ =
n n
x x
pentru 1 > n , n care 1
0
< x
este un numr real fixat. S se studieze monotonia irului i s se afle
n
n
x

lim
.
(a) ) (
n
x cresctor i
2 lim =

n
n
x
; (b) ) (
n
x cresctor i
1 lim =

n
n
x
;
(c) ) (
n
x descresctor i
2
1
lim =

n
n
x
; (d) ) (
n
x descresctor i
2 lim =

n
n
x
;
(e) ) (
n
x cresctor i
2
3
lim =

n
n
x
.
6. Se d funcia f : RR prin relaia
x
e
x
m x
x f

+
+
=
1
) (
2
, n care m este un parametru
real. S se afle valoarea lui m pentru care f are extrem n punctul x = 1 i s se precizeze
natura acestui extrem.
(a)
1 ,
2
3
= = x m
punct de maxim; (b) 1 , 3 = = x m punct de minim;
(c)
1 ,
2
1
= = x m
punct de maxim; (d)
1 ,
2
1
= = x m
punct de maxim;
(e)
1 ,
1
= = x
e
m
punct de minim.
7. Valoarea integralei
)
+ +
1
0
2
) 1 ( ) 1 ( x x
dx x
este:
(a)
2
3
=
; (b)
2
1 t +
=
; (c)
) 2 ln 2 (
8
1
= t
; (d)
) 2 ln (
4
1
+ = t
; (e)
) 3 (
4
1
+ = t
.
8. Se consider dreptunghiul ABCD unde AB=3BC. Pe latura CD se ia punctul E astfel
c ECBC , iar AEDB={O}. Se cere msura unghiului DOA.
(a) 30
o
; (b)
3
5
arccos
; (c) 45
o
; (d)
3
5
arcsin
; (e) 60
o
.
9. S se determine soluiile ecuaiei
x x x 2 cos
16
29
cos sin
4 10 10
= +
.
(a)
2 8
t t
k +
; (b)
2 8
t t
k +
; (c)
t
t
k +
8
3
; (d)
4 8
t t
k +
; (e)
8
t
k
(keZ) .
10. O mrgea se obine dintr-un corp sferic sfredelind o gaur de forma unui cilindru
circular drept. tiind c nlimea cilindrului este 6 se cere volumul mrgelei.
(a) 24t ; (b) 36t ; (c) 48t ; (d)
3
100t
; (e)
3
50t
.
Not. Timp de lucru 3 ore. Fiecare problem are numai una din cele cinci variante corect.
51
Capacitate - teste pregtitoare
Ion SECRIERU
1
i Cezar Marius ROMACU
2
Testul 1
A. 1. tiind c viteza luminii este 300 000 km/s, atunci distana de 1 an-lumin este egal
cu. km.
2. n timpul semestrului I al anului colar 2001-2002, un elev a obinut la matematic
notele 7, 7, 9, 7, 7, 7, iar la tez nota 8. Media la sfritul semestrului este .
3. Valoarea raportului
36 din % 12
12 din % 36
este ..
4. Un avion pleac de la Londra la ora 5
40
G.M.T. i ajunge la Bucureti la ora 12
00
ora local. Timpul n care a parcurs avionul distana Londra-Bucureti este..

5. Sistemul are soluiile.
6. Fie punctele A(-2, -2), B(1, 2). Distana AB este.
7. tiind c raza Pmntului este 6400 km, atunci:
a) lungimea Ecuatorului estekm;
b) lungimea paralelei 45
0
este.km.
8. Un teren are forma unui dreptunghi cu lungimea de 150 m i limea de 50 m.
a) Suprafaa acestui teren este..m
2
;
b) Costul aratului pentru un hectar este 1 000 000 lei; atunci aratul terenului
cost lei.
9. O piramid patrulater regulat are latura bazei de 12 cm, iar unghiul diedru dintre o
fa lateral i planul bazei este de 60
0
.
a) Aria seciunii diagonale a piramidei estecm
2
;
b) Volumul piramidei este.cm
3
.
B. 10. La un concurs de matematic, valoarea cumulat a premiilor primilor patru clasai a
fost 3 200 000 lei. Premierea a fost fcut dup cum urmeaz: meniunea primete o optime
din sum, iar premiile III, II i I au fost rspltite direct proporional cu numerele 2,5; 5,
respectiv 6,5. Ce sum a primit fiecare premiant?
11. Fie funciile f, g: RR, f(x) = ax+b, g(x) = 4 2 + x .
a) Determinai funcia f tiind c graficul acesteia trece prin punctele A(1, 1),
B(0, 2);
b) Reprezentai grafic, folosind acelai sistem de axe, funciile f i g;
c) Determinai aria suprafeei cuprinse ntre graficele celor dou funcii i axa Oy.
12. Fie trapezul isoscel ABCD nscris ntr-un cerc de raz 24. tiind c bazele AB i CD
sunt de o parte i de alta a centrului cercului i subntind unghiuri de 120
0
, respectiv 60
0
,
s se calculeze:
a) lungimile bazelor;
b) aria i perimetrul trapezului;

1
Profesor, Grupul colar Economic nr. 2 Virgil Madgearu, Iai
2
Profesor, coala Picioru Lupului, Ciurea (Iai)

+ = +
= + +
9 ) 1 ( ) )( (
10 ) 3 ( ) 1 (
2 2
2 2
y x y x y x
y x
52
c) raportul volumelor corpurilor obinute prin rotirea trapezului o dat n jurul
bazei mici, apoi n jurul bazei mari.
Testul 2
A. 1. Numerele naturale y x3 4 divizibile cu 45 sunt
2. Soluia inecuaiei 0 20 4 ) 5 )( 2 ( s + + + + x x x este ..
3. Calculnd ) 3 2 3 2 4 )( 3 1 ( + + , obinem..
4. Dac numerele naturale a, b sunt astfel nct
7
4
=
b
a
i [a, b] =308, atunci
a+b=..
5. Pentru A={ } 4 1 ; s e x x N ; B={ } 3 1 ; > + e x x R i C=
)
`

+
e 0
3
2
x
x
x Z; , avem
c = C B A .
6. Aria lateral a unui cilindru avnd ca seciune axial un ptrat este egal cu aria
lateral a unui con echilater. Raportul volumelor este
7. Un romb are un unghi de 120
0
i nlimea 3 4 cm.
a) Perimetrul rombului este.cm;
b) Latura unui hexagon regulat echivalent cu rombul estecm.
8. ntr-un cub D C B A ABCD ' ' ' ' distana de la vrful A' la mijlocul E al laturii [BC]
este 15 cm.
a) Aria cubului estecm
2
;
b) Volumul cubului este..cm
3
.
9. Dac la
11
4
dintr-un numr adugm
5
3
din alt numr obinem
55
53
;
5
3
din primul
numr este cu
55
13
mai mare dect
11
4
din al doilea numr.
a) = +
53 13
y x
..;
b) x + y =
B. 10. Se dau proporiile
3 2
b a
= i
5 4
c b
= , cu a, b, ceZ .
a) S se determine k i p aa nct
p
c
k
b a
= =
8
;
b) Pentru k =12 i p=15, s se determine a, b, c n fiecare din urmtoarele cazuri:
(i) a+b+c=70; (ii) a
2
+b
2
+c
2
=433.
11. Pe laturile [BC] i [AD] ale paralelogramului ABCD se construiesc n exterior
ptratele BCMN i ADPQ. S se demonstreze c:
a) [BP] [ND];
b) PN, MQ, AC i BD sunt drepte concurente.
12. O piramid patrulater regulat SABCD are toate muchiile de lungimi egale. tiind
c apotema piramidei este de 3 6 cm , s se calculeze:
53
a) aria lateral i volumul piramidei;
b) unghiul diedru format de planele (SAB) i (ABC);
c) distana de la centrul bazei la o faa lateral.
Testul 3
A. 1. Dac yz xyz = , atunci = xyz
2. Pentru n eZ, fracia Z e
+
+ +
1
6
2
n
n n
dac ne {.}.
3. Rezultatul calcului
2 2 3
2 2 3
2 2 3
2 2 3
+

+
este .
4. ntre bazele trapezului ABCD exist relaiile m
a
=10 i m
g
=5; lungimea
segmentului paralel cu bazele care trece prin punctul de intersecie a diagonalelor este
5. ntr-un paralelogram ABCD (BC>CD), mediatoarea diagonalei [BD] intersecteaz
dreptele AB i CD n E, respectiv F. Patrulaterul BFDE este
6. Lungimea i limea paralelipipedului dreptunghic D C B A ABCD ' ' ' ' sunt valorile
absolute ale soluiilor ecuaiei ,
) 3 4 (
2 2
a x
x a a
a x
a x
a x
a x

=
+

+
aeR
+
, iar nlimea reprezint
jumtate din lungime. Dac M, N sunt mijloacele segmentelor | | AD , respectiv | | B A ' ' ,
atunci d(M, N)=.
7. Seciunea axial a unui con echilater are nlimea de 3 6 cm.
a) Aria total a conului este..cm
2
;
b) Volumul conului estecm
3
.
8. Fie mulimile A=
)
`

s s e

e 4 6 ,
1
6
x
x
x Z Z i B={ } 13 4 3 2 s + s e x x Z .
Atunci A={..} i B={}.
9. Fie mulimile A={-2, 0, 2}i B={-2, 4, 10}. O funcie liniar f: AB are legea de
compoziie f(x)=. sau f(x)=.
B. 10.a) S se compare numerele a=
333
111
5
3
i b=
453
291
5
3
.
b) S se arate c numrul a= 125 2 5
3

+ n n
este divizibil cu 45 pentru orice neN.
11. Fie funciile f, g: RR, f(x) = ax+b, g(x) = cx+d.
a) S se determine f i g tiind c ( ) ( ) 2 5 3 1 f x x f = i ( ) ( ) 5 2 3 2 + = + x g x f ;
b) Determinai valoarea parametrului real m pentru care ecuaia mx
2
+f(x)=0 are
soluiile egale; aflai aceste soluii.
12. Doi elevi privesc simultan i din aceeai parte un turn. Primul vede vrful turnului
sub un unghi de 45
0
, iar al doilea sub un unghi de 30
0
. tiind c nlimea copiilor este de
1,50 m iar distana de la turn pn la al doilea elev este 3 9 m, aflai:
a) nlimea turnului;
b) distana dintre cei doi copii.
54
Testul 4
A. 1. S se determine numerele naturale a i b tiind c (a, b)=15 i ab=3150.
2. Aflai elementele mulimii A=
)
`

e N c abc abc , tiind c a=b=c=a.


3. Gsii numerele naturale a, b, c tiind c a reprezint 20% din b, suma dintre a i c
este 24, iar b i c sunt direct proporionale cu 5 i 7.
4. Dac a
2
+b
2
+c
2
+14=4a+2b+6c, calculai
b
c a
.
5. Determinai funciile liniare f: R R cu proprietatea c f(x-2) f(x+2)=x
2
-2x-3,
xeR.
6. ntr-un triunghi ascuitunghic n care m( ) B

este cu 50
0
mai mare dect m( ) C

se
construiesc nlimea AD i bisectoarea CE. tiind c m( ) D A B

i m( ) A E B

sunt direct
proporionale cu 3 i 4, s se calculeze msurile unghiurilor triunghiului ABC.
7. O prism din lemn are ca baz un ptrat cu diagonala 2 12 cm. tiind c masa
prismei este 30,24 kg, aflai masa unui cilindru din acelai material nscris n prisma dat.
8. Un trapez isoscel are aria 85 cm
2
i nlimea 5 cm. tiind c diferena bazelor
trapezului este 24 cm, aflai perimetrul su.
9. Un paralelogram are laturile de 10 cm, respectiv 14 cm, iar msura unghiului ascuit
este 60
0
. Aflai lungimile nlimilor paralelogramului.
B. 10. Preul unui produs a fost redus de dou ori succesiv, prima oar cu 10% , iar a doua
oar cu 30%, ajungnd n final de 69 300 lei. S se afle:
a) preul iniial al produsului;
b) preul dup dou scumpiri succesive cu 10% i 30%, pornind de la preul final;
c) preul n dolari la un curs al zilei de 32 000 lei, n toate cele trei variante.
11. S se rezolve sistemul

= +

+ = + + +
y
x y y x
x y y y x y x y x
9
4
2
7
2
) ( 2 63 ) )( ( ) 1 (
2
.
12. Apotema bazei unei prisme triunghiulare regulate este 3 , iar diagonala unei fee
laterale este 10 S se calculeze:
a) aria total a prismei;
b) nlimea unui tetraedru regulat ce are o fa echivalent cu o baz a prismei;
c) raportul dintre volumul tetraedrului astfel determinat i volumul prismei.

Testul 5
A. 1. tiind c x i y sunt direct proporionale cu 2 i 3, atunci =
+
+
2 2
2 2
3
2 3
y xy x
y xy x
............
2. Dac (a, b)=6, [a, b]=336 i a< 10, atunci a+b=..............
3. Dac x, y, z sunt invers proporionale cu
8
1
,
5
1
i 3, iar 2x-y+3z=9, atunci y+z
reprezint ............% din x.
55
4. n triunghiul isoscel ABC, [AB] [AC], m )

( C A B =36
0
, BC=4, iar BD este
bisectoare cu De(AC). Atunci ABCD=
5. Cercurile C
1
(O
1
, 3) i C
2
(O
2
, 2) sunt tangente exterior, iar tangentele lor comune
exterioare se intersecteaz n P. Atunci PO
2
=................
6. Aria unui dreptunghi este 60 cm
2
, iar lungimea este cu 7cm mai mare dect limea.
Lungimea diagonalei dreptunghiului este .................
7. Un triunghi dreptunghic are o catet de 12 cm, iar suma dintre cealalt catet i
ipotenuz este 24 cm.
a) Lungimea ipotenuzei este.............cm;
b) Aria triunghiului este ............cm
2
.
8. Fie A=
3 4
1
2 3
1
1 2
1
+
+
+
+
+
, B=
5 3
5 2
5 3
5 2
3 2
3 1
3 2
3 1
+

+
+
+

+
.
a) Numrul A este un numr...........;
b) Produsul AB este egal cu
9. Se d ecuaia mx= 6
2
2
+
m x
, meR.
a) Dac ecuaia are soluia x=3, atunci m=.............;
b) Pentru m=2, ecuaia are soluia x=..............
B. 10. Fie E
1
(x)=
5
:
1
2
1
1
1
1
1
1
1
1
2
x
x
x x
x x

+
+
+

+
+

; E
2
(y)=
y
y y
y
y
y
y
3
9 3
: 1
3
3
2
+
|
|
.
|

\
|

.
a) Aducei expresiile la forma cea mai simpl;
b) Rezolvai sistemul

= +
=
1 ) (
2
1
) (
4 ) ( ) (
5
2
2
2 1
y E x E
y E x E
1
.
11. Msurile unghiurilor A, B, C ale triunghiului ABC sunt direct proporionale cu
numerele 6,5; 5 i 0,5, iar mediatorea laturii [AC] taie latura [BC] n E. Aflai m( ) E A B

.
12. Fie VABC o piramid triunghiular regulat cu nlimea de 2 6 i unghiul diedru
dintre o fa lateral i planul bazei de msur 45
0
.
a) Aflai aria total a piramidei;
b) Determinai aria lateral i volumul trunchiului obinut prin secionarea
piramidei printr-un plan paralel cu baza aflat la o distan de 2 2 cm fa de vrf;
c) Aflai raportul dintre volumele piramidei i cel al unui con circular drept n care
poate fi nscris aceasta.

56
Bacalaureat - teste pregtitoare
Gabriel MRANU
1
Testul 1
I. 1. a) Se consider funciile C B g B A f : , : unde A, B, C sunt submulimi
ale lui R. S se arate c:
(i) dac f g D este injectiv, atunci f este injectiv;
(ii) dac f g D este surjectiv, atunci g este surjectiv.
b) Fie ) , 0 ( ) , 0 ( : f o funcie cu proprietatea ) , 0 ( , ) )( (
2
e = x x x f f D .
S se arate c :
(i) f este o funcie bijectiv;
(ii) ) , 0 ( ), ( ) (
2 2
e = x x f x f ;
(iii) ) , 0 ( ), ( ) ( e = x x f x f .
2. Fie irul
0
) (
> n n
x definit prin :
n n
x x x = =
+
6 , 0
1 0
.
a) S se studieze monotonia i mrginirea irului
0 2
) (
> n n
x .
b) S se studieze monotonia i mrginirea irului
0 1 2
) (
> + n n
x .
c) S se arate c irul
0
) (
> n n
x este convergent i s se calculeze
n
n
x

lim .
3. Fie elipsa de ecuaie : 0 1
2
2
2
2
= +
b
y
a
x
.
a) S se arate c dreapta n mx y + = este tangent elipsei dac i numai dac
2 2 2
n m a n + = .
b) S se determine locul geometric al punctelor din plan din care se pot duce tangente
perpendiculare la elips.
II. 1. Se consider determinantul
4 4 4 4
2 2 2 2
1 1 1 1
x c b a
x c b a
x c b a
= A , unde a, b, c, x e R.
a) Dezvoltai determinantul dup coloana a patra punnd rezultatul sub forma unui
polinom p(x).
b) Determinai coeficientul dominant i rdcinile polinomului p(x).
c) Scriei polinomul sub form de produs.
2. Fie funcia continu ] , [ : b a f R. S se arate c pentru orice cuplu de numere
reale pozitive | o, se poate determina un numr ] , [ b a c e , astfel nct
) ( ) ( ) ( ) ( c f b f a f | o | o + = + .
1
Profesor, Liceul Teoretic Gr. Moisil , Iai
57
III. Fie ) , ( k k G = , unde keR, k>0. Pe G se definete o operaie:
y x k
y x k
y x y x
+
+
= -
2
2
) (
) , ( .
a) Artai c (G, - ) este grup abelian.
b) Artai c funcia G f : R ,
x k
x k
k
x f

+
= ln
2
1
) ( este un izomorfism ntre
grupurile (G, - ) i (R, +).
IV. a) Fie : f RR o funcie continu i periodic de perioad principal T. S se arate c
) )
+
=
nT a
a
T
dx x f n dx x f
0
) ( ) ( , e a R i neN.
b) Folosind punctul a) , s se calculeze :
dx
x
x n
)
+
+

>
c
c
c
c
1
1
2 0
1
arccos sin
lim
0
.
Testul 2
I. 1. Se d o mulime A care are n elemente. mprim toate submulimile lui A n clase
(disjuncte), punnd n aceeai clas toate submulimile lui A care au acelai numr de
elemente. Care dintre aceste clase este cea mai numeroas ?
2. Fie f : R \ {1, 2}R,
2 3
1
) (
2
+
=
x x
x f i ) 3 (
!
1
) (
0
2
+ =
_
=
n f
k
n a
k
n
k
n
.
a) S se determine determine derivata de ordin n a lui x.
b) S se calculeze
n
n
a

lim
3. Se consider punctele A(-2,1), B(2,-1), C(-1,8).
a) S se calculeze perimetrul triunghiului ABC.
b) S se determine coordonatele punctului D , al patrulea vrf al paralelogramului
ABCD , pentru care BC este o diagonal.
c) S se scrie ecuaia cercului circumscris triunghiului ABC.
II. 1. Fie familia de funcii de gradul al doilea :
, 1 ) 1 ( ) (
2
+ + = x m x m x f
m
, MeR\{-1}
a) S se arate c parabolele asociate acestor funcii trec prin dou puncte fixe.
b) S se determine locul geometric al punctelor descris de vrfurile parabolelor.
2. S se determine o primitiv a funciei ] 1 , 0 [ : f R , ) 1 1 ln( ) ( x x x f + = .
III. Fie polinomul feZ
6
[X] , 3

2 3 4
+ + + + = x x x x f .
a) S se arate c f nu admite rdcini n Z
6.
b) S se arate c f se poate descompune n produsul a dou polinoame , din care unul
de gradul nti.
IV. Fie funcia D f : R ,
x
x x
x f
ln 0 1
1 1 0
0 ln
) ( = .
a) S se determine domeniul maxim de definiie D i apoi s se studieze continuitatea
i derivabilitatea funciei f.
58
b) S se arate c restricia lui f la ) , 0 ( este bijectiv i apoi s se calculeze
( ) ( ) ) 0 ( ), 0 (
1 1
" '

f f .
c) S se determine primitivele lui f pe R*.
d).S se calculeze aria suprafeei plane determinate de graficul funciei f , axa Ox i
dreptele x=1, x=3.
Testul 3
I. 1. S se rezolve inecuaia : 0 2 3 8 2 3 8 2
3
s + + +
x x x x
.
2. Fie xeR \ {1} i
n
x x x x S + + + = !
2
) ( .
a) S se calculeze S(x) i ) (x S' .
b) Folosind a) s se calculeze
n
nx x x x x T + + + + = !
3 2
3 2 ) ( .
c) S se calculeze limita irului
1
) (
> n n
a cu
_
=
=
n
k
k
n
k
a
1
2
.
II. 1. Fie M mulimea matricilor de tipul (m, n) n care toate elementele sunt egale cu 1
sau (1) i astfel nct produsul numerelor din fiecare linie, respectiv coloan s fie egal
cu (1). S se calculeze numrul de elemente al mulimii M .
2. a) Enunai teorema lui Lagrange.
b) Aplicai teorema lui Lagrange funciei ] , [ : t e f R, x x f ln ) ( = .
c) Demonstrai inegalitatea :
e
e t
t
> .
III. 1. Fie { } 0 *, , ) ( ) ( ) , ( : > e + = = b R b x b x f R b f G o
o
o o
.
a) S se arate c ) , ( D G este grup abelian.
b) S se demonstreze c ) , ( ) , ( ~ R G D .
2. Fie PeZ[X] un polinom pentru care P(0), P(1) sunt impare. S se arate c ecuaia
P(x)=0 nu are rdcini ntregi.
IV. 1. Fie funcia I f : R , I interval inclus n R aeI i funcia f este derivabil n a.
S se calculeze
(

+ + + + +

) ( ) ( )
2
( )
1
( lim a kf
n
k
a f
n
a f
n
a f n
n
" , unde keN* este fixat.
2. Fie irul
1
) (
> n n
a cu termenul general dx x x a
n
n )
=
1
0
) 1 ( , neN*.
a) S se determine a
n
.
b) S se calculeze
_
=
=
n
k
k n
a b
1
.
c) S se calculeze
n
n
b

lim .
Testul 4
I. 1. Fie ! ! , , , ,
2 1 n
x x x un ir de numere reale nenule. S se arate c acest ir este o
progresie aritmetic , dac i numai dac pentru orice 2 > n , avem relaia :
59
n n n
x x
n
x x x x x x
1 1 3 2 2 1
1 1 1 1
= + + +

" .
2. Se consider irul
0
) (
> n n
x , arctgn n n x
n
2 = t .
a) S se arate c
0
) (
> n n
x este un ir monoton i mrginit.
b) S se calculeze
n
n
x

lim .
3. S se demonstreze c produsul distanelor unui punct oarecare al hiperbolei
0 1
2
2
2
2
=
b
y
a
x
, la cele dou asimptote , este egal cu
2 2
2 2
b a
b a
+
.
II. 1. Determinai prile stabile finite ale lui Z n raport cu nmulirea. Este R\Q parte
stabil a lui R n raport cu adunarea , respectiv cu nmulirea ?
2. Fie funcia ] , 0 [ : x f R , ) 1 , 0 ( ,
1
1
ln ) (
0
e
+

=
)
x dt
t
t
x f
x
.
a) S se calculeze ) (x f ' .
b) S se calculeze
2
0
arcsin
) (
lim
x
x f
x
.
III. Fie ) , ( G i ) , ( ' G dou grupuri, G G f ' : un morfism de grupuri i mulimea
Ker } ) ( { e x f G x f ' = e = , e' fiind elementul neutru al lui G' .
a) S se arate c Kerf este subgrup al lui G.
b) S se arate c morfismul f este injectiv dac i numai dac Kerf={e}.
IV. 1. Fie funcia ] 1 , 0 [ : g R ,

e
e
=
}
1
,
2
1
, 1 { , 3
}
1
,
2
1
, 1 { \ ] 1 , 0 [ ,
) (
3
n
x
n
x x
x g
x
!
!
neN*.
S se arate c g este integrabil pe [0, 1] i s se calculeze
)
1
0
) ( dx x g .
2. Se consider funcia ) , 0 ( : f R,
x
x
x
p
x f
ln
) ( = , neN*.
S se calculeze S(m) , aria suprafeei mrginit de graficul funciei f, axa Ox, i dreptele
x=1 , x=m (m >1). S se determine m, astfel nct 2 / 1 ln ) ( = m p m S .
Testul 5
I. 1. Fie irul de numere complexe
1
) (
> n n
z definit prin ) ( , ,
2 1 1 2 1
= = =
n n n n
z z q z z i z i z
pentru 2 > n , q fiind un numr complex dat diferit de 1.
a) Calculai z
3
i z
4
.
b) Artai c
2
1
2

=
n
n n
iq z z i deducei c
q
q
i z z
n
n

1
1
2
1
1
.
c) Artai c afirmaiile i z
n
= i q este rdcin de ordinul n-1 a unitii sunt
echivalente.
60
2. a) Definii noiunea de punct de inflexiune al unei funcii i dai interpretarea
geometric.
b) S se determine punctele de inflexiune ale funciei : f : RR,
3 3
1 1 ) ( + + = x x x f .
3. Fie elipsa (E) : 0 1
3 4
2 2
= +
y x
.
a) S se scrie ecuaiile tangentelor la elips, duse din punctul A(-16, 9).
b) Dac se noteaz cu T
1
, T
2
punctele de contact ale celor dou tangente de la
punctul a) cu elipsa (E) , s se determine ecuaia dreptei T
1
T
2.
c) Calculai distana de la punctul A la dreapta T
1
T
2.
II. 1. a) Rezolvai n R inecuatia .
4
3 1 1
2
>

x
x
b) S se gseasc rangul celui mai mare termen din dezvoltarea
2002
7
1
7
6
|
.
|

\
|
+ .
2. a) Artai c funcia f : RR,

e
e
=
Q R x
Q x
x f
\ , 0
, 1
) ( nu admite primitive pe R.
b) Gsii o funcie g : RR care nu admite primitive pe R , astfel nct funcia
compus g f D s admit primitive pe R (f fiind funcia de la punctul a)).
III. 1.Artai c polinomul 3 3
2
+ + = x x f divide polinomul, n x x g
n
+ + + =
+
, 2 ) 1 (
2 3
eN*.
2. Fie e b a

, Z
n
, i ecuaia b x a

= . Artai c:
a) dac (a, n)=1, atunci ecuaia b x a

= admite soluie unic.


b) dac (a, n)=d > 1 i d nu divide pe b, atunci ecuaia b x a

= nu are soluii.
IV. 1. Fie funcia ) , 0 [ : f R ,

=
e
=
0 , 0
) , 0 ( , ln
) (
x
x x x x
x f .
a) Studiai continuitatea i derivabilitatea lui f pe [0, )
b) Studiati variatia funciei f i reprezentai graficul ei.
c) Artati c restricia lui f la [e, ) are invers, f
- 1
, i studiai derivabilitatea lui f
- 1
.
d) Calculai ) (
3
e f ' i ) 2 ( ) (
2 1
e f ' '

.
e) Calculati aria suprafeei plane mrginit de graficul funciei f , axa Ox, i dreptele
2
, e x e x = = .
2. Fie ) , 0 ( : f R,
4 2
1 ) (
2
x x e x f
x
= .
a) S se arate c . 0 , 0 ) ( > > ' x x f
b) S se arate c . 0 , 0 ) ( > > x x f
c) S se demonstreze c : 43 , 1
1
0
2
>
)
dx e
x
.
61
PROBLEME SI SOLUTII
Soluiile problemelor propuse n nr. 1/2001
Clasele primare
P.7. Crizantema are cu 38 timbre mai puin dect colega ei, Maria. Cte timbre
trebuie s mai cumpere Crizantema pentru a avea cu cel mult 4 timbre n plus fa de
Maria?
Crizantema Mironeanu, elev, Iai
Soluie. Pentru a o egala pe Maria, Crizantema mai are nevoie de 38 timbre. Pentru a
o depi cu cel mult 4 timbre, ea trebuie s mai cumpere, pe lng cele 38, nc 1,2,3 sau 4
timbre. Deci Crizantema trebuie s mai cumpere 39,40,41 sau 42 timbre.
P.8. S se arate c din numerele 1,2,3,,10 nu se pot forma dou iruri de numere,
cu acelai numr de numere, astfel nct adunnd numerele din fiecare ir s obinem sume
egale.
Maria Mursa, elev, Iai
Soluie. Suma numerelor 1,2,3,,10 este 55. Dac ar exista cele dou iruri de
numere cu sume egale, atunci cele dou sume adunate ar trebui s dea 55. Acest lucru este
imposibil deoarece nu exist dou numere naturale egale care s dea suma 55.
P.9. Aflai vrsta n prezent a tatlui unui biat tiind c biatul are 7 ani, iar atunci
cnd biatul va avea vrsta tatlui, tatl va avea 55 ani.
nv. Elena Marchitan, Iai
Soluie. S figurm cele dou vrste innd cont de relaiile dintre ele. Notm cu t
vrsta tatlui i cu f vrsta fiului.
Se observ c vrsta de 55 de ani este format din
vrsta fiului i din dublul diferenei dintre vrsta tatlui
i a fiului n prezent.
1. Care este dublul diferenei dintre cele dou vrste n prezent? 55 7 = 48
2. Care este diferena dintre cele dou vrste n prezent? 48 : 2 = 24
3. Care este vrsta tatlui n prezent? 24 + 7 = 31
R : 31
P. 10. George i-a propus s citeasc n cinci zile o carte ce are 42 file. Numrul
filelor citite n primele trei zile este reprezentat de numere pare consecutive. n a patra i a
cincea zi a citit 12 file. tiind c n ultima zi a citit de dou ori mai mult dect n ziua
precedent, s se afle cte file a citit George n fiecare zi.
nv. Geta Dragnea, Iai
Soluie. S figurm numrul de file citite de George n fiecare zi
I zi
a II-a zi
a III-a zi
a IV-a zi
a V-a zi
1. Cte file a citit n primele trei zile? 42 12 = 30
55 ani
t
f
7
(42-12) file
12 file
2
2
62
2. Care este triplul numrului de file citit n prima zi? 30 6 = 24
3. Cte file a citit n prima zi? 24 : 3 = 8
4. Cte file a citit n a doua zi? 8 + 2 = 10
5. Cte file a citit n a treia zi? 10 + 2 = 12
6. Cte file a citit n a patra zi? 12 : 3 = 4
7. Cte file a citit n a cincea zi? 4
.
2 =8
R : 8 file, 10 file, 12 file, 4file, 8 file.
P. 11. Ci spectatori au fost asear la Teatrul Naional Vasile Alexandri, din Iai,
dac la balcon au fost 160 de spectatori, la loj un sfert din restul spectatorilor, iar la stal
cu 80 spectatori mai mult dect la loj i balcon mpreun?
nv. Rodica Agrici, Iai
Soluie. S figurm repartiia spectatorilor, n fiind numrul total.

1. Ct reprezint jumtate din restul spectatorilor? 80 + 160 = 240
2. Ci spectatori au stat la loj i stal? 240
.
2 = 480
3. Ci spectatori au fost la teatru? 480 + 160 = 640
R : 640 spectatori
P.12. Mo Crciun mparte daruri elevilor clasei a IV-a. Dac ar da fiecrui copil
cte 2 pachete, ar rmne n sac 2 pachete. Dac ar oferi fiecrui copil cte 3 pachete, ar
rmne 9 copii fr daruri. Cte pachete are Mo Crciun n sac?
nv. Fnic Dragnea, Iai
Soluie. Figurm cele dou situaii din problem folosind simbolurile E (elev) i P
(pachet) Primul rnd de simboluri sugereaz fiecare elev a primit cte 2 pachete 2 pachete
au rmas nerepartizate.
Al doilea rnd de simboluri sugereaz
c primii elevi au primit cte 3 pachete iar
ultimii 9 elevi nu au nici un pachet. Putem
considera c al treilea pachet a fost oferit
ultimilor 9 elevi. Acum putem scrie:
1.Ci elevi au primit cte 3 pachete? 2
.
9 + 2
.
1 = 20
2.Cte pachete are Mo Crciun? 20
.
3 = 60
R : 60 pachete
P.13. Scriitorul Ion Creang a publicat povestea Capra cu trei iezi n 1875. Se
spune c pe atunci capra ar fi avut o vrst egal cu dublul sumei vrstelor ieziorilor ei,
anii acestora fiind exprimai prin numere naturale consecutive. Peste un an, cnd s-a
abtut necazul asupra caprei, lupul avea vrsta egal cu dublul sumei vrstelor de atunci
ale iezilor, iar toi cinci aveau mpreun 40 ani. Ce vrst avea fiecare n anul publicrii
acestei poveti?
nv. Mihai Agrici, Iai
Soluie. Figurm datele corespunztoare anilor 1875 i 1876.
I1
I2
I3
C
balcon restul spectatorilor
loj
loj
stal
80+160
n
PP
elevi
PP PP PP PP PP
E E E E E
, ......... ..... ..........
9

PP
elevi
PP PP PP PP PP
E E E E E
, ......... ..... ..........
9


elevi
PP
P
PP
P
PP
P
E E E E E E
9
......... .........
de 9 ori segmentul I
1
i nc 9 ani
1
1
1 1 1 1 1 1
63
I
1
+1
I
2
+1
I
3
+1
C+1
L+1
Unde I
1
, I
2
, I
3
, C, L reprezint vrstele celor cinci vieuitoare n anul 1875. Analiznd
figurarea corespunztoare anului 1876, putem scrie:
1. Care este numrul segmentelor ce reprezint vrsta I1? 3
.
1+ 2
.
6 = 15
2. Ci ani reprezint aceste segmente? 40 25 = 15
3. Ce vrst avea mezinul? 15 : 15 = 1
4. Ce vrst avea iedul mijlociu? 1 + 1 = 2
5. Ce vrst avea iedul cel mare? 2 + 1 = 3
6. Ce vrst avea capra? (1+2+3)
.
2 = 12
7. Ce vrst avea lupul? (2+3+4)
.
2 1 = 17
R : 1an, 2ani, 3ani, 12ani, 17ani.
Clasa a V-a
V.16. Un automobilist vede la un moment dat pe kilometrajul de la bord numrul
12921. Dup dou ore de mers cu vitez constant, pe kilometraj a aprut urmtorul
numr care se citete la fel n ambele sensuri. Aflai viteza de deplasare a automobilului.
Gabriel Mranu, Iai
Soluie. Urmtorul numr care se citete la fel n ambele sensuri este 13031. n dou
ore de mers automobilistul a parcurs cu vitez constant distana de 110 km = 13031 km
12921 km, deci viteza sa a fost de 55 km/h.
V.17. S se arate c fiecare termen al irului: 19204, 9012004, 900120004, ... este un
ptrat perfect.
Constantin Chiril, Iai
Soluia 1. Se observ c 9120 = 302
2
, 9012004 = 3002
2
etc. n general, se pare c
2
1 1
2 0 ..... 00 3 4 0 ..... 00 12 0 ..... 00 9

+ +
=
p p p
. Se verific prin ridicare la ptrat c egalitatea este
adevrat; ca urmare, numerele din ir sunt ptrate perfecte.
Soluia 2 (Schibinschi Greta, Botoani). Scriem:
. 2 0 ..... 0 3 ) 2 10 3 ( ) 2 10 3 ( 2 ) 2 10 3 ( 10 3
4 10 6 10 6 10 9 4 10 12 10 9 4 0 ..... 0 12 0 ..... 0 9
2
1
2 2 2 2 2
2 2 4 2 2 4 2
1


+
+ + + +
+ + + + +
+
= + = + + + =
= + + + = + + =
p
p p p p
p p p p p
p p
V.18. Artai c, dac suma a n numere naturale nenule este un numr prim, atunci
aceste numere sunt prime ntre ele.
Cristiana Artenie, elev, Iai
Soluie. Fie p = a
1
+a
2
+...+a
n
, p numr prim i d = (a
1
, a
2
, ... , a
n
). Atunci d |a
1
, d |
a
2
,
, d | a
n
i deci d | a
1
+a
2
+ ... + a
n
, adic d | p. Ca urmare sau d = 1, sau d = p; artm c nu
putem avea d = p i de aici va rezulta concluzia. ntr-adevr, dac d = p, atunci numerele de
date se scriu:
n n
a p a a p a ' = ' = ,...,
1 1
i vom avea p a p a p a p
n
= ' + + ' + ' ...
2 1
sau
1 ...
2 1
= ' + + ' + '
n
a a a , ceea ce nu se poate!
1
1
1 1 1 1 1 1 1
1
40 ani
1 1 1 1 1 1 1 1 1 1 1 1
64
V.19. Fie a, b, p, q e N
*
. Aflai valorile pe care le poate lua numrul (1+A)
B
,
unde

termeni p
b a b a b a A ... + + + = i

termeni q
a b a b a b B ... + + + =
Cristiana Constanda, elev, Iai
Soluie. Observm c numrul A are valorile 0, a, a + b sau b, dup cum p = M4, p =
M4 + 1, respectiv p = M4 + 3. Ca urmare, 1+Ae{1,1+a,1+a+b, 1+b}. n mod analog, B
e{0, b, a+b, a}. n consecin, avem: (1+A)
B
e {1, (1+a)
a
, (1+b)
b
, (1+a)
a+b
, (1+b)
a
, (1+b)
b
,
(1+b)
a+b
, (1+a+b)
0
, (1+a+b)
b
, (1+a+b)
a+b
}.
V.20. S se aranjeze 12 puncte pe 6 drepte astfel nct pe fiecare dreapt s fie
situate 4 puncte (indicai cel puin dou aranjamente de acest fel).
Andrea Balla, elev, Braov
Soluie. Reproducem trei dintre numeroasele soluii primite de redacie.
Clasa a VI-a
VI.16. Fie a i b dou numere ntregi. Artai echivalena afirmaiilor urmtoare: 1
1000a+b# 43; 2 a+4b# 43; 3 11b-8a# 43; 4 7b-9a# 43.
Gheorghe Costovici, Iai
Soluie. Vom arta c 1 2 3 4 1. Avem:
1 1000a+b = 43k (2343+11)a+b = 43k 2343a+11a+b = 43k
42343a+44a+4b = 443k 42343a+43a+(a+4b) = 443k a+4b # 43;
2a+4b = 43k -8a32b = -843k (11b8a) 43b = -843k 11b-8a# 43;
3 11b8a = 43k 99b72a = 943k 43b+56b72a = 943k 43b+8(7b -
9a) = 943k 7b9a# 43;
4 7b9a = 43k 1357b1359a = 13543k 945b1215a = 13543k 946bb
1000a 215a = 13543k 43(22b5a)(1000a+b) =13543k 1000a+b# 43.
VI.17. Fie E = 2
4n+2
+ 3
4n
+ 4
4n
+ 5
2n
+ 6
2n
, n e N.
1) Artai c E nu este ptrat perfect.
2) Aflai n astfel nct E# 9.
Cristiana Constanda, elev, Iai
Soluie. 1) Dac n = 0, atunci E = 8 i nu este ptrat perfect. Dac n > 1, atunci ultima
cifr a numrului E este ultima cifr a sumei 4+1+6+5+6 = 22, deci E se termin n 2 i de
aici rezult c nu poate fi ptrat perfect.
2) Dac n = 0, E = 8 nu se divide prin 9. Dac n > 1, atunci 6
2n
# 9 i deci avem:
65
E# 9 2
4n+2
+4
4n
+5
2n
# 9 2
n+2
2
3n
+ 16
2n
+ (9-4)
2n
# 9 2
n+2
(9-1)
n
+ 2
2n
(9-1)
2n
+ 4
2n
# 9
(-1)
n
2
n+2
+2
2n
+2
n
(9-1)
n
# 9 (-1)
n
2
n
4+2
n
2
n
+(-1)
n
2
n
# 9 5(-1)
n
+2
n
# 9.
Dac n = 6k (k > 1), atunci 5(-1)
n
+2
n
# 9 (5+2
6k
) # 9 5+(9-1)
2k
# 9 (5+1)# 9,
fapt care nu este adevrat. La fel se arat c E nu se divide cu 9 dac n = 6k+1, n = 6k+3
sau n = 6k+4 (k > 0).
Dac n = 6k+2 sau n = 6k+5 (k > 0), atunci E se divide cu 9, deoarece:
5(-1)
6k+1
+2
6k+2
# 9 5+4(9-1)
2k
# 9 (5+4) # 9 etc.
V.18. S se descompun n factori primi numrul S dat de:
S = 123456789 + 234567891 + 345678912 + ... +12345678.
Paraschiva Brsan, Iai
Soluie. Scriind reprezentarea n baza 10 al fiecrui termen din suma S, obinem :
S = (1+2+ ... +9)10
8
+ (1+2+ ... +9)10
7
+ ... + (1+2+ ... +9) =
=45(10
8
+10
7
+ ... +10 + 1) = 45[10
6
(10
2
+10+1) + 10
3
(10
2
+10+1) + (10
2
+10+1)]=
= 45(10
2
+10+1)(10
6
+10
3
+1) = 3
2
51111001001 = 3
4
537333667,
aceasta fiind descompunerea n factori primi a numrului S.
VI.19. S se afle cinci numere raionale tiind c:a) suma lor este 351; b) primele trei
sunt invers proporionale cu primele trei numere prime; c) ultimele trei sunt direct
proporionale cu 7, 11 i 13.
Cristiana Artenie, elev, Iai.
Soluie. Fie a, b, c, d i e cele cinci numere. Condiiile din enun se scriu:
a+b+c+d+e = 351, 2a = 3b = 5c i
13 11 7
e d c
= = . Din ultimele relaii deducem c
2
5c
a = ,
3
5c
b = ,
7
11c
d = ,
7
13c
e = i nlocuind n prima egalitate obinem o ecuaie cu necunoscuta
c etc.
VI.20. Fie a, be N i ce Q direct proporionale cu p
1
, p
2
, p
3
, unde p
1
< p
2
< p
3
sunt
numere prime.
a) Artai c c e N
*
.
b) Determinai p
1
, p
2
, p
3
dac a+b < 35 = c.
Gheorghe Iurea, Iai
Soluie. a) Din
3 2 1
p
c
p
b
p
a
= = deducem ap
2
= bp
1
i
3
1
p
p
a
c = . Din prima egalitate
rezult p
1
| ap
2
i deci p
1
| a, adic k
p
a
=
1
e N
*
. Folosind a doua egalitate, obinem c e
N
*
.
b) Avem: a+b < 35 = c k (p
1
+ p
2
) < 35 = kp
3
35 = kp
3
i p
1
+ p
2
< p
3
p
3
e{5,
7} i p
1
+p
2
< p
3
. Dac p
3
= 3, atunci p
1
= 2 i p
2
= 3 (deoarece p
1
< p
2
< p
3
) i nu avem p
1
+p
2
< p
3
. Rmne p
3
=7, caz n care inegalitatea p
1
+p
2
< p
3
este ndeplinit numai de numerele
prime p
1
= 2, p
2
= 3.
Clasa a VII-a
VII.16. S se cerceteze care dintre elementele mulimii A={(x,y)eQQ; 4x
2
+12x+9 +
|y
2
-25|=0} aparin graficului funciei f(x) = -2x+2, xeR.
66
Cristiana Constanda, elev, Iai
Soluie. Avem: A ={(x,y)eQQ; (2x+3)
2
+|y
2
-25| = 0} = {(x,y)eQQ; 2x+3 = 0 i y
2
-
25 = 0} = {(x,y)eQQ; x =
2
3
i y = 5 }=
)
`

5) ,
2
3
(- 5), - ,
2
3
- ( . Cum f(
2
3
) = 5, numai
al doilea punct al lui A aparine graficului lui f.
VII.17. a) Fie x, y, z e [2, +). Artai c (x
2
+y)(y
2
+z)(z
2
+x) > 27xyz.
b) Fie x, y, z e [3, +). Artai c (x
2
+y)(y
2
+z)(z
2
+x) > 64xyz.
Lucian Tuescu, Craiova
Mai general, pentru orice neN fixat are loc inegalitatea:
(x
2
+y)(y
2
+z)(z
2
+x) > (n+1)
3
xyz, x, y, z e[n, +). (1)
(pentru n = 2 i n = 3 se obin inegalitile din enun).
Soluia 1 (n maniera autorului). Cum x > n x
2
> nx x
2
+y > nx+y i cum nx+y =
x++x+y > (n+1)
1 + n n
y x , obinem x
2
+y > (n+1)
1 + n n
y x . Analog, y
2
+z >
>(n+1)
1 + n
n
z y , y
n
+x > (n+1)
1 + n n
x z . Prin nmulirea membru cu membru a ultimelor trei
inegaliti se obine (1).
Soluia 2 (n maniera mai multor elevi din Braov). Are loc x
2
+y > (n+1)x, x, y e
[n, +) cci x
2
+ y > (n+1)x x
2
(n+1)x + n + (y-n) > 0 (x-1)(x-n) + (y-n) > 0.
nmulind membru cu membru inegalitatea x
2
+ y > (n+1)x cu analoagele ei, obinem (1).
VII.18. S se determine numerele reale x i y pentru expresia
2 2 2 2
) ( ) ( a y a x y x + + + + +
2 2 2 2
) ( ) ( ) ( c y x a y b x + + + (a,b,ce R
+
) este minim
i s se afle apoi aceast valoare minim.
Cristiana Artenie, elev, Iai
Soluie. n raport cu un sistem Oxz de axe coordonate figurm punctele O(0,0),
A(-a,a), B(b,a), C(0,c) i M(x,y). Deoarece
2 2
y x + = MO,
2 2
) ( ) ( a y a x + + = MA,
2 2
) ( ) ( a y b x + =MB i
2 2
) ( c y x + =MC,
problema revine la determinarea minimului
expresiei E = MO+MA+MB+MC, atunci cnd
M este un punct oarecare din plan.
Avem inegalitile: MO + MC > OC
(AMOC), MA+MB > AB (AMAB), deci E > >
AB+OC, cu egalitate pentru M e (OC)
(AB), adic M coincide cu P(0,a). Ca urmare,
E
min
se obine pentru x = 0, y = a i are valoarea E
min
= 2a+b+|a c|.
VII.19. Se consider un triunghi dreptunghic isoscel ABC cu vrful n A i se noteaz
cu E i F punctele de intersecie ale cercurilor C(C,
4
3
BC) i C(A,
4
1
BC). S se arate c
punctele B, E i F sunt coliniare.
67
Adrian Zanoschi, Iai
Soluie. Fie AB = AC = a. Avem: AE = =AF
=
4
BC
=
4
2 a
i CE = CF =
4
3BC
=
4
2 3a
. Se
observ de aici c
AE
2
+ AC
2
=
2
2
16
2
a
a
+ =
16
18
2
a
=EC
2
, deci AAEC
este dreptunghic n A, adic AC AE , de unde
rezult c EeAB. Analog se arat c FeAB, deci
B, E, F sunt coliniare.
VII.20. S se mpart cu ajutorul unui echer negradat un segment [AB] n trei pri
de lungimi egale.
Constatin Cocea, Iai
Soluie. Vezi E. Cohal - Construcii geometrice cu echerul, p.41 din acest
numr.
Clasa a VIII-a
VIII.16. Fie neN
*
i A
n
={ }
n
x x x ...
2 1
. Definim funcia d: A
n
A
n
N prin d(x, y) =
Card { }
i i
y x , n i i = e , 1 / , unde x =
n
x x x ...
2 1
i y =
n
y y y ...
2 1
.
S se arate c d este o distan, adic satisface condiiile: 1) d(x, y) = 0 x = y; 2)
d(x,y) = d(y,x), x,y e A
n
; 3) d(x,y) s d(x,z)+d(z,y), x,y,z e A
n
.
Petru Asaftei, Iai
Soluie. Verificm pe rnd 1), 2) i 3):
1. d(x, y) = 0 x
1
= y
1
, x
2
= y
2
, ... , x
n
= y
n
x = y.
2. d(x, y) = d(y, x), x,yeA
n
, cci perechile de numere (x, y) i (y, x) au aceleai cifre
distincte.
3. S presupunem c d(x, y) = k = Card{i
1
, i
2
, , i
k
}(deci
1 1
i i
y x = , ,
n n
i i
y x = i x
j
=
y
j
pentru je{i
1
, ..., i
k
}) i fie z =
n
z z z , ... , ,
2 1
. Fie p numrul indicilor din {i
1
, ..., i
k
} pentru
care coincid cifrele lui z i x i fie q numrul acelora pentru care coincid cifrele lui z i y.
Evident, avem p+qs k. Deoarece
1 1
i i
x z =
1 1
i i
y z = i
1 1
i i
y z =
1 1
i i
x z = etc., rezult c
d(x, z) > k-q i d(z, y) > k-p. Ca urmare, d(x, z) + d(z, y) > 2k- (p+q) > k = d(x, y), adic este
verificat 3).
VIII.17. Artai c oe(0,1), tiind c numrul real o este o soluie a ecuaiei
2x
5
+ x
3
1= 0.
Dumitru Neagu, Iai
Soluia 1. Avem: 2o
5
+ o
3
1 = 0 o
3
(2o
2
+1) = 1 o
3
=
1 2
1
2
+ o
0 < o
3
< 1
0 < o< 1 (s-a utilizat faptul c o = 0, zero nefiind soluie a ecuaiei date).
Soluia 2. Dac x s 0, atunci 2x
5
+x
3
1 s -1 < 0, deci x s 0 nu este soluie a ecuaiei
68
date. Dac x > 1, atunci 2x
5
+x
3
1 > 2 + 1 1 > 0, deci x > 1 nu este soluie a ecuaiei date.
Cum numrul o este soluie a acestei ecuaii, rezult c o e (0,1).
VIII.18. Dac x,y e [0,1], atunci avem:
x+ ys
2 2 2 2
1 1 1 1 1 1 1 1 y x y x + + + .
Constantin Cocea, Iai
Soluie. Ridicnd inegalitatea dat la ptrat (avem voie!), obinem: x
2
+y
2
+2xy s
(1+
2
1 y -
2
1 x - ) 1 )( 1 (
2 2
y x ) + (1-
2
1 y +
2
1 x - ) 1 )( 1 (
2 2
y x )+2xy,
echivalent cu (1-x
2
) + (1-y
2
) > 2 ) 1 )( 1 (
2 2
y x , adevrat conform inegalitii mediilor.
VIII.19. S se determine numerele naturale n > 2 tiind c mulimea

e

e N
n
x x
N x
1
/
2
} ,..., 2 , 1 { n este format dintr-un singur element.
Cristinel Mortici, Constana
Soluie. Fie n > 2 un numr cu proprietatea dorit i fie x e {1, 2, , n} unicul numr
pentru care n | x
2
+x+1. Cum n+1-x e{1, 2, , n} i n | (n+1-x)
2
(n +1-x)-1 (dup cum se
vede din egalitatea (n+1-x)
2
(n+1-x)-1= n(n-2x+1) + (x
2
x+1), din unicitatea lui x rezult
c n+1-x=x, deci x =
2
1 + n
i atunci n este impar: n = 2k+1, x = k+1. n consecin, n | x
2

x1 2k+1 | (k+1)
2
(k+1)1 2k + 1 | k
2
+k1 2k + 1 | 4k
2
+4k4 2k+1 | (2k+1)
2
5
2k + 1 | 5 k e {1, 5}. Se verific direct c numai n = 5 satisface cerinele problemei.
VIII.20. Fie cubul D C B A ABCD ' ' ' ' cu O' centrul feei D C B A ' ' ' ' . Calculai
tangenta unghiului dintre O B ' i D A ' .
Cristiana Constanda, elev, Iai
Soluie. Dublnd cubul ca n figur se obine paralelipipedul D C F E EFCD ' ' ' ' , n
care D A B F ' ' (ambele paralele cu A E ' ). Observm c <( D A ' , B O' ) = <( B F ' , B O' ) =
=< O B F ' ' =o. Pentru laturile triunghiului O B F ' ' avem: B F ' =a 2 , B O' =
=
2
6
' ' '
2 2
a
B B B O = + i F O ' ' =
2
10
' '
2 2
a
P F P O = + (unde p este mijlocul
muchiei C B ' ' ) .
Aplicnd teorema cosinusului n A O B F ' ' ,
obinem:
4
10
2
a
= 2a
2
+
4
6
2
a
-2 a 2
2
6 a
coso, de
unde coso =
6
3
. Ca urmare, sin o =
=
6
33
36
3
1 = i deci tgo = 11 .
69
Clasa a IX-a
IX.16. n ipoteza c ecuaia cu coeficieni reali 0 c bx ax x
2 3
= + + + are rdcinile
1
x ,
2
x ,
3
x reale, s se demonstreze echivalena:
0 x
i
> , 3 2, 1, i = , 0 a < , 0 b > , 0 c < , c ab < .
Adrian Corduneanu, Iai
Soluie. Conform relaiilor lui Vite, avem:
a x x x = + +
3 2 1
, b x x x x x x = + +
1 3 3 2 2 1
, c x x x =
3 2 1
(1)
Ecuaia din enun se poate scrie i sub forma c ab a x b x
2
= + + ) )( ( (2).
Dac 0 >
i
x , 3 2, 1, i = , atunci din (1) rezult c 0 < a , 0 > b i 0 c < . Tot din (1) se
poate deduce c 0 ) x (x a x
3 2 1
< + = + . De aici i din egalitatea obinut punnd
1
x n
locul lui x n (2) deducem c 0 c ab < .
Reciproc, s presupunem c sunt ndeplinite condiiile din enun privind coeficienii.
Din (1) avem 0 c x x x
3 2 1
> = i atunci nu toate numerele
3 2 1
x , x , x pot fi negative. Fie
0 x
1
> ; rezult c are loc i 0 x x
3 2
> (3). Pe de alt parte, din c ab < i din (2) cu
1
x n
locul lui x , se obine c 0 a x
1
< + . ns ) x (x a x
3 2 1
+ = + , deci 0 x x
3 2
> + (4). Din
(3) i (4) decurge c 0 x
2
> i 0 x
3
> .
IX.17. Artai c ecuaia } { } { } {
2 2 2
z y x = + are o infinitate de soluii n Q \ Z ({a}
reprezint partea fracionar a numrului real a).
R. Brbulescu i M.B. Ion, elevi, Lucian Tuescu, prof. , Craiova
Soluie (dat de un grup de elevi din Braov). Scriem egalitatea
2 2 2
5 4 3 = + sub
forma:
2 2 2
7
5
7
4
7
3
|
.
|

\
|
= |
.
|

\
|
+ |
.
|

\
|
n n n
,
*
N e n .
Deoarece:
2 2
7
3
7
3
|
.
|

\
|
=

|
.
|

\
|
n n
,
2 2
7
4
7
4
|
.
|

\
|
=

|
.
|

\
|
n n
,
2 2
7
5
7
5
|
.
|

\
|
=

|
.
|

\
|
n n
,
*
N n e ,
urmeaz c tripleta ( ) |
.
|

\
|
n n n
= z y, x,
7
5
,
7
4
,
7
3
este o soluie a ecuaiei din enun
*
N n e .
IX.18. Determinai funciile f: R R pentru care avem: x 3x) + 3x + f(x
2 3
s s
3f(x) + (x) 3f + (x) f
2 3
e x R. Generalizare.
Gabriel Popa, Iai
Soluie. Vom rezolva direct urmtoarea problem mai general:
Dac : R R este o funcie surjectiv i strict cresctoare, s se determine funciile
f: R R pentru care: ( ) ( ) ( ) x f x x f( s s ) , e x R. (1)
Din ipotez, este inversabil i avem ( ) ( ) t x t x
1
= = Atunci, innd seama i
de monotonia lui , au loc:
70
(1)
( ) ( )
( ) ( )
( ) ( )
( ) ( )
( ) ( ) ( )
1 1
1
1
,
,

= e s s

e s
e s

e s
e s

f x x f x x f
x , x f x
t , t t f
x , x f x
x x x f
R
R
R
R
R
.
n cazul particular considerat, ( ) 1 1) + ( = 3 + 3 + =
3 2 3
x x x x x i este evident strict
cresctoare i surjectiv, deci , 1 1 x = (x) = f(x)
3 1
+

e x R.
IX.19. Artai c n orice triunghi are loc inegalitatea: (a
8
+b
8
+ c
8
+3) R
4
> 8r
2
a
2
b
2
c
2
.
Mihai Bogdan Ion, elev, Craiova
Soluie. Se tie c n orice triunghi au loc r R 2 > i
2
16S c b a
4 4 4
> + + , cu egalitate
pentru triunghiul echilateral. Atunci:
( ) =
|
.
|

\
|
+ + >
|
|
.
|

\
|
+
+
+
+
+
= + + +
4 8 8 8 4
8 8 8
4 8 8 8
2
2
1
2
1
2
1
2 3 R c b a R
c b a
R c b a
( )
2 2 2 2 2 2
2
2 2 2
4 2 4 4 4 4
8 4 2 16 2 2 c b a r r R
R
c b a
R S R c b a = > > + + = .
IX.20. Pe laturile AB i AC ale tringhiului ABC se consider punctele M, respectiv N,
astfel nct BM CN = k (constant). Dac B i C sunt fixe i ( ) A m

este constant, s se
afle locul geometric al mijlocului segmentului MN.
Ctlin Calistru, Iai
Soluie. Fie P, Q, R, S respectiv mijloacele
segmentelor [MN], [BC], [BN], [CM]. Laturile
patrulaterului PRQS sunt linii mijlocii n triunghiurile
NMB, BNC, CMB, respectiv MNC i cum BM CN
=k, rezult c PRQS este romb de latur
2
k
. n plus,
unghiurile acestui romb au msuri constante (o,
respctiv t-o), deci indiferent de poziia punctului A,
diagonala PQ a rombului are lungime constant
2
cos
o
= k QP
. Cum Q este fix, rezult c P descrie
un arc de cerc de centru Q i raz
2
cos
o
= k r
, delimitat de dreptele CA
1
i CA
2
corespunztoare poziiilor limit ale punctului A pe arcul capabil de unghi o (BA>k, CA>k).
Dac A descrie arcul simetric din semiplanul inferior, P parcurge un alt arc de cerc,
simetricul primului fa de BC.
Clasa a X-a
X.16. S se determine numrul funciilor { } { } 1 , 1 , 2 , 1 : n f ! cu proprietatea c
_
=
n
i 1
f(i) = k, N e n k, , n > 2, k < n.
71
Petru Asaftei, Iai
Soluie. Fie } {1,2,...,n X c mulimea acelor elemente a cror imagine prin f este 1.
Evident c C = X , altfel ( ) k n i f
n
i
> =
_
=1
. Notm X x card = . Atunci
( ) ( )
2
2 ) ( ) (
1
k n
x k n x x n x j f i f i f k
X j X i
n
i

= = + = + = =
_ _ _
e e =
.
Dac n i k au pariti diferite, nu exist funcii ca n enun. Pentru n i k de aceeai
paritate, funciile cutate sunt bine determinate de submulimea X; numrul lor va fi deci
egal cu numrul submulimilor cu
2
k n
elemente ale mulimii { } ,...,n ,2 1 , adic C
2
k n
n

.
X.17. Fie ( ) , 0 : R f definit prin :
f(x) =

e
e
Q R
Q
\ ,
,
x b
x a
x
x
a,b e N\{0,1}, a = b.
Notm cu M
a,b
mulimea acestor funcii i cu I = { f e M
a,b
| f injectiv }, S ={ e f M
a,b
|
f surjectiv }.
1) I C = , S C = i I = S.
2) dac (a, b) = 1, atunci f nu-i nici injectiv i nici surjectiv.
3) este adevarat reciproca afirmaiei de la punctual 2)?
Dumitru Gherman, Pacani
Soluie. Vom demonstra c f surjectiv f injectiv
*
Q e - r astfel nct
r
a b = .
a) f surjectiv f injectiv. ntr-adevr, s presupunem prin absurd c f nu este
injectiv; atunci e -
1
x Q, i e -
2
x R\Q astfel nct f(x
1
) = f(x
2
), de unde
2 1
x x
b a = , adic
b x x
a
log
2 1
= , deci e b
a
log R\Q, prin urmare ecuaia ( ) b x f = nu are soluie, ceea ce
contrazice faptul c f este surjectiv.
b) f injectiv e -r Q
*
astfel nct
r
a b = . Vom proceda tot prin reducere la
absurd; presupunem c e = r a b
r
, Q
*
. Atunci e a
b
log Q i, ca urmare,
( ) ( ) 1 log
log
f a b a f
a
b
b
= = = . Cum f injectiv, urmeaz c 1 log = a
b
, imposibil.
c) e -r Q
*
astfel nct
r
a b = f surjectiv. Pentru ye( ) , 0 , considerm ecuaia
y f(x) = . Dac log
a
y eQ, atunci y x
a
log
1
= este soluie a acestei ecuaii; n caz
contrar, Q R \ log
1
e = y
r
x
a
este soluie a ecuaiei.
Acum cerinele problemei sunt imediate. Pentru a=2, b=4, avem e f I iar I=S
conform primei echivalene dovedite. Punctul 2) rezult din ultima echivalen, iar
reciproca sa este fals: de exemplu, e f I pentru a=2 i b=6, ns ( 2,6 ) = 1.
72
X.18. Fie ABCD un patrulater convex, orientat pozitiv, ale crui vrfuri au afixele a,
d c b , , .S se arate c ABCD este ptrat dac i numai dac ) ( a b i a d = i a+c=b+d.
Adrian Corduneanu, Iai
Soluie. Prin translaia a - z z , obinem patrulaterul D C B A ' ' ' ' congruent cu
ABCD, unde ( ) 0 A' , ( ) a b B ' , ( ) a c C ' , ( ) a d D ' . Avem: ABCD ptrat D C B A ' ' ' '
ptrat D C B A ' ' ' ' paralelogram i D O ' se obine din B O ' printr-o rotaie de unghi
2
t
n
sens direct ). ( ) ( a d a b a c + = i ) ( a b i a d = a+c=b+d i. ) ( a b i a d = .
X.19. Fie x, y, z, a, b, c eC. Notm = xa + yb + zc, = xb + yc + za i = xc + ya
+ zb. S se arate c dac numerele complexe , , sunt afixele vrfurilor unui triunghi
echilateral, atunci cel puin unul din tripletele (x, y, z) i (a, b, c) reprezint afixele
vrfurilor unui triunghi echilateral.
Constantin Cocea, Iai
Soluie. Concluzia problemei se obine imediat folosind urmtoarele observaii :
(i) u, w, z sunt afixele vrfurilor unui triunghi echilateral dac i numai dac u
2
+
w
2
+ z
2
uw uz wz = 0;
(ii) Are loc relaia (ce se poate verifica printr-un calcul de rutin):
( )( ).
2 2 2 2 2 2 2 2 2
ac bc ab c b a yz xy xy z y x + + + + + + + = + + + + + | o o| | o
X.20. Stabilii natura triunghiului n care au loc simultan relaiile:
2
1 cos cos + B A
= 2
A cos 2
+ 2, 2
1 cos cos + + C B
= 2
B cos 2
+ 2, 2
1 cos cos + + A C
= 2
C cos 2
+ 2.
Neculai Roman, Mirceti (Iai)
Soluie. Cu notaiile
A
x
cos
2 = , y = 2
B cos
i z = 2
C cos
, ipoteza problemei se scrie
2 2
2
+ = x xy , 2 2
2
+ = y yz , 2 2
2
+ = z zx . Avem:
2 2
2 y x xy + s i analoagele i atunci
se obine c
2
2 y s ,
2
2 z s ,
2
2 x s . Deci, B cos 2 1 s , C cos 2 1s , A cos 2 1 s , de unde
m( A

) s60, m( B

) s60, m(C

) s60. Deoarece m() + m( B

) + m() = 180,
inegalitile precedente se transform n egaliti, adic triunghiul ABC este echilateral.
Clasa a XI-a
XI.16. Fie numerele e b a, C i matricele
n
M B A e , (C) astfel nct
n
I bBA aAB = + ,
I
n
fiind matricea unitate de ordin 1 > n . S se demonstreze c ( ) 0 det = BA AB sau exist
} 1 | { = e = e
n
n
z C z U e nct 0. b a = + e
Dan Popescu, Suceava
Soluie. Din ipotez obinem c ( ) ( )
n
I BA b a BA AB a = + + , deci are loc egalitatea
( ) ( )BA b a I BA AB a
n
+ = . Analog ( ) ( )AB b a I BA AB b
n
+ = . Pe de alt parte,
( ) ( ) tBA I tAB I
n n
= det det e t C, cci matricele AB i BA au acelai polinom caracte-
ristic. Rezult c ( ) ( ) ( )( ) ( ) BA AB b BA AB a = det det . Presupunnd c , 0 ) det( = BA AB
73
urmeaz c
n n
b a ) ( = , relaie care mpreun cu ipoteza asigur c ab = 0 (deci b = 0)
i atunci ( ) 1 =
n
b a , ceea ce ncheie demonstraia.
XI.17. Fie
2 1
0 x x < < i ( ) 1 , 0 , e | o . S se arate c irul ( )
1 > n n
x dat de
o o
+
=
1
2 1 2 1 2 n n n
x x x ,
| |
+
=
1
1 2 2 2 2 n n n
x x x , 1 > n , este convergent.
Gheorghe Costovici, Iai
Soluie. Pornind de la observaia c
2 4 3 1
x x x x 0 < < < <
,
se arat prin inducie
matematic faptul c
2n 2 2n 1 2n 1 - 2n
x x x x 0 < < < <
+ +
, 1 > n . Atunci subirul ( ) x
1 n 1 - 2n >
este cresctor i mrginit superior de x
2
,deci este convergent la l
1
, subirul ( ) x
1 n 2n >
este
descresctor i mrginit inferior de x
1
, deci este convergent la l
2
i n plus
2 1
l l s . Trecnd
la limit n relaiile de recuren, obinem c l l = l
- 1
2 1 1
o o
, deci
o 1
1
l =
o 1
2
l , adic l
1
= l
2
,
aadar
( )
1 > n n
x
este un ir convergent.
XI.18. S se demonstreze inegalitatea
( )
7
99
1
100 arcsin
99 1999
t

>
_
= k
k
e .
Lucian-Georges Ldunc, Iai
Soluie. Plecnd de la inegalitatea cunoscut ] 1 , 0 ( , sin e < x x x i folosind monotonia
funciei arcsin, rezult c x < arcsin x, x e (0,1 ), de unde
x x
e e >
arcsin
, ( | 1 , 0 e x . Pe de
alt parte, se arat imediat c
2
1
2
x
x e
x
+ + > , 0 > x , deci
2
1
2
arcsin
x
x e
x
+ + > , e x
(0,1) . Facem, pe rnd, 100 99 , , 100 2 , 100 1 ! = x i summ inegalitile obinute. Se
deduce c
( )
7
99
1
2
2
99
1
99
1
100 arcsin
1999 99
1200
1999 99
100 2
1
100
1
99
t

>

+ + >
_ _ _
= = = k k k
k
k k e .
XI.19. Determinai funciile f: RR continue n x
0
= 1 i care verific relaia:
, ) 1 ( ) 1 ( ) ( ) 1 ( 2 ) 1 (
2 2 2
= + + + x a x f a ax f a x a f e x R,
unde a>1 este un numr real dat.
Dumitru - Dominic Bucescu, Iai
Soluie. Cu substituia t x = 1 , ecuaia funcional dat devine:

, ) 1 ( ) ( ) 1 ( 2 ) 1 (
2 2
t a t f at f t a f = + + + e t R (1)
Evident c funcia identic 1
R
verific (1); fie f o alt soluie. Avem
, 0 ) )( 1 ( ) 1 )( 1 ( 2 ) 1 )( 1 (
2
= + + + t f at f t a f
R R R
R e t , echivalent cu
0 ) ( ) ( 2 ) = + t g at g t g(a
2
e t R, unde funcia g: RR, ) 1 )( 1 ( ) ( + = t f t g
R
este
continu n 0
0
= t . Definim R R : h , ( ) ( ) ( ) t g at g t h = , funcie continu n 0
0
= t ,
cu proprietaile: ( ) 0 0 = h , ( ) ( ) 0 = t h at h , e t R. Inductiv, obinem
74
( ) |
.
|

\
|
= |
.
|

\
|
= |
.
|

\
|
= t
a
h t
a
h t
a
h t h
n
1 1 1
2
! , e t R, e t N
*
i fcnd n , rezult c ( ) ( ) 0 0
1
lim
1
lim = = |
.
|

\
|
= |
.
|

\
|
=

h t
a
h t
a
h t h
n
n
n
n
, e t R, deci
( ) ( ) 0 = t g at g , e t R. Relund raionamentul anterior obinem c ( ) = |
.
|

\
|
=

t
a
g t g
n
n
1
lim
( ) c h = = 0 , deci c g + =
R
1 ( ) R e c . Reciproc, aceste funcii verific evident relaia (1).
XI.20. ntr-un plan dat se consider punctele fixe A, A' i punctul mobil P. Se noteaz
cu P' proiecia punctului P pe mediatoarea segmentului A A' i cu P" simetricul lui P fa
de dreapta A A'. S se afle locul geometric al punctului P tiind c dreapta P'P" este
paralel cu dreapta lui Euler a triunghiului PAA'.
Paraschiva Brsan, Iai
Soluie. Fie { } G = POP'P", unde O este mijlocul lui AA'. Din AOGP' ~ APGP",
obinem
2
1
=
' '
'
=
P P
P O
GP
OG
, deci G este centrul de greutate al APAA'. Cum dreapta lui Euler
conine punctul G, rezult din ipotez c aceasta esta chiar P'P". Deoarece centrul cercului
circumscris A AA' P este intersecia dreptei lui Euler cu o mediatoare, urmeaz c P' este
punctul din plan egal deprtat de P, A, A'. Atunci P'P
2
=P'A
2
=P'O
2
+OA
2
. Raportnd
planul la un reper ortogonal cu originea n O i avnd pe AA' drept ax a absciselor, fie
( ) y x, coordonatele lui P. Relaia precedent conduce la
2 2 2
a y x = , unde AA'=a. Deci
locul geometric al punctului P este hiperbola echilater de semiaxe a i avnd ca axe de
simetrie dreptele AA' i mediatoarea segmentului [AA'].
Clasa a XII-a
XII.16. Cercetai dac exist funcii continue f : R
*
R
*
pentru care
-
e = + R x x F x F x x F ), / 1 ( ) ( 2 ) / 1 2 ( ,
unde F este o primitiv a funciei f.
Gabriel Popa, Iai
Soluie. Pentru nceput, s observm c relaia din enun are sens, ntruct
0 / 1 2 = + x x
*
R e x , adic putem defini ) / 1 2 ( x x F + . Cum F este derivabil, derivnd
membru cu membru relaia dat, se obine: = + ) / 1 2 ( ) / 1 2 (
2
x x f x
. ), / 1 ( ) ( ) / 2 ( ) / 1 ( ) ( 2
* 2
R x x f x F x x F x f e = Lund n aceast egalitatea e =1 x R
*
, obinem
c f f Im 0 ) 3 ( e = , deci nu exist funcii f cu proprietile cerute.
XII.17. Dac funcia f : RR
*
este continu, impar i periodic de perioad
principal T, calculai
)
e + =
T
k dx kx x f f I
0
, ) ) ( ( Z .
Dumitru Gherman, Pacani
Soluie. Cu schimbarea de variabil x=T- u obinem:
75
) ) )
= = + = + =
0
0 0
) ) ( ( ) ) ( ( )) ( ) ( (
T
T T
du ku u f f du kT ku u f f du u T k u T f f I
)
= = + =
T
I I du ku u f f
0
0 ) ) ( ( .
XII.18. Calculai
)
+
= dx
x
x
I
1
2668
2000
i
)
+
= dx
x
x
J
1
2668
666
, ) , 0 ( e x .
Lucian-Georges Ldunc, Iai
Soluie.
( )
( )
=
+
'

=
+
+
=
+
+
= +
) ) )
dx
x x
x x
dx
x x
x x
dx
x
x x
J I
2 / 1
/ 1
667
1
/ 1
/ 1
1
2
667 667
667 667
1334 1334
668 666
2668
666 2000
.
2
/ 1
arctg
1334
2
667 667
C
x x
+

=
( )
( )
)
+
+ +
+
=
+
'
+
= = C
x x
x x
dx
x x
x x
J I
2 / 1
2 / 1
ln
2668
2
2 / 1
/ 1
667
1
...
667 667
667 667
2
667 667
667 667
.
Formnd un sistem din relaiile obinute, aflm valorile lui I i J.
XII.19. Dac T > 0 este perioad pentru funcia continu f : ) , 0 [ R i
g : [0,T] R este continu, s se demonstreze relaia:
|
|
.
|

\
|

|
|
.
|

\
|
=
) ) )

T T T
n
dx x g dx x f
T
dx nx f x g
0 0 0
) ( ) (
1
) ( ) ( lim .
Dan Popescu, Suceava
Soluie. A se vedea nota D. Popescu i F. Popovici O generalizare a lemei lui
Riemann , n acest numr al revistei, p.12.
XII.20. Fie ) , ( G un grup comutativ de ordin n. S se arate c: =

=
[
2
1
1
) (
n
n
i
i
a
=
+

=
[
2
1
1
) (
n
n
i
i
a
e = , unde
1 2 1
, , ,
n
a a a e ! sunt elementele grupului G, e- elementul neutru.
Cristian Frsinaru, Iai
Soluie. Deoarece n G = , rezult c 1 , 1 , = = n i e a
n
i
(fie din teorema lui
Lagrange, fie cf. pb. R-4, cap.III, 3 din manualul n vigoare n cazul comutativ), deci
e a
n
n
i
i
=
[

=
) (
1
1
(1). Aplicaia G G f : , G x x x f e =

, ) (
1
, este injectiv i cum G este
finit, rezult c f este bijectiv. Atunci: } ,..., , { } ,..., , {
1
1
1
1
1
1 1

= =
n n
a a e G a a e i deci
are loc egalitatea
1
1
1
1
1
1 1
... ....

=
n n
a a e a a e , ceea ce implic e a
n
i
i
=
[

=
2
1
1
) ( (2). Din
(1) i (2), concluzia problemei urmeaz imediat.
76
Probleme propuse
1
Clasele primare
P.24. Aflai numerele a, b, c, d tiind c verific n acelai timp urmtoarele egaliti:
a + 3 = b, b + 3 = c, c + 3 = d, a + 3 = 10.
( Clasa I ) nv. Maria Racu, Iai
P.25. Un elev din clasa I, fixnd un numr din irul numerelor naturale, constat c
suma numerelor din faa lui nu este mai mic dect 55, iar suma aceasta adunat cu
numrul fixat nu depete pe 66. Despre ce numr este vorba?
( Clasa I ) Luminia Popa, elev, Iai
P.26. Pe trei borcane de compot, unul de ciree, altul de viine i al treilea cu amestec
de ciree i viine, toate etichetele au fost puse greit. Scond un singur fruct dintr-un
singur borcan, determinai coninutul fiecruia.
( Clasa a II-a ) ***
P.27. S se scrie numrul 31 folosind cele patru operaii aritmetice i numai cifra 3
(se cer cel puin dou soluii).
( Clasa a II-a ) Andrea Balla, elev, Braov
P.28. Cte pagini are o carte dac pentru paginarea ei s-a folosit cifra 9 de 117 ori?
( Clasa a III-a ) Crizantema Mironeanu, elev, Iai
P.29. Ioana i Alina au cules mpreun 165 de nuci. Ioana a cules mai puine nuci
dect Alina; ea face un calcul i observ c triplul diferenei dintre numrul nucilor culese
de ele reprezint tocmai numrul nucilor culese de Alina. Cte nuci a cules fiecare fat?
( Clasa a III-a) nv. Maria Racu, Iai
P.30. Artai c dintre oricare patru numere naturale diferite, mai mici dect 1000000,
se pot alege dou a cror diferen s se mpart exact la 3.
( Clasa a IV-a ) Roxana Bolocan, elev, Iai
P.31. O veveri descoper un alun ncrcat cu fructe i i face provizii pentru iarn
transportnd la scorbura sa alternativ: o dat dou alune, o dat trei alune. Dup ce
transport 47 de alune, face o pauz pentru a se odihni. S se calculeze ce distan a parcurs
veveria n total, dac de la alun la scorbura ei este o distan de x hm x dam x m, unde x are
ca valoare cel mai mic numr natural posibil.
( Clasa a IV-a ) nv. Mihai Agrici, Iai
P.32. Un printe i mparte averea astfel: la primul copil 10 milioane plus o cincime
din rest, la al doilea copil 20 de milioane plus o cincime din noul rest, la al treilea 30 de
milioane plus o cincime din noul rest i aa mai departe. S se afle suma mprit de
printe, precum i numrul copiilor, tiind c toi au moteniri egale.
( Clasa a IV-a ) Mihai Grtan, Iai
Clasa aV-a a 0 b
c 0 d
V.26. S se determine cifrele distincte i e * f
nenule a, b, c, d, e, f, g pentru care rezultatul g * *
nmulirii alturate este cel mai mare posibil: g * * * f
Ioan Scleanu, Hrlu
V.27. Trei apicultori au tras mpreun 700 kg miere de albine. Cnd au mprit
mierea, primul apicultor a luat jumtate, al doilea jumtate din rest, al treilea jumtate din

1
Se primesc soluii pn la data de 15. 01. 2003.
77
noul rest, apoi operaiunea se repet pn se mparte toat mierea. S se afle ct miere a
luat fiecare.
Ctlin-Cristian Budeanu, Iai
V.28. Artai c
2001 2001
1
2 3 + = N i
2002 2002
2
2 3 = N sunt numere divizibile cu 5.
Dorina Carapanu, Iai
V.29. S se afle numerele abc pentru care
2
b ac abc = .
Romana Ghi i Ioan Ghi, Blaj
V.30. Dac x
i
, i= 500 , 1 , sunt numere naturale nedivizibile cu 5, atunci numrul
2000
500
12
3
8
2
4
1
2000 ... 12 8 4 x x x x N + + + + = este divizibil cu 5.
Tamara Culac, Iai
Clasa aVI-a
VI.26. Fie c b a C c b a B c b a A 28 11 3 , 3 4 , 6 4 = = + = , unde a,b,c Z e . Dac
(A, B)=23, artai c (A, B, C)=23.
Cristiana Constanda, elev, Iai
VI.27. S se rezolve n Z sistemul
. 1 1 3 ; 1 ; 8 2 3 = s s + x y x y x .
Mihai Crciun, Pacani
VI.28. S se rezolve n N ecuaia
. 2 ... 4 2 2 ... ) 2 ( ) 1 ( ) 1 ( 3 2 2 1 n n n n n n + + + = + + + + +
Dumitru - Dominic Bucescu, Iai
VI.29. n triunghiul ascuitunghic ABC, bisectoarea interioar a unghiului B

intersecteaz nalimea din A n E. Fie Fe(DC astfel nct AE=EF. Artai c BE AF.
Tamara Culac, Iai
VI.30. Pe ipotenuza (BC) a triunghiului dreptunghic ABC se consider punctele N i
M astfel nct BN=AB, CM=AC. Dac P i Q sunt proieciile punctelor M i N pe dreptele
AN, respectiv AM, demonstrai c segmentele (MP), (NQ) i (PQ) se pot constitui n laturile
unui triunghi.
Ctlin Calistru, Iai
Clasa a VII-a
VII.26. Determinai aeQ tiind c + 2 a 2 eQ.
Gheorghe Iurea, Iai
VII.27. Determinai aeR astfel nct sistemul
1
2
2
2
1
2
1 2
2
1
2
1
4
) 1 ( ,
4
) 1 ( ,
4
) 1 ( x
a
x a x x
a
x a x x
a
x a x
n n n n n
= + + + = + + + = + + +

, ...
s admit numai soluii ntregi.
Ctlin Calistru, Iai
VII.28. Fie zece numere naturale nenule care au suma egal cu 55. S se arate c
printre ele exist trei care pot fi lungimile laturilor unui triunghi.
Adrian Zanoschi, Iai
VII.29. Fie ABCD un ptrat, O centrul su, iar M i P mijloacele segmentelor (OA),
78
respectiv (CD). S se arate c triunghiul BMP este dreptunghic isoscel.258
Constantin Cocea i Dumitru Neagu, Iai
VII.30. Fie ABCD un ptrat de latur 1 i punctele Me(AD), Ne(BC),
{P}=BMAN. Dac S
DCNPM
=
2
1
, demonstrai c
3
4
1 s + < BN AM i
9
4
s BN AM .
Emil Vasile, Ploieti
Clasa aVIII-a
VIII.26. Demonstrai c ecuaia 0 4 4 ) 1 (
2 2 2 2
= + + + t t x t x t are numai dou soluii
n ZZ.
Mihai Crciun, Pacani
VIII.27. Determinai x,yeZ pentru care fracia
2 2
y xy x
y x
+ +
+
este echivalent cu
19
5
(n legtur cu E: 9314* din G.M. 11-12/1987).
Gabriel Popa, Iai
VIII.28. Fie a,b numere naturale de pariti diferite. Aflai valorile lui n pentru care
S
n
=a
n
+ a
n-1
b + a
n-2
b
2
++ ab
n-1
+ b
n
este divizibil cu a+b.
Mihaela Predescu, Piteti
VIII.29. Se consider piramida triunghiular regulat VABC cu latura bazei a, iar
muchia lateral 2a. Fie M mijlocul lui (VA), iar N un punct pe (VB) astfel nct VN=
4
3a
.
Aflai distana de la V la planul (MNC).
Adrian Corduneanu, Iai
VIII.30. Fie A, B, C, D patru puncte necoplanare astfel nct 73 4 = AB ,
29 4 = CD . Notm cu E, F mijloacele segmentelor (AB), respectiv (CD). S se arate c
mijloacele segmentelor (AF), (BF), (CE), (DE) sunt vrfurile unui paralelogram i s se
calculeze aria acestuia tiind c are o latur de lungime 194 .
Romana Ghi i Ioan Ghi, Blaj
Clasa a IX-a
IX.26. Dac ( ) e 0, a , s se rezolve ecuaia
| |
| |
{ }
{ }
x
a
x
x
a
x
+ = + . Discuie.
D. M. Btineu-Giurgiu, Bucureti
IX.27. S se determine funciile f, g :
| ) | ) 0 0 , , , unde g este surjectiv i
aditiv i ( ) ( ) ( ) ( ) ( ) y g x f x f g y g + = + ,
| ) x y , , e 0 .
Ioan Scleanu, Hrlu
IX.28. S se determine funciile f : R R pentru care (

D ! D D
ori n
f f f ) ( ) o + = x x ,
R e o fixat, iar funcia
R
1 = f g este monoton.
Mihail Bencze, Braov
IX.29. S se arate c n orice triunghi ABC are loc inegalitatea
79
) 3 (
2
3
3
3 3 3
abc p p
r
R
h
l
h
l
h
l
c
c
b
b
a
a
s + + .
Viorel Cornea i Dan tefan Marinescu, Hunedoara
IX.30. n patrulaterul ABCD considerm punctele R i S pe diagonala BD, n
interioarele triunghiurilor ABC, respectiv ACD. Notm { } AB CR M = , { } BC AR N = ,
{ } CD AS P = i { } AD CS Q = . tiind c 4
2
2
2
2
2
2
2
2
= + + +
QA
DQ
PD
CP
MC
BM
MB
AM
, s se
arate c 4 = + + +
n
n
n
n
n
n
n
n
QA
DQ
PD
CP
MC
BM
MB
AM
, N e n .
Ctlin Calistru, Iai
Clasa a X-a
X.26. Fie ecuaia x S x Sx mx m
4
1
3 2
1 0 + + = , unde S este aria unui triunghi
neechilateral ABC , iar S
1
este aria triunghiului A B C
1 1 1
determinat de punctele de
intersecie ale bisectoarelor interioare cu cercul circumscris triunghiului ABC . S se
determine meR tiind c ecuaia admite un numr impar de rdcini n ( ) 0 1 , .
Dumitru Gherman, Pacani
X.27. Fie
| ) r e 1, , { } r z z D s e = ; C i | | X P C e , ( ) c bX aX X P + + =
2
. S se
arate c dac ( ) D z P e , D z e , atunci D c b a e , , .
D.M. Btineu-Giurgiu, Bucureti
X.28. Rezolvai ecuaia 0 1 ) 1 2 ( ) 1 2 (
1 2
2
= + +
z z
z z , e z C\R.
Emil Vasile, Ploieti
X.29. Un motan scoate cu ajutorul unui pahar un numr de petiori dintr-un acvariu.
Ci petiori trebuie s conin acvariul astfel nct motanul s aib matematic sperana c
va scoate 5 dintre ei?
Gabriel Popa, Iai
X.30. Fie { } n M ,..., 2 , 1 = . S se afle numrul de k-uple ( )
k
A A A ,..., ,
2 1
de submulimi
ale lui Mastfel nct M A
k
i
i
=
=
*
1
i Card

k
i
n l l A
i
1
, ) (
=
s = fixat.
Lucian-Georges Ldunc, Iai
Clasa a XI-a
XI.26. Fie ( ) R
n
M Ae . Dac A n A A A A
t t
det 2 ) ) ( ( Tr
* *
= + , atunci
*
A A
t
= .
Iuliana Georgescu i Paul Georgescu, Iai
XI.27. Fie | ) 1 , 0 e a i ( )
0 > n n
x un ir de numere reale astfel nct
( )
)
`

+ s

2
1
2 2
1
2
2
1
, max
n n n n
x x x a x ,
*
N e n .
Artai c irul ( )
0 > n n
x este convergent i determinai limita sa.
Aurel Muntean, Sibiu
80
XI.28. S se determine R e p pentru care limita irului ( )
1 > n n
a definit prin termenul
general
_
=
+
=
n
k
p
n
k k
n
a
1
2
1
este finit i nenul.
Constantin Chiril, Iai
XI.29. S se arate c 1 1
1
.....
2
1
1
ln ln
lim =
|
|
.
|

\
|
+ + +

n
n n n
n
.
Marian Tetiva, Brlad
XI.30. Fie R R : f o funcie discontinu i care are proprietatea lui Darboux. Dac
exist o funcie R R R : g astfel nct ( ) ( ) ( ) y x f g y x f , = + , pentru orice R e y x, ,
atunci funcia f nu are limit la .
tefan Alexe, Piteti
Clasa a XII-a
XII.26. Se consider mulimea

e
|
|
|
|
.
|

\
|

+

= A a
a a
a a a
a a a
M ;
1 2 2
2 1
2 1
2 2
2 2
, unde A = Z
sau A = Q sau A = R. Artai c ( ) , M este grup; este acesta izomorf cu ( )
+
,
*
A ?
Gheorghe Costovici, Iai
XII.27. Fie ( ) , G un grup cu ( ) { } e G Z = i H un subgrup netrivial al lui G . S se
demonstreze c exist H G y x \ , e ,
1
= y x , astfel nct H xy e i H yx e . Dai exemplu
de grup care nu are aceast proprietate.
Ovidiu Munteanu, student, Braov
XII.28. Calculai
)
|
.
|

\
|
e
2
, 0 ,
t
x dx tgx
n
, pentru { } 4 , 3 , 2 e n .
Daniel Jinga, Piteti
XII.29. Fie R R : f o funcie continu i 0 > t . Pentru 0 , > b a , s se calculeze
( )
( ) ( )
_
=
+

|
|
.
|

\
|


n
k
b a
t
b a t
t
t t
n
k k n f
n
k k
1
1
1
lim .
Mihail Bencze, Braov
XII.30. S se arate c
( )
( )
)
(

e
+
+
1
0
2
2 16
, 2 ln
8
1
1 ln
2
t t e
dx
x
x
e
x
.
Cristian Moan, Craiova
Probleme pentru pregatirea concursurilor
1

A. Nivel gimnazial
G6. Dac un numr natural se poate scrie ca sum de dou ptrate perIecte, atunci
orice putere a sa se poate scrie, de asemenea, ca sum de dou ptrate perIecte.
```
G7. Artati c numrul a aa... (2001 ciIre) nu poate Ii ptrat perIect, oricare ar Ii ciIra
a n baza 10.
```
G8. Determinati nZ pentru care
1 3
28 ) 13 18 ( 3
+
+
n
n n
este Iractie reductibil.
Dumitru- Dominic Bucescu, Iayi
G9. Se dau trei Iisicuri de monede asezate vertical, asupra crora putem eIectua una
dintre operatiile O1: lum cele dou monede de deasupra unui Iisic si le asezm peste altul,
sau O2: lum cele dou monede de deasupra unui Iisic si le asezm cte una peste Iiecare
dintre celelalte dou Iisicuri.
a) Gsiti o conditie necesar pentru ca, dup un numr de operatii, toate Iisicurile
s contin la Iel de multe monede;
b) Artati c aceast conditie nu este suIicient dac este permis o singur
operatie, ns este suIicient n cazul n care sunt permise amndou.
Gabriel Popa, Iayi
G10. Pentru nN, n 2, rezolvati ecuatia
1 ...
1
...
1 1
2 1 2 1
+ = + + + +
+
+ +
+
+
+
n x x x x
n
n
x
n
n
x
n
n
n n
.
Mihai Totolici, Galati
G11. Rezolvati n N` N`

ecuatia 5445
2 2
= + y x .
Daniela Iosub, eleva, Iayi
G12. S se determine n, mN* pentru care [ ] [ ] [ ] . ... 2 1
m
n n = + + +
Adrian Zanoschi, Iayi
G13. Artati ca numerele 18
n
si 2
n
18
n
, nN, au acelasi numr de ciIre.
Gheorghe Iurea, Iayi
G14. S se arate c nu exist nici un triunghi dreptunghic avnd catetele numere
rationale, iar ipotenuza egal cu 2001.
Constantin Cocea, Iayi
G15. S se arate c E(x,y,:) 3 , dac
E(x,y,:) 10 cos 6 sin 2 5 cos 4 sin 2
2 2
+ + + : : y y : : x x , x,y,:R.
Cristiana Artenie, eleva, Iayi
G16. Fie M un punct n interiorul triunghiului echilateral ABC astIel nct
MA
2
MB
2
MC
2
-
2
MBMC ; calculati msura unghiului C M B
`
. Generalizare.
Corneliu Bradateanu, Paycani
G17. Fie ABCD un patrulater convex ce nu are diagonalele perpendiculare, B1 si D1
proiectiile punctelor B, respectiv D pe AC, iar A1 si C1 proiectiile punctelor A, respectiv C
1
Se primesc solutii pn la data de 15.01.2003
81
pe BD. S se arate c
2
1 1
1 1

=
AC
BD
S
S
AA CC
DD BB
si ) , ( cos
2 2
AC BD S
ABCD

1 1 1 1
AA CC DD BB
S S
.
Claudiu-Stefan Popa, Iayi
G18. Fie ABC un triunghi cu m( ) A
`

90 . Pe latura (BC) se consider punctele M si


N astIel nct AM si AN s Iie simetrice Iat de bisectoarea unghiului A. Cercul circumscris
triunghiului AMN intersecteaz laturile AB si AC n E, respectiv F. Dac
{ } CE BF I =
si
{ } BC AI P =
, demonstrati c AP
2
BC
.
Florin Nicolaescu, Baly
G19. Fie A1A2A3 un triunghi echilateral nscris n cercul C(O,R) si cercurile Ci
(i1,2,3) de aceeasi raz r, tangente interior cercului C n vrIurile Ai corespunztoare. S
se arate c pentru orice PC(O,R) are loc relatia = + +
2
3
2
2
2
1
t t t constant, unde ti (i1,2,3)
este lungimea tangentei dus din P la cercul Ci.
Temistocle Brsan, Iayi
G20. S se arate c pentru orice alegere a 12 numere naturale consecutive nu se pot
numerota muchiile unui cub astIel ca suma numerelor aIlate pe trei muchii care au un vrI
comun s Iie aceeasi pentru toate vrIurile cubului (nu se numeroteaz dou muchii cu
acelasi numr). S se arate c este posibil numerotarea descris dac se aleg convenabil 12
numere dintre oricare 13 numere naturale consecutive.
Constantin Chirila, Iayi
B. Nivel liceal
L6. Fie
1
x
,
2
x
,

n
x
,
n
N \ 0,1}, numere reale cu proprietatea
+

2
2
1
1
x S
x
x S
x
, 1 ... =

+ +
n
n
x S
x
unde

=
=
n
i
i
x S
1
. Artati c
n
S
x S
x
x S
x
x S
x
n
n
2 3
2
3
2
1
3
1
.....

+ +

.
Razvan Barbulescu, elev, Craiova
L7. n triunghiul ABC, m( ) A
`
60 , considerm medianele CN, N B

si bisectoarele
BE, E C . Notm
{ } { } N B E C Q BE CN P = = ,
. Artati c punctele P si Q nu pot Ii
ambele pe naltimea din A.
Ioan Sacaleanu, Hrlau
L8. Fie triunghiul ABC si ABC M Int ,
( ) { }
1
, A M MBC C MA =
,
( ) = MCA C MB

{ }
1
, B M
,
( ) { }
1
, C M MAB C MC =
. S se arate c 6
1 1 1
+ +
MC
MC
MB
MB
MA
MA
.
Neculai Roman, Mirceyti (Iayi)
L9. Fie ABC un triunghi ascutitunghic cu c b a si
( ) , 0 , , w v u
, w v u .
Dac
( ) R w v u wGC vGB uGA + + = + +
, unde G este centrul de greutate al triunghiului, iar
R este raza cercului circumscris, atunci triunghiul ABC este echilateral. Generalizare
Paul Georgescu yi Gabriel Popa, Iayi
L10. a) Fie
n N
*
, 2 n . S se arate c exist o progresie aritmetic de numere
naturale care nu are nici un termen de Iorma
x
n
, x N.
82
b) Dac o progresie aritmetic de numere naturale contine un termen de Iorma
x
n
,
x N, atunci s se arate c progresia contine o inIinitate de termeni de aceast Iorm.
Adrian Zanoschi, Iayi
L11. S se rezolve n
N
*
ecuatia 174 2 3 3 2 + =
y x
.
Daniela Iosub, eleva, Iayi
L12. Fie ABCD un patrulater convex; notm O}ACBD, M mijlocul lui (AB), N
mijlocul lui (CD). Pentru propozitiile
ABCD P :
1
inscriptibil;
CD OM P :
2
;
AB ON P :
3
, s se arate c: a)
3 2 1
P P P
; b)
1 3 2
P P P
; c)
2 1 3
P P P
(n
legtur cu problema C.2265 din G.M. 3 2000).
Viorel Cornea yi Dan Stefan Marinescu, Hunedoara
L13. Fie P R[X],
n 1 - n
1 - n
2
n
1
a X a X a X a P(X) + + + + = ,
2 n
cu
0
1
> a
si cu
toate rdcinile pozitive si subunitare. S se arate c ( ) ( ) 0 1 1
1
2 1
> + +

n
n
a a a n .
Gheorghe Molea, Curtea de Argey
L14. Pentru
`
N n
considerm polinomul
( )
n X n n n
X
X
X P
n
n
+
+
+
=

2 2 2 2
1 1 1 1
2
.
a) Artati c zero este rdcin multipl de ordin
( )
2
1 n n
a acestui polinom;
b) Dac n este par,
n
P
nu are rdcini reale nenule, iar dac n este impar,
n
P
are o
singur rdcin real nenul, care este simpl si situat n intervalul
( ] 1 , 2
.
Temistocle Brsan, Iayi
L15. Fie

R \ Z si
( )
1 n n
c
un sir convergent de numere reale. S se arate c sirul
( )
1 n n
x
deIinit prin
{ }
n n
c n x + =
nu este monoton.
Iuliana Georgescu yi Paul Georgescu, Iayi
L16. a) Fie
b a <
si
[ ] [ ] { } monoton I ; b a, b a, : I = M
. Artati c exist
M f

cu
( ) x x f
, [ ] b a x , si c orice asemenea Iunctie nu are proprietatea lui Darboux.
b)Demonstrati c
M f
, [ ] b a c , astIel nct
( ) [ ] ( ) c b a c c f b a f(c) + = +
.
Stefan Alexe, Piteyti
L17. Fie A un numr real pozitiv si
[ ) [ ) , 0 , 0 : f
o Iunctie derivabil pentru care
( ) 0 0 = f
si ( ) ( ) x Af x f
n

,
[ ) , 0 x
, unde n este un numr natural dat, 1 n , iar
f f f f
n
= . Atunci
f
este identic nul.
Sorin Puypana, Craiova
L18. Fie
( ) Z
n
M A
, N n , 2 n astIel nct
sA I
n
+
este inversabil si
( )
1
+ sA I
n

( ) Z
n
M
pentru orice
{ }. 2 , 2 , 1 n s
a) S se arate c
kA I
n
+
este inversabil pentru orice Z k si ( )
1
+ kA I
n
( ) Z
n
M
;
83
b) Dac
n
A 0
2
= , s se arate c
{ } Z k kA I G
n
+ = ;
este grup n raport cu nmultirea
matricelor si s se determine toate subgrupurile lui G .
Marian Ionescu, Piteyti
L19. Fie H un subgrup al grupului altern
( ) ,
2002
A
. Dac

=
2000 2002 2001 1999 2 1
2002 2001 2000 1999 2 1

H
si H
1
,
2002
A
, s se arate c
2002
A H =
.
Lucian-Georges Ladunca, Iayi
L20. Fie R a , 1 > a . Se consider Iunctia
[ ] R a f , 1 :
de dou ori derivabil. S
se arate c dac Iunctia
[ ] R a g , 1 :
,
( ) ( ) x f x x g =
este monoton cresctoare ,atunci.
( )
( )
at
t
t f
a a f
a

1
ln .
Marcel Chirita, Bucureyti
LISTA MEMBRILOR FILIALEI IASI a S.S.M.
`
- continuare din nr. 1/2000 si nr. 1/2001 -
68. ANDONE Elena - Scoala 'Vasile Conta Iasi
69. GRIGORAS Julieta - Liceul Teoretic 'Vasile Alecsandri Iasi
70. TIMOHE Gheorghe - Liceul Teoretic 'Emil Racovit Iasi
71. TRANDAFIR Ilie - Liceul Teoretic 'Emil Racovit Iasi
72. BRSAN Paraschiva - Liceul Teoretic 'Garabet Ibrileanu Iasi
73. BRSAN Temistocle - Universitatea Tehnic 'Gh. Asachi Iasi
74. BRSAN Mircea - Universitatea 'Al. I. Cuza Iasi
75. CARAPANU Dorina - Liceul Teoretic 'Garabet Ibrileanu Iasi
76. MIRON Nicu - Liceul Teoretic 'Emil Racovit Iasi
77. DUCA Alina Nicoleta - Liceul de Arte 'Octav Bncil Iasi
78. CADIS Mihaela Narcisa - Grupul Scolar Energetic Iasi
79. PALAGHIU Maria - Grupul Scolar Energetic Iasi
80. PLESU Veronica - Scoala 'Titu Maiorescu Iasi
81. COVALCIUC Margareta - Scoala 'Titu Maiorescu Iasi
82. COVALCIUC Dumitru
83. MIHALACHE Vasile - Scoala 'Titu Maiorescu Iasi
84. SIMIRAD Constantin - Colegiul National 'C. Negruzzi Iasi
85. SIMIRAD Cristina - Scoala nr. 10 Iasi
86. PAVLIUC Cristina - Liceul Teoretic 'Al. I. Cuza Iasi
87. HAIVAS Mihai - Institutul de cercetri Iasi
88. SUSANU Gelu - Liceul Teoretic 'Gr. Moisil Iasi
89. GEORGESCU Iulia - Liceul Teoretic 'Gr. Moisil Iasi
*
Lista va Ii continuat n numerele urmtoare.
84
85
Pagina rezolvitorilor
BRAOV
coala general nr.5. Clasa a V-a. POSTEUC Raluca: P(12,21), V(21-23);
POSTEUC Bogdan: P(12,21), V(21-23).
Liceul Teoretic N. Titulescu. Clasa a IX-a. BORO Olivia-Dana: VII.16, VIII(17,
23,24), IX.17; BOSCORNEA Ionu Bogdan: VII.16, VIII(17,23,24), IX.17; BURSUCANU
Lucian: VII.16, VIII(17,23,24), IX.17; CENU Nicoleta Veronica: VII.16, VIII(17,23,24),
IX.17; EDU Vladimir: VII.16, VIII(17,23,24), IX.17; GHEORGHE Liana Elena: VII.16,
VIII(17,23,24), IX.17; HALIPPA Andra: VII.16, VIII(17,23,24), IX.17; LITR Anatolie:
VII.16, VIII(17,23,24), IX.17; MOARCS Liviu George: VII.16, VIII(17,23,24), IX.17;
ONCIOIU Cristina: VII.16, VIII(17,23,24), IX.17; PAN Andreea: VII.16, VIII(17,23,24),
IX.17; PASCAL Andreea: VII.16, VIII(17,23,24), IX.17; PLATON Alexandru: VII.16,
VIII(17,23,24), IX.17; RCHITEANU Lavinia: VII.16, VIII(17,23,24), IX.17; RUSEN
Adrian Nicolae: VII.16, VIII(17,23,24), IX.17; OIMU Adelina: VII.16, VIII(17,23,24),
IX.17; TUDOR Ioana: VII.16, VIII(17,23,24), IX.17.Clasa a X-a. BRAOVEANU Adina:
VIII.24, IX.23, X(20,22-24); CHELMEA Andreea: VIII(17,24), IX.23,X(22,23); CHI
Cristina: VIII.24, IX.23, X(20,22-24); CURCAN Diana: VIII.24, IX.23, X(20,22-24);
DINU Cristina: VIII.24, IX.23, X(20,22-24); DIOR Mihaela: VIII(17,24), IX.23,
X(22,23); ELEKE Alexandra: VIII(17,24), IX.23, X(22,23); EPURE Alexandru: VIII.24,
IX.23, X(20,22-24); EZARU Lucian: VIII.24, IX.23, X(20,22-24); GAVRIL Ramona:
VIII(17,24), IX.23, X(22,23); HOOLEANU Cristina: VIII.24, IX.23, X(20,22-24);
JEGAN Alina: VIII(17,24), IX.23, X(22,23); LUNGU Cezar: VIII(17,24), IX.23, X(22,23);
NEAGOE Lcrmioara: VIII(17,24), IX.23, X(22,23); OANCEA Vlad: VIII.24, IX.24,
X(20,22,23); OSTAFI Andrei: VIII.24, IX.23, X(20,22-24); PAHONU Radu: VIII(17,24),
IX.23, X(22,23); PETRIORU Doris-Georgiana: VIII.24, IX.23, X(20,22-24); PUCAU
Andreea: VIII.24, IX.23, X(20,22-24); PUCOIU Lucian: VIII(17,24), IX.23, X(22,23);
SAVEI Ionela: VIII(17,24), IX(17,23), X(22,23); TODORU Anca: VIII.24, IX.23,
X(20,22-24); UIC Tatiana: VIII(17,24), IX.23, X(22,23); VRREANU Andrei:
VIII.24, IX.23, X(22-24). Clasa a XII-a. BCLEA Timea: X.24, XI(22,23,25), XII(20,24):
CPRARU Andrei: X(20,24), XI(22,25), XII.20; CERCELEA Rare: X.24, XI(22,25),
XII(20,24); DOBO Claudiu: X(22-24), XI(22,23,25), XII.24; DUMITRU Andrei:
X(20,24), XI(22,23,25), XII(20,24); FERSTOARU Valentin: X(20,23), XI(22,23,25),
XII(20,24); FORSEA Adrian: X(22-24), XI(22,23,25), XII(20,24); GHERGHE Mihai:
X(20,24), XI(22,25), XII.20; IORDACHE Florin: X(20,23,24), XI(22,23,25), XII(20,24);
MUOIU Rare: X(23,24), XI(22,23,25), XII(20,24); NSTAS Gabriel: X(22,23),
XI(22,25), XII.20; ROCA Silviu: X(20,24), XI(22,23,25), XII(20,24); ROTOPNESCU
Roxana: X(20,22-24), XI.25, XII.20; AGU Marian: X(23,24), XI(22,23,25), XII(20,24);
RU Ana-Maria: X(20,22-24), XI(22,25), XII.20.
CRAIOVA
Colegiul Na. Fraii Buzeti. Clasa a V-a. TUESCU Anca-tefania:V(21-
25),VI.22.
IAI
coala nr. 3 Alexandru Vlahu. Clasa a IV-a (nv. GALIA Gabriela). CHITIC
Andreea: P(14-18,20); CIOBANU Diana Luciana: P(14-18,20); SOFICU Corina-Maria:
P(14-17,19,20); VRABIE Corina-Roxana: P(14-16,19,20).
coala nr. 7 Nicolae Tonitza. Clasa a IV-a ( nv. MELINTE Rodica). ANDRONIC
86
Mihaela: P(14, 16-23); DUMITRIU Alexandru: P(14, 16-19).Clasa a IV-a (nv. TUDOSE
Elena). BOTEZ Alin: P(14-23); CHILCO Claudia: P(14-23); CHIRIAC Alexandra:P(14-
23); CIUBOTARIU Remus: P(14-18,21); IONESCU Mihai-Bogdan: P(14-21,23); ZORICI
Iulian: P(14-18,20-23). Clasa a IV-a (nv. PANAINTE Adriana). CURC Bogdan P(14-
18,21); ONU Adina: P(14,15,17,19-21); SRGHE Nicoleta: P(14,15,17, 19-21);
TRUFANDA Alexandru: P(14-17,20,21).
coala nr. 11 Otilia Cazimir. Clasa a III-a. (nv. PRIAL D-tru). TIBA Marius:
P(14-22).
coala nr.17 Ion Creang Clasa a VI-a BDI Vasilic Valentin: V(16, 19-21,
23),VI.17; CASIAN Manuela: P(9,12), V(16,18) VI.17; IFTODE Andreea P(9,12),
V(16,18,20,21); LUPU Andreea-Mdlina: P(9,12), V(20,21), VI.17; SOFRONEA
Gabriela: P.9, V(20,21), VI(16,17); TANANA Irina-Eliza: P(9,12), V(16,19,21) VI.17.
coala nr. 22 B.P. Hadeu Clasa a IV-a (nv. TRZIORU Iuliana). BARAN Ligia-
Maria P(14-23); BLUA Bogdan-Alexandru: P(14-23); CHIHAIA Mihai: P(14-18, 20-
23); MACOVEI Smaranda-Teona: P(14-23); PRODAN Andreea: P(14-23); RAI
Bogdan: P(14-23); SILION Ctlina: P(14-23); SPNU Drago: P(14-23).
coala nr. 23 Titu Maiorescu .Clasa a II-a (nv. CHIRIL Beatrice).
TUDORACHE Alexandru-Gabriel: P(7-11, 14-18, 20,21).
coala nr. 43 Dimitrie Sturdza. Clasa a IV-a (nv. OBREJA Rodica). OPRIC
Adelina: P(14,16,17,21,22)
Colegiul Naional C. Negruzzi. Clasa a IX-a. ARTENIE Cristiana: VII(16,22),
VIII(17, 18, 20-24), IX(16,18,22,24).
Liceul Teoretic Gr. Moisil Clasa a VII-a. COSTIN Ciprian : V(21-23, 25), VI(21-
23,25), VII(22,23,25), G(2,3). Clasa a XII-a. ALECSANDRESCU Ioana: XI(21-24),
XII.24.
Liceul Teoretic Garabet Ibrileanu Clasa a V-a . BABIUC Sanda: P(9,12,13,21),
V(16,20,21,23); BUDEANU tefana: P(9,13,22), V(16,20,21); FUIOREA Bogdan:
P(9,12,21-23), V.16; GAVRILESCU Ioana: P(10,19,20,22), V.16; TURCHIN Ariadna:
P(9,12,13,21-23), V(20,21); UNGUREANU Drago: P(9,10,13,21,22), V(20,21).
Clasa a VI-a. PETRUC Magda: P(9,12,13), V(19,21). Clasa a VII-a. CHIRVASE
Ariadna : V.23, VI(22,23,25), VII.22; JUVERDEANU George: V(21,23), VI(22,23,25),
VII.23; PRUTEANU Irina: : V.23, VI(22,25), VII(22,23); STANCIU Filip: V(16, 20, 21,
23), VI(18,19,22).
Liceul Teoretic M. Eminescu. Clasa a VII-a. AVRAM Mircea: V(21-23,25),
VI(23,25), VII.22; CIUCANU Radu: V(21-23,25), VI.25; DNIL MIHAI: V(21-23),
VI(22,23,25), VIII.23; EANU Ruxandra: V(16,21,23,25), VI.19, VIII.23; GRAMSCHI
Raluca: V(21,23,25), VI(19,25), VIII.23; TOFAN Andrei: V(21-23,25), VI(23,25);
TUDOSE tefan: V(16,20,21,23,25), VI.25; TURLIUC Clin Rare: V(22-25), VI(21,25);
Clasa a VIII-a. BUZDUGA Alexandru: VI(21,25), VIII(21,24,25).
Redacia revistei Recreaii matematice acord cte un premiu n cri elevilor:
COSTIN Ciprian (Lic. Teoretic Gr. Moisil): 1/2001(8pb), 2/2001(8pb), 1/2002(12pb).
CIUCANU Radu (Lic. M. Eminescu): 1/2001(8pb), 2/2001(6pb), 1/2002(5pb).
DNIL Mihai (Lic. M. Eminescu): 1/2001(5pb), 2/2001(7pb), 1/2002(7pb).
TOFAN Andrei (Lic. M. Eminescu): 1/2001(6pb), 2/2001(8pb), 1/2002(6pb).
TURLIUC Clin Rare (Lic. M. Eminescu): 1/2001(6pb), 2/2001(6pb), 1/2002(6pb).
Revista RECREAII MATEMATICE apare de dou ori pe an
(la datele de 1 martie i 1 septembrie) i se adreseaz elevilor, profesorilor,
studenilor i tuturor celor pasionai de matematicile elementare.
n atenia tuturor colaboratorilor
Materialele trimise redaciei spre publicare (note i articole,
chestiuni de metodic, probleme propuse etc.) trebuie prezentate ngrijit,
clar i concis; ele trebuie s prezinte interes pentru un cerc ct mai larg de
cititori. Se recomand ca textele s nu depeasc patru pagini. Evident, ele
trebuie s fie originale i s nu fi aprut sau s fi fost trimise spre
publicare altor reviste.
Problemele originale destinate rubricii Probleme propuse vor fi
redactate pe foi separate cte una pe fiecare foaie, cu enun i
demonstraie/rezolvare, fiind nsoite de numele autorului, coala i
localitatea unde lucreaz/nva.
Redacia revistei va decide asupra oportunitii publicrii
materialelor primite.
n atenia elevilor
Numele elevilor care vor trimite redaciei soluii corecte la exerciiile
i problemele din rubrica Probleme propuse vor fi menionate n Pagina
rezolvitorilor. Elevii vor ine seama de urmtoarele reguli:
1. Pot trimite soluii la minimum cinci probleme propuse n
numrul prezent i cel anterior al revistei; pe o foaie va fi redactat
soluia unei singure probleme.
2. Elevii din clasele VI-XII au dreptul s trimit soluii la
problemele propuse pentru clasa lor, pentru orice clas mai mare, din dou
clase mai mici i imediat anterioare. Elevii din clasa a V-a pot trimite
soluii la problemele propuse pentru clasela a IV-a, a V-a i orice clas mai
mare, iar elevii claselor I-IV pot trimite soluii la problemele propuse pentru
oricare din clasele primare i orice clas mai mare. Orice elev poate trimite
soluii la problemele de concurs (de tip G i L).
3. Vor fi menionate urmtoarele date personale: numele i
prenumele, clasa, coala i localitatea.
4. Plicul cu probleme rezolvate se va trimite prin pot (sau va fi
adus direct) la adresa Redaciei:
Prof. dr. Temistocle Brsan
Catedra de Matematic
Universitatea Tehnic Gh. Asachi Iai
Bulevardul Carol I nr. 11, 6600, Iai
(pentru Recreaii Matematice)
E-mail: tbi@math.tuiasi.ro
CUPRINS
Alin Spum .1
Petre Osmatescu (1925 2001).2
Academicianul Petru Soltan la 70 ani..3
Niels Henrik Abel 200 de ani de la natere4
ARTICOLE I NOTE MATEMATICE
P. MINU Asupra ipotezei lui Goldbach.5
C. COCEA Generalizarea teoremei de omologie a lui Barbilian.7
I. GEORGESCU, P. GEORGESCU Asupra unor iruri de integrale...9
D. POPESCU, F. POPOVICI O generalizare a lemei lui Riemann.12
M.TETIVA Cteva relaii metrice deduse vectorial....14
D.-T. MARINESCU, I. ERDEAN Inegaliti geometrice. Aplicaii...17
D. POPESCU Asupra unei clase de iruri recurente....20
T. BRSAN Un criteriu de concuren a dreptelor...23
I. SCLEANU Cteva consecine ale unei relaii a lui Gergonne....25
GH. COSTOVICI Unele iruri monotone cu limita e sau e
-1
..28
C.-T. POPA Studiu comparativ privind cteva medii uzuale.31
NOTA ELEVULUI
C. ANDREI, T. RUSU Aplicaii ale monotoniei mediilor n raport
cu ordinul lor..33
CHESTIUNI METODICE
D. GLEAT, G. POPA Funcii care admit / nu admit primitive..36
CHESTIUNI COMPLEMENTARE MANUALELOR
E. COHAL Construcii geometrice cu echerul....41
DIN ISTORIA MATEMATICII
S. IONESEI Teorema celor patru culori..43
MATEMATICA N CLASELE PRIMARE
P. ASAFTEI Introducerea operaiei de adunare la clasa I ..46
CONCURSURI I EXAMENE
Concurs de admitere 2001, Iai...48
Capacitate teste pregtitoare (I. SECRIERU, C.M. ROMACU)...51
Bacalaureat teste pregtitoare (G. MRANU)...56
PROBLEME I SOLUII
Soluiile problemelor propuse n nr.1/2001....61
Probleme propuse..76
Probleme pentru pregtirea concursurilor...81
Pagina rezolvitorilor...85

S-ar putea să vă placă și